飛行機って何で飛べるんですか3

このエントリーをはてなブックマークに追加
1ご冗談でしょう?名無しさん
前スレで一応、結論らしきものが出たような出てないような..そんな感じは
するのですが、前スレがパンクしまして、新スレを建てざるを得ませんでした。
で、一応新スレ建てました。まだ、意見のあるかたや、疑問のある方はどうぞ。
もし沈んだら放置されても構いません。関連サイトのURLは>>2をどうぞ
2ご冗談でしょう?名無しさん:01/09/06 09:40 ID:NrlOLdqU
関連サイト集

オリジナルスレ:「飛行機ってなんで空を飛べるんですか」
http://cheese.2ch.net/test/read.cgi?bbs=sci&key=988866699&ls=50
(一応HTML化されているようです。もしDirectなURL知っている人あれば
カキコ,キボンヌ)

継続スレ:「飛行機ってなんで空を飛べるんですか2」(大き過ぎて当分見れないでしょう。)
http://cheese.2ch.net/test/read.cgi?bbs=sci&key=992901156&ls=50
(まだHTML化されてないでしょう。未だ倉庫入りもしていないでしょう。)

今話題の新空気力学理論についての記事や各種資料

http://cheese.2ch.net/test/read.cgi?bbs=sci&key=992799218&ls=50
http://www.geocities.co.jp/Technopolis/4764/52/530302.htm
http://ikupara.pro.tok2.com/riron/_berunui.html
http://www.japa.or.jp/kouen/soukai1.pdf
http://homepage.mac.com/koichi_takasaki/lib/6thSSS.pdf
http://journal.msn.co.jp/worldreport.asp?id=010703s_suzuki&vf=1
(以上、中身NoCheckのコピペなので、ヘンなのも混じってるかも知れません。)

もし参照URLで他にもあれば、コピペキボンヌ
3ご冗談でしょう?名無しさん:01/09/06 09:40 ID:NrlOLdqU
前スレの最後の雰囲気だけお伝えします。

・分子運動論に基づき...という理由は、離散性が別に現れているワケではないので却下。流体で行きましょう
ということに。(中身スカスカの空洞だらけの、流体=気体とする。)

・争点は粘性が、ダランベールのパラドックスを引き起こし、揚力を失わせるのか、それとも粘性そのものが
(前縁の吹き上げが粘性により、後方に伝播する関係で低圧部が持続される)揚力に寄与するのか?
という議論
と、オリジナルスレからずっとある人が主張してきた、”翼と空気の粘着性が無視出来ないファクター
であるどころか、本質である”という説に対する反論
に二分されているよう。
・2次元の流体に基づく議論は、魑魅魍魎とした雰囲気になっており、このスレとしてはメジャーな議論では
無くなりつつある。

といった感じに見えたが、関係者の方で異論がありましたらよろしく。
では
4ご冗談でしょう?名無しさん:01/09/06 10:30 ID:U7Cq.6TY
>>3
新スレ&リンクをアリガトね。
でも、前スレの雰囲気は伝わってないぞ。

>・分子運動論に基づき...という理由は、離散性が別に現れているワケではないので却下。流体で行きましょう
>ということに。(中身スカスカの空洞だらけの、流体=気体とする。)

「離散性が現れないから却下」は無意味でしょ。ちゃんと説明できればそれで良い。
「理解できないから止めておく」ってことならわからんでもないが...
あと、2次元だろーが3次元だろーが「流体」で考えるかぎり、翼周りの空気の
流速分布と圧力分布をうまく説明できないってことだったよ。
これまで出てきた「説明」は全て破綻してる。

>・争点は粘性が、ダランベールのパラドックスを引き起こし、揚力を失わせるのか、それとも粘性そのものが
>(前縁の吹き上げが粘性により、後方に伝播する関係で低圧部が持続される)揚力に寄与するのか?

これは全然「争点」にはなってないよ。
ってゆーかパラドクスを誤解しているね。
「粘性が無ければ流体中を運動する物体に抗力が働かないけど、それは変」
ってのがダランベールのパラドクス。で、粘性を無視しなければ一撃で解決。
それと「吹き上げが後方に伝播する」なら「下降気流」じゃ無くて「上昇気流」が
発生するよ。何が言いたいの?

第3スレまでくるとは思わなかったけど、やるんだったら

 「流体力学で揚力を説明してみよう」

でしょ。同時到着説、曲げ派はダメ。空洞説もダメ。循環保存説もダメ。
今はネオ空洞説とネオ循環説待ちかな。
5ご冗談でしょう?名無しさん :01/09/06 11:10 ID:7Fhs9uV.

>「流体力学で揚力を説明してみよう」

ええっ〜、お得意の分子論での説明はどうなったの?
自己矛盾に陥ったから、スレが変わったドサクサに
まぎれて無かったことにするの???

分子論に拘るなら、偉そうなことを言う前に、自説を
ご披露頂きたいぞ。

個人的には、流体で説明がついて、ほぼ皆が納得した
ところで、それを分子論で説明できるか否かに話しを
進めることを勧めるが、貴君の意見は如何か?
6ご冗談でしょう?名無しさん:01/09/06 12:50 ID:U7Cq.6TY
>>5
こーゆー書き方はイケナイな。
分子運動論的な説明のどこが自己矛盾してるのかな? 指摘してごらん。
ドサクサにまぎれて逃げてるのはコテハン「前スレ27ではないが」だぜ。

>個人的には、流体で説明がついて、ほぼ皆が納得した
>ところで、

だから、流体で説明できてないよ。できたと思い込んでるだけ。
欠点を指摘されても、反論できないからそれを無視してるんでしょ。
7ご冗談でしょう?名無しさん:01/09/06 13:28 ID:7Fhs9uV.
5だけど、指摘したいのはやまやまだが、
貴君(=元スレ27氏)の分子運動論がわからんのよ。
っていうか貴君はほとんどまともに説明してないだろ。

急がずとも良いから、最初から最後まで貴君の
分子運動論を展開してみては如何?

ちなみに自己矛盾というのは前スレの後半で、空気
分子と翼には引力が働くといいつつ、突込みに耐え
られずに「低圧部だから斥力が働く」などと言った
事をもって判断している。これは忘れてあげるから、
新スレになったところで、ちゃんと最初から説明す
ればどう?

ちなみに俺は貴君の質問にはきちんと答えたつもり
だが?何を持って無視していると言っている?
前スレ(第2スレ)最後の書きこみは貴君か?
そこで書きこみができなくなったようだが、俺はその
最後の書きこみは見ることができていない。
8ご冗談でしょう?名無しさん:01/09/06 19:35 ID:nFElAx1s
俺も27派にはもっと詳しく説明して欲しいと思う。
なんで流れが曲がるかって事に対してこれまでに

>コアンダ効果は「粘着の条件」で説明できるよ。
>物体の表面に接している流体の流速=物体の速さになるってこで、
>流体が表面にくっついていること。
>一様な流れの中にカーブした物体を置けば、物体の表面の流速は
>ゼロだから流れはカーブに沿って曲がるでしょ。

>確かに「粘着の条件」は経験則だけど、気体分子が物質の表面に
>吸着しているのは事実。なぜ吸着するのかは、量子力学的に扱う
>必要があるので、ちょっと複雑だな。

位しか説明して無くないか?
経験則だとかちょっと複雑だとか言ってたら流体力学派と変わらんぞ。
そろそろ説明してちょ。
9ご冗談でしょう?名無しさん:01/09/06 20:38 ID:???
>>8

>経験則だとかちょっと複雑だとか

経験則なのは「粘着の条件」。分子が吸着しているのは「事実」。
ちょっと複雑なのは電子の交換相互作用。満足したか?
まだ質問があるなら答えるぜ。
10ご冗談でしょう?名無しさん:01/09/06 20:40 ID:???
>>8に付け加えると27(一人だから派はいらないか)は、曲がる理由
はおろか、低圧部と高圧部ができる理由、翼上面の流れが速くなる理由
そして、そもそも揚力が生ずる理由を分子論的にはほとんど何も説明し
てないね。>>7,>>8と同様に漏れも説明聞きたいな。
(しかし、これじゃ27もどこぞのコテハン「○○派」と変わらんな。
11ご冗談でしょう?名無しさん:01/09/06 20:44 ID:???

>>9みたいに、流体と分子論的見方をごっちゃにしてるところとか、
結局煙に巻いたような口答えしかせず、きちんと説明しないところ
とか?(藁
12ご冗談でしょう?名無しさん:01/09/06 21:21 ID:???
流体力学の説明に行き詰まったから、逆に質問攻勢に出たわけね。
でも、第1スレの説明でわからないのなら、理解するのは無理だと思うぞ。

一度に説明してもダメだろーから、少しずつわからないことを質問してみれば?
煽るコドモがいなくなったら教えてもらえるかもな。

あと、自分が理解できないのに「煙に巻かれた」と思うのは止めよ−ぜ (藁
13ご冗談でしょう?名無しさん:01/09/06 21:43 ID:???
>>7
>ちなみに自己矛盾というのは前スレの後半で、空気
>分子と翼には引力が働くといいつつ、突込みに耐え
>られずに「低圧部だから斥力が働く」などと言った
>事をもって判断している。

圧力があれば翼は空気から斥力を受ける。
空気に及ぼした力積の反作用が揚力。つまり空気から引力を受ける。

これがどーゆー状況なのか、よく考えてみれば。

「マクロに見ると翼と空気の間には引力も斥力も働かない」
ってウソを書いているよーでは、理解不能だろーけどな。
14ご冗談でしょう?名無しさん:01/09/06 21:49 ID:???
>>10
>低圧部と高圧部ができる理由、翼上面の流れが速くなる理由
>そして、そもそも揚力が生ずる理由を分子論的にはほとんど何も説明し
>てないね

してるけどねぇ。理解できなかっただけじゃないの?

>>11
>>9みたいに、流体と分子論的見方をごっちゃにしてるところとか、

どこがだ?

スレのレベル急降下中だぜ...
15ご冗談でしょう?名無しさん:01/09/06 22:22 ID:CA.P7R6c
7だけど、俺は君と違って、自分が全て正しいとは
思っていないし、自分の考え方に間違いがあって
それに対して合理的な指摘をされれば訂正するよ。

これは、単に言葉の問題で揚げ足を取り合ってる
気がする。前スレ(第2スレ)の最後の辺りにあ
った俺の説明に少し補足しつつ話すと、迎角を持
った翼の前縁部付近で空気が膨張するって言った
よね。

膨張した空気(=低圧の空気)は縮もうとするけど、
その空気の下端に翼があるから、翼が上向きの力
を受ける訳だと思う。これが揚力じゃないかな。

これを、マクロ的に見て翼と空気に引力が働くと
いうの。翼の上面(特に前縁付近)だけの話しだ
し、膨張する事によってのみ作用するんだけど。

言葉の使い方の問題だと思う。「翼と空気に引力が
働く」っていうと常にお互いが引き合うイメージが
あるだろう。間違ってるなら訂正するけど、君も
きちんと分子論で説明してくれよ。ちなみに俺は
分子論での説明の試み自体は否定してないぞ。
16ご冗談でしょう?名無しさん:01/09/06 23:11 ID:RZCLiZok
>>15
膨張しても引力は働かんだろ。斥力(圧力)が小さくなるだけだと思うけど。

>>27
とりあえず電子の交換相互作用ってのを解説キボン
こいつが翼と気体の間の引力の原因という理解でいいの?
17ご冗談でしょう?名無しさん:01/09/06 23:43 ID:???
>>15
間違いを指摘するから訂正してちょ。

>迎角を持った翼の前縁部付近で空気が膨張するって言ったよね

これは http://www.allstar.fiu.edu/aero/airflylvl3.htm のFig.4を見ても
膨張する流れの場にはなっていないのでは? 低圧になるのは別の原因でしょ。

>(=低圧の空気)は縮もうとするけど、その空気の下端に翼があるから、
>翼が上向きの力を受ける訳だと思う。

低圧であっても翼が空気から受ける力は斥力。

>言葉の使い方の問題だと思う。「翼と空気に引力が働く」っていうと
>常にお互いが引き合うイメージがあるだろう。

空気の運動量を変化させた力積の反作用が翼に働く揚力であるとするならば、
翼と空気の間に働いているのは引力。言葉の使い方の問題じゃないよ。

分子論で説明しているのは、それらの力が働く原因なんだけどな。
18ご冗談でしょう?名無しさん:01/09/06 23:44 ID:???
質点だとか、分子(モデル球?)だとか、惑星同士だとか、離散的なものを考える
時には引力・斥力って言い方するのは理解できる。したがって27派が分子論(
相互作用をする質点モデル?)を語るときにこの言葉を用いる事は理解できる。
しかし、空気を連続体(のみ)で考えているときにはこの言葉は出てこないんじゃ
ないか?この辺りごっちゃになってない?
19ご冗談でしょう?名無しさん:01/09/07 00:01 ID:???
>>16
>とりあえず電子の交換相互作用ってのを解説キボン

例えば水素分子で、水素原子同士を結合させる力。
まぁ、吸着の話は複雑で、簡単に説明するのはちょっと無理。

>こいつが翼と気体の間の引力の原因という理解でいいの?

これは翼に吸着している分子の説明。
離れている分子間に作用するのは Van der Waals 力(引力)。
分子が接近しすぎると斥力が作用する。
20ご冗談でしょう?名無しさん:01/09/07 00:01 ID:4OGWhTRk
>17
その図で言うと、翼上面の前方から1/3くらいの位置
で膨張する事になるでしょ。わかってると思うけど、こ
の図は翼の形を誇張して書いており、実際はもっと細長
いよね。さすればごくごく前縁付近で膨張すると言って
も良いと思うが。

以前に参考リンクで出ていた図で、前方方向に負圧が
働いていた図があるね。あれの理由はよく判らないが
(あの図正しいの?計算図だったよね。)、その点を
持って、この流体のコンセプト全体が間違っていると
は言えないと思うが。

あと、>>18も言っているように、こちらは流体で考えて
いるのでやっぱり引力/斥力って概念は無いと思うよ。
もちろん、そちらは相互作用をする質点モデルで考えて
いるので、そちらの説明をする限りにおいては使って
頂いて構わない。でも連続体、流体で考えているとき
に「引力/斥力」を持ち出すのは適当でないと思う。
21ご冗談でしょう?名無しさん:01/09/07 00:27 ID:???
>>20
>その図で言うと、翼上面の前方から1/3くらいの位置
>で膨張する事になるでしょ。

では、この図の翼前縁下面側で流線が開いているのは膨張かい?
ここで圧力が減少すると言っているのかい。

>あと、>>18も言っているように、こちらは流体で考えて
>いるのでやっぱり引力/斥力って概念は無いと思うよ。

密閉された容器内の気体は、容器の内壁面から内側に向かう法線応力を
受けているので、外側に流れていかないんでしょ。この法線応力は斥力。
22ご冗談でしょう?名無しさん:01/09/07 00:32 ID:/FFHbcaI
>>19
なるほど。
すると流れが曲がるのはVan der Waals 力によるって事かな?
翼の上面でVan der Waals 力による引力が強くなるのはどういう
仕組みなのだろうか。分子間の距離が詰まってるとか?
もうちょっと詳しくそのあたりについて聞きたいんだけど。
23ご冗談でしょう?名無しさん:01/09/07 00:47 ID:???
>>22
>すると流れが曲がるのはVan der Waals 力によるって事かな?

そう。

>翼の上面でVan der Waals 力による引力が強くなる

引力が強くなってるわけじゃないよ。
翼の上面に沿った流れは、上昇気流→下降気流と変化するでしょ。
つまり鉛直方向の運動量が大きく変化したわけで、大きくなった
のは翼が空気に対して作用した力積。力そのものではない。
24ご冗談でしょう?名無しさん:01/09/07 01:09 ID:???
>>23
ちょっとコンフュージングかな。
書きたかったのは、流れが曲がる山の頂上付近で力積が最大になる
ってこと。圧力分布ともだいたいコンシステントでしょ。

今夜は出血大サービスだぜ (藁
25ご冗談でしょう?名無しさん:01/09/07 01:36 ID:/FFHbcaI
>>23、24
なんで力が大きくならずに力積が大きくなるの?
>翼の上面に沿った流れが上昇気流→下降気流と変化する
その為には力が強くなってないとおかしいと思うんだけど。
26ご冗談でしょう?名無しさん:01/09/07 08:14 ID:A0GnxBI6
20だけど、

>21
>この図の翼前縁下面側で流線が開いているのは膨張かい?
>ここで圧力が減少すると言っているのかい。

そこで膨張になっているかどうかは知らない。圧力減少に
なっているのかも知らない。逆に教えてくれよ。どうなっ
ているんだい?

ただ、この質問と流体コンセプトでの説明との関連が良く
判らんが...。その質問の答えがダイレクトに、この流体
コンセプトが正しい間違っているの判断にはならないと思
うけど。

>この法線応力は斥力。

相互作用をする質点モデルで考える限りにおいては
「引力/斥力」という言葉を使って構わないが、
連続体/流体で考えているとき に「引力/斥力」を
持ち出すのは適当でないと思う...と繰り返さざる
を得ないね。
27ご冗談でしょう?名無しさん:01/09/07 08:35 ID:???
>>25
>なんで力が大きくならずに力積が大きくなるの?

空気の分子は沢山あるよ。個々の相互作用の大きさは一定でも、
分子の数が増えればその和は大きくなる。圧力も力積の反作用
だけど、高圧だと単位時間に壁に衝突する分子の数が増えるから
力積が大きくなる。このとき温度が同じであれば個々の分子の
衝突による運動量の変化(の平均)は同じ。

>その為には力が強くなってないとおかしいと思うんだけど。

ええと、滑り台をすべり降りるときには重力が増えているのかい?
重力は常に作用しているけど、抗力と釣り合っているので落下しない。
で、その釣り合いのバランスを崩すと動くんでしょ。
分子間力も同じ。常に作用しているけど、他の力と釣り合って
平衡状態になっているんだな。翼でそのバランスを崩すと思えば良い。
28ご冗談でしょう?名無しさん:01/09/07 09:09 ID:???
>>26
>そこで膨張になっているかどうかは知らない。圧力減少に
>なっているのかも知らない。逆に教えてくれよ。どうなっ
>ているんだい?

前スレをみられないけど、翼前縁の下面側では高圧になるって書いてただろ。
知らないで書いたのかい?
「膨張するから圧力が下がる」って単純な思い込みは、空気を連続体として
とらえる場合には捨てたほうが良いぞ。ベルヌーイの定理で考えるべきだぜ。

>連続体/流体で考えているとき に「引力/斥力」を
>持ち出すのは適当でないと思う

そーしたら、少なくとも

「マクロに見ると翼と空気の間には引力も斥力も働かない」

とゆー表現は不適切すぎるので訂正しなさい。
作用している力を「引力/斥力」を呼びたくないだけでしょ。
29ご冗談でしょう?名無しさん:01/09/07 10:12 ID:mq5X1Udk
>28
26だけど、

言っている事がやっとわかったよ。俺はごくごく前縁に
近い部分を聞いているのかと思ってた。そこは翼の曲率
が大きく変化している上に空気の流れが複雑だから判ら
ないって言ったんだよ。っで、

>翼前縁下面側で流線が開いているのは膨張かい?
>ここで圧力が減少すると言っているのかい。

という元々の質問に対しては、圧縮でしょ。流線(とい
ってもリンク図は模式図だけど)が開いているからって
膨張するわけでなく、空気の「慣性」で圧縮されて圧力
が高くなる(=正圧)だよ。

>ベルヌーイの定理で考えるべきだぜ。

は何を言いたいか意味不明だし、まさか「元スレ
27氏」からこんな言葉が出るとは思わなかった。
30ご冗談でしょう?名無しさん:01/09/07 10:13 ID:mq5X1Udk
--- 上の続き ------------
>28
>「マクロに見ると翼と空気の間には引力も斥力も働かない」

は別に間違っている表現とは思わないが...って言うか、
この言葉の使い方については相互作用する質点モデル
(かな?)の君の方が(>>16 氏も含めて)間違っているぞ。

圧力ってのは、相互作用する粒子モデルの場合、粒子と
壁面の間の弾性衝突による作用/反作用で生じる力だろ。
粒子と壁面の間の引力/斥力ではないぞ。

引力/斥力ってのは、相互作用する相手粒子の距離に応じ
てその力の大きさは変わって良いんだけど、時間に対して
は普遍的に働くし、その粒子の存在位置(例えば箱内部の
上の方でも真中でも)に対しても普遍に働く力の事じゃな
いのか...普通の定義では?

君のモデルの詳細説明が無いんで良く判らないが、粒子
同士間には引力/斥力が働いてるが、粒子と壁面には
引力も斥力も働いていないんじゃないのか?粒子は温度に
応じて飛びまわっているから、粒子間に引力/斥力があって
もバラバラのまま動きつづけるけど、粒子と壁面に引力/
斥力が働いていれば、粒子は飛びまわっても、壁面近くの
粒子密度が高くなる/低くなるぞ。
31ご冗談でしょう?名無しさん:01/09/07 10:35 ID:???
>>29
あのねぇ、キミは流体力学を知らないんでしょ。

>という元々の質問に対しては、圧縮でしょ。流線(とい
>ってもリンク図は模式図だけど)が開いているからって
>膨張するわけでなく、空気の「慣性」で圧縮されて圧力
>が高くなる(=正圧)だよ。

誰かこの子に愛の手を差し伸べてやってくれぃ。

元スレ27はベルヌーイの定理を否定したことはないけどな。
じゃあねぇ、キャブレターを考えてみようか。管の途中で内径が
小さくなっているヤツね。この細い部分では流速が速くなって
圧力(静圧)は減少してるけど、空気は膨張してるのかい?
32ご冗談でしょう?名無しさん:01/09/07 10:55 ID:???
>>30
>圧力ってのは、相互作用する粒子モデルの場合、粒子と
>壁面の間の弾性衝突による作用/反作用で生じる力だろ。
>粒子と壁面の間の引力/斥力ではないぞ。

これは間違い。もうキミは救いようがないな。
粒子と壁に斥力が作用するから「衝突」するんだよ。
どーゆー斥力かは第1スレに書いてあるよ。

>時間に対しては普遍的に働くし、その粒子の存在位置(例えば箱内部の
>上の方でも真中でも)に対しても普遍に働く力の事じゃないのか

意味不明な定義だが、粒子‐粒子、粒子‐壁に作用する力は普遍的だよ。
クーロン力とパウリの排他律で説明したよ。

>粒子と壁面に引力/斥力が働いていれば、粒子は飛びまわっても、
>壁面近くの粒子密度が高くなる/低くなるぞ。

またそんなウソ書いちゃって。なんでそーなるのか「証明」できるの?

キミは自らの間違いを認めるまで放置だな。
33ご冗談でしょう?名無しさん:01/09/07 11:01 ID:???
>>32 訂正
>クーロン力とパウリの排他律で説明したよ。
ここは
 クーロン力とパウリの排他律で説明してたよ。
の間違い。
オレが説明したみたいな書き方になってしまったぜ (藁
34ご冗談でしょう?名無しさん:01/09/08 01:19 ID:gSM8vPZU
>>27
空気分子に働いているのは引力と斥力。
そしてバランスが崩れていてかつ引力が変化してないってことは
変わってるのは斥力って事でいいのかい。
35 :01/09/08 03:19 ID:n2hYHjdE
空気分子間に引力ねえ。
クーロン力がその正体だって言うの?
36ご冗談でしょう?名無しさん:01/09/08 08:51 ID:???
>>34
引力だけでも力のバランスは保てるけどな。
分子間に作用する引力は距離の関数だぜ。不変じゃない。

>>35
ぷりさいすりー
37 :01/09/08 11:11 ID:krEfUYk6
栄光の27は元スレの27と奇しくも一致か?
でも、このスレの27は再度、レスする時は27というコテハンに
するんだな。
38ご冗談でしょう?名無しさん:01/09/08 13:19 ID:HUkn5MZo
>>36
翼上面で流れが曲がるのは何となく分かった気がするんだけど、どうして翼上面に
低圧部が発生するの?あと、どうして翼上面の空気の速度があがるの?
39ご冗談でしょう?名無しさん:01/09/08 13:50 ID:???
>>38
訊いてばかりいないで少しは考えてみようぜ。
分子運動論的には圧力ってなんだい?
揚力があるとき、翼近傍の空気分子はどのような力を受けるかな?

翼が無ければ空気の流速はゼロだぜ。
風洞実験から離れて、静止している大気中を翼が動くときの空気の
流れを想像してごらん。
40ご冗談でしょう?名無しさん:01/09/08 13:54 ID:18wx6hCE
理想気体では分子間の引力はありませんが、それでも空気の流れが互いにはがれて真空を作るようなことはまずありません。
なぜなら 圧力Pの気体内に体積Vの真空を作るのには、PVの仕事が必要なのです。

また圧力が無い場合でも、水の様に分子間力があれば表面張力により
真空との界面の面積を小さくしようとしますので、
流体は勝手に真空を作ったりしません。
実際の翼の場合こちらの効果は小さいのでは?

翼の上面と下面は、共に翼を押しつぶそうとしていますが、
上面の押し方が下面に比べて小さい場合を、
「翼が吸い上げられる」と言っているのでしょ。
掃除機で引きが悪いとの表現と同じです。
41ご冗談でしょう?名無しさん:01/09/08 14:04 ID:HUkn5MZo
>>39
いや、考えてもわかんないから訊いているんです。
圧力は分子運動論的には、分子が壁面で跳ね返った反作用で生じる力だと思うんで
すが、低圧部があるという事は分子の数が減っているという事ではないですか?
どうして減るんですか?
あと揚力があるときの分子が受ける力という意味がよく分かりません。空気分子
の動きが何か変わった時の反作用として揚力が生じるんではなかったですか?

私のイメージも風洞実験ではないです。分子がランダムに斥力と引力を及ぼしな
がら飛びまわっている中を、翼が横切るというイメージです。
42ご冗談でしょう?名無しさん:01/09/08 15:07 ID:SEL6Hbpk
分子の存在ってのは、流体の密度が、離散して存在している領域
に集中していて、密度が0でない領域の近くの質点が平行運動
しているってことじゃないの?
マクロな流体は、この特殊な密度場の流れの時間的空間的平均
として捉えられる。こう考えれば、分子運動論と、流体は無理に
区別する必要が無くなって、物事がクリアに見えて来る。
圧力は「分子」内部に働く力ということになるから、「分子」
同士に働く力は、遠隔力ということになるだろうけどね。
43ご冗談でしょう?名無しさん:01/09/08 16:05 ID:HbKCMZh2
>>36
結局どうやってバランスを崩してるんだい?
そこが知りたい。
44ご冗談でしょう?名無しさん:01/09/08 16:59 ID:18wx6hCE
揚力の原理は
>2 で紹介されているサイト(英語のサイトだけど、シミュレーションなども載っている)
http://www.allstar.fiu.edu/aero/airflylvl3.htm
で様々な疑問に対してに詳しい解説が載っていましたよ。
大体これで納得できるのでは?
45 :01/09/08 17:08 ID:w9d3GBlY
あーう、又しても議論をリセットする人が〜。
46ご冗談でしょう?名無しさん:01/09/08 17:36 ID:04bxXeUk
>>44
はい。大変良いサイトでした。要するに、ベルヌーイの定理だけ
では不正確だけど、揚力は上下の圧力差であり、飛行機通過後の
気流の下向き成分は、揚力の一部が為した仕事で、大部分の揚力
の反動は地面の圧力上昇で説明出来るというのですね。
(地面での大気圧上昇は相当広い範囲に広範に起こる)
47Ledの教え子:01/09/08 18:10 ID:s/pwevnQ
>>46
元から判ってるの!!!!

ってか、何で無駄な議論続けているのですか?
48ご冗談でしょう?名無しさん:01/09/08 18:58 ID:???
>>47
物理厨房発見。
元から判ってるって、貴方自然の裁判官?
無駄な議論って、貴方がそんな価値を何の権限で決めるの?
49ご冗談でしょう?名無しさん:01/09/08 21:14 ID:HSU7kQ0Y
>>40
エントロピー増大則と同じ文脈で、気体の一部が勝手に膨張して
真空が発生することは無いと言っているようだけど、確かにそれは
そうかも知れない。だけど、ここで言われている真空は、運動に
よって発生する(かも知れない)仮想真空のことだから同義では
無いと思うよ。

真空発生肯定派=空洞派?の論理は次の通りだと思うけど。
違ったらゴメソ

流体がある瞬間、内部に発生する力が無くなったとすると、それを
構成する要素は外力のみを受けて運動する。その軌跡が障害物に常に
接して運動することは保証出来ない。だから、真空が発生することも
否定は出来ない。実際には、構成要素は、相互作用をしており、特に
密度を最小化するように力が働くから、真空そのものにはならない
だろうが、低圧になることも起こり得る。またこの考えからは高圧
が発生する理由も同じ理由で説明出来る。
50ご冗談でしょう?名無しさん:01/09/08 22:33 ID:???
>>45,>>47
流れから考えれば、>>44および>>46(おそらく>>48も)は
>>41>>43の答えに窮した>>39(=>>36=「元スレ27」)
だぞ。多分、必死に答えを考えてた(=ごまかそうとしてた)
んだが、うまいごまかし方が見つからなかった故の足掻きだ。
変に騒がなくても良いから、>>44>>46の答えを待とう。
51ご冗談でしょう?名無しさん:01/09/08 22:46 ID:???
>>48
っつーか、>>46はネタ。件のページはそんな結論になっていない。
それを間に受けてレスしている厨房は逝ってよし。

>>44
このスレが続くのは、down wash が空気の viscosity で生じる
理由がわからないからですよ。
5250:01/09/08 23:53 ID:???
間違えてもうた。

× >>44>>46の答えを待とう。
○ >>41>>43の答えを待とう。
53ご冗談でしょう?名無しさん:01/09/09 00:40 ID:PHnwWSm.
とりあえず>>41に関しては
>低圧部があるという事は分子の数が減っているという事ではないですか?
これは違うと思うけど。ベルヌーイの定理を知らないのかな。
>>43の答えについてはぜひ聞きたい。
54ご冗談でしょう?名無しさん:01/09/09 00:45 ID:???
↑「元スレ」27 (=>>39=>>36) 参上
5553:01/09/09 01:27 ID:PHnwWSm.
悪いが俺は元スレ27じゃないぞ。

それはいいとしてもう一つ疑問があるんだけど>>4
>同時到着説、曲げ派はダメ。空洞説もダメ。循環保存説もダメ。
なんて言ってるけど最初の三つは良いとしてなぜ循環保存が駄目なのかも
説明してもらいたい。これを否定するのは結構すごいことだと思うが。
56ご冗談でしょう?名無しさん:01/09/09 01:43 ID:TLBg3U6Q
まあ、あまり元スレ27を質問攻めにして負担かけても
悪いから、取り敢えず>>41>>43>>55 に対する
彼(彼女じゃないわな?)からの回答待ちというとこ
ろかな。
57ご冗談でしょう?名無しさん:01/09/09 03:20 ID:???
>>56
そういう言い方すると出て来られなくなるだろ!(藁
4=36=39からの>>41,>>43,>>55に対する回答待ちだろ。
58真打登場だ:01/09/09 03:47 ID:???
けど、煽るコドモが多いのでナエ。質問した人だけ考えてちょ。

>>41
気流が上昇→下降となるあたりで、分子の面に対する速度成分はどーなってる?

>>43
翼は空気にどーゆー仕事をしてる?

>>55
「循環の保存」を否定しているわけではないぞ。
循環保存「説」は翼周りに循環が発生する理由をちゃんと説明できているのかい?
59ご冗談でしょう?名無しさん:01/09/09 10:03 ID:SqR/4QFs
>>58
面(翼表面の接平面という意味だと思いますが)に対する速度成分(の変化で
すよね?)というのが、分子運動論的に考えて分かりません。分子運動論では、
いろいろな速さを持つ非常に多くの分子がいろんな方向に飛びまわっています
よね。面に対する速度成分(全ての分子の平均でしょうか?)は平均すれば
0だと思います。翼の上面が通る際に、分子論的に考えて、翼表面への速度の
平均がトータルで0でなくなると言う事ですか?分子が一斉に翼表面から離れ
ていくか、寄って来るということですか?翼が仕事をしているはずなのですが、
分子論で考えているとどういう事なのかピンときません。

ちょっとわかりにくいので、禅問答みたいな意地悪をしないで、元々の質問で
ある「どうして翼上面に低圧部が発生するか」と「どうして翼上面の空気の
速度があがるか」を分子論的に教えて頂けないですか?
60ご冗談でしょう?名無しさん:01/09/09 10:26 ID:???
>>59
風が吹いていると分子の平均速度は風速(向きも含めて)。ゼロじゃない。
翼上面の形状と気流をよーく考えてみれば答えはわかるでしょ。

禅問答ねぇ。マヌケなコドモは「煙に巻く」って書いてたな (藁
61ご冗談でしょう?名無しさん:01/09/09 11:11 ID:LuCIr5Xg
>>60
ID変わっていると思いますが、59です。
今は風(?分子論的な言葉ではないですが)など考えていないと思います。
>>39であなたが言われてるように、静止大気中(というより、分子がランダム
に運動しており、平均速度0の状態の分子の海)で翼が横切るイメージです。
>>39で言ったのは、空気に対する静止系で考えろって事ですよね?

とにかく、翼がする仕事の話しもありますから、>>58であなたが>>43さんに求
めている答えともからむのかもしれません。>>43さんの答えを待ってからでも
良いのですが、その答えを待たずとも(最初に尋ねているのは>>43さんなのです
から)「どうして翼上面に低圧部が発生するか」と「どうして翼上面の空気の
速度があがるか」を分子論的に教えて頂けないですか?
62ご冗談でしょう?名無しさん:01/09/09 11:39 ID:???
>>61
風って気流のことさ。
翼が静止した大気中を横切るとどんな気流が生じるのかな。

これからPCのパーツの買出しに行くから、オフで会うかい (藁
63ご冗談でしょう?名無しさん:01/09/09 13:03 ID:???
結局、元スレ27は他人からの質問にまともに答えてない
し、間違いを指摘されれば適当にごまかしてるね。更には
お得意の分子論で説明できないから流体(気流)に話しを
すりかえようとしてるし...、お話しにならないね。
64ご冗談でしょう?名無しさん:01/09/09 15:06 ID:4h9Sjvlo
翼がする仕事って流体力学的には粘性に逆らってする仕事位じゃないか?
粘性を考慮しなけりゃ二次元翼理論だと仕事は0だったはずだが。
分子論的にはどうなんだろ?
65ご冗談でしょう?名無しさん:01/09/09 19:57 ID:???
>>64
翼の周りで「気流」を生じさせるのが仕事でしょ。
気流って平均速度がゼロじゃない分子の集団運動。

平均速度がゼロの空気分子中を翼が横切るとき、気体分子は翼上面の
前縁側の「斜面」でどのような力を受ける? もうわかるでしょ。
で、その後の分子の平均的集団運動はどのようになる?
66ご冗談でしょう?名無しさん:01/09/09 23:16 ID:p72Bhnqk
>>64
翼がする仕事についてだけど、翼は迎角を持って前に進んでる
から、単純に言えば翼上面では空気を下向きに引っ張る仕事を
し、翼下面では空気を下に押出す仕事をする。(これを引力/
斥力とは呼ばないけれども...)

鉛直方向に伸びている円筒シリンダーを考えて、シリンダー内
に円形の可動隔壁が2つ平行に上下に存在するとする。この隔
壁間の空気が翼上面あるいは下面の空気と考える。

翼上面を考えると、翼が通過する際にする仕事は下側の隔壁を
急激に下向きに引っ張る時にする仕事と同じ。慣性のために隔
壁内の空気は一瞬膨張して低圧になるけど、上側の隔壁がすぐ
に下がってきて圧力は元に戻る。翼下面はこの逆で、上側の隔
壁を急激に下向きに押すときにする仕事と同じ。慣性のため
一瞬空気は圧縮されて高圧になるけど、下側の隔壁がすぐに下
がって圧力は元に戻る。

このシリンダーモデルで考えると、可動隔壁が3枚あって、真
中の隔壁が翼と考えても良いかもしれない。翼のする仕事とは
真中の隔壁を急激に下向きに動かす仕事のこと。この時、慣性
のために一瞬の間、上方向の空間は膨張低圧、下空間は圧縮高
圧になるけど、この差圧が揚力。
67ご冗談でしょう?名無しさん:01/09/09 23:23 ID:1ZzFuZ6s
>平均速度がゼロの空気分子中を翼が横切るとき、気体分子は翼上面の
>前縁側の「斜面」でどのような力を受ける? もうわかるでしょ。
これまでの話だと引力と斥力?

>で、その後の分子の平均的集団運動はどのようになる?
現象としては翼通過後に下向き成分の速度を増すわけだが。
皆さんはその理由を知りたがってるんじゃないかい?

ところで翼の前側では流れが上向きになってるけどこれも分子論的には
どういう事なのかな?
68ご冗談でしょう?名無しさん:01/09/09 23:30 ID:p72Bhnqk
>>66の補足だけど。もちろんこれはマクロな見方ね。
分子論で説明できるのかもしれないけど、翼との作用?等
難しい設定をしているとできないような気がする。

あと、勘違いしてる人もいるけど、流れという観点で見た
場合、あくまで翼上面では下向き流れを起こしたいんだよ
ね。剥離等の問題があるから翼の形状を工夫して、少し
ふくらませてある。翼の前の方で流れをちょっとだけ上向
きにするのは、その後に剥離させずに流れを下向きに曲げ
るための準備。

以上はもちろん流体的(マクロ的)な見方ね。
69ご冗談でしょう?名無しさん:01/09/10 00:09 ID:XpIh8uKE
>>68
>翼の前の方で流れをちょっとだけ上向
>きにするのは、その後に剥離させずに流れを下向きに曲げ
>るための準備。

これはどういう事だろ。
できればもう少し詳しく説明してもらえないかな?

あと、翼が通過するときにシリンダー的に低圧部が発生っていう考えは
否定されてるみたいだよ。シリンダーと違って開放されてるし。
前スレでも出てたけどマッハ数が小さいうちは空気を非圧縮とみなしても
誤差は1%程度とか。
70ご冗談でしょう?名無しさん:01/09/10 00:41 ID:2K0PKEZE
>>69
翼前方での上向き流れに関しては、うまく説明できない
けど、構造強度を出すためには翼にはある程度の厚みが
必要で且つ前縁を滑らかな曲線にして、それを上面につ
なげなければならないと思うんだよ。すると、下向き流
れをつくる前に必然的に上向きの流れができる事になる
んじゃないかな。

シリンダーについては、イメージを分かりやすくするた
めに閉鎖系で放したけど、慣性があるから開放系であっ
ても局所的には同じような事が起きると思うけど、どう?

ただ、自分自身ちょっと分からないのは、流体が水の場
合でも同じように考えられるかなって事。膨張・圧縮が
全くない仮想流体を考えても揚力は発生するのかな?
この仮想流体の場合、翼上面の低圧部の発生が翼上面の
流速の増加によるものとして、その流速の増加の駆動力
がイメージできないんだ。
71ご冗談でしょう?名無しさん:01/09/10 00:53 ID:2K0PKEZE
70だけど話しの流れと外れる話しをして申し訳なかった。

今は
1. >>61,>>43の質問(更に>>67が催促)に対する
   「元スレ27」の分子論説明待ち と
2. >>55の質問に対する>>58の反駁に対する「55」の
   反応待ち

だったね。話しの筋が見えにくくなるから(それを望んで
いる人も居るみたいけど)上の2つの結果を待ちましょう。
72ご冗談でしょう?名無しさん:01/09/10 00:57 ID:ars5NOQI
揚力の説明はたいていコップを水道の水に近づけるところから始まるのですが。
73ご冗談でしょう?名無しさん:01/09/10 01:05 ID:2K0PKEZE

スプーンでもいいんだけど、そうなんだよねぇ。
コアンダ効果により曲面で曲がる水は膨張してるのかな?
膨張していないとすると、圧力が下がって且つ流速が速く
なるのは分かるんだけど、その流速を速くする駆動力は
何なんだろう? >>72さん分かる?
74ご冗談でしょう?名無しさん:01/09/10 01:22 ID:???
隊長、振り出しに戻りました!
75ご冗談でしょう?名無しさん:01/09/10 02:28 ID:???
いや、込み入った時には振出しに戻る事は重要だよ。
76ご冗談でしょう?名無しさん:01/09/10 07:57 ID:???
>>67
翼上面では、水平に飛んできた分子が前縁で「衝突」した後、斜面に沿って上昇する
とゆーイメージでオッケーじゃないのかな。
で、気流が上昇→下降と変化するところで作用するのは引力でしょ。
分子間に働く力は接近すれば斥力、離れていれば引力。
翼が静止していれば平衡状態にあるんだな。
77ご冗談でしょう?名無しさん:01/09/10 10:18 ID:???
>>66,>>68-75
これは、流体力学では揚力を説明できないって主張が正しいことを示してるの?
78ご冗談でしょう?名無しさん:01/09/10 10:40 ID:???
>>77
そんな事ないと思うけど。
79ご冗談でしょう?名無しさん:01/09/10 10:46 ID:???
説明が破綻したのでリセットしてるね。テレビゲームみたい。
80ご冗談でしょう?名無しさん:01/09/10 13:18 ID:2Mah6W86
>>77
水の場合に膨張説でも説明できるとすれば、膨張説は否定されてい
ないと思うけど、水の場合に説明できなければ破綻してるね。そう
すると流体で説明してきた今まで挙がった説は今のところ全滅かな。
ただし、分子論的考えが充分に説明されていない今、結局、揚力を
説明し得る説は、このスレでは出てないってことじゃないか?

新説、あるいは既存の説の不合理をうまく説明する説の登場が待た
れるんでは?(それより、水を反例にした膨張説の否定と分子論で
の合理的かつ充分な説明の方が先だと思うが)
81底値氏:01/09/10 13:19 ID:F.dlYONE
82ご冗談でしょう?名無しさん:01/09/10 14:10 ID:jbwV8nyE
>>81
んっ?何て読むの?「そこ氏ね」じゃ無いし...

>>77,>>80
まとめるとこういう事じゃない?
・標準派(ベル派)
エネルギーが保存しない環境下で非粘性・非圧縮性でもない
空気に対してベルヌーイの定理を適用している(ベル派)し、
何よりも翼上面の低圧部と速い流れを説明できないので破綻。
(同時到着説は実験事実より否定)

・流体的反作用派
翼により空気の流れを曲げて、その反作用として揚力が生じ
ているという立場であり、ここまでは合理的である。ただし、
圧力分布が揚力を与える事を否定していないものの、実際の
所、翼周りの流れや圧力分布を考えていない。すなわち、や
はり翼上面の低圧部と速い流れを説明できていない。膨張説
も唱えられているが、水を考えると説明できなさそう?

・27派(分子論派)
充分な説明がされておらず、海のものとも山のものともつか
ない状態。

・その他
省略
83ご冗談でしょう?名無しさん:01/09/10 18:15 ID:Q7aP2bHU
66の意見は、翼の上の空気を跳ね飛ばして、下から膨張のために
吹き上がろうとする空気の上に乗るというイメージに近いのかな?
跳ね飛ばした空気が、落ちてきて翼に乗っかっちゃったら、揚力は
小さくなるってこと。
84ご冗談でしょう?名無しさん:01/09/10 21:33 ID:T4TnALjo
元スレのURLはこちら。
http://cheese.2ch.net/sci/kako/988/988866699.html
継続スレは大き過ぎて読めないから誰か、スレに直接関係ないレス削除
してもらって、暫く待てばHTML化されて倉庫入りするでしょう。
取りあえず2番目は読めない。
85ご冗談でしょう?名無しさん:01/09/10 23:20 ID:grrnsPWM
>>84
リンクありがとう。
でも、込み合う時間にDLしたら現在の2chでは顰蹙を買うか。
結構大きい。
ざっと見た限り、やっぱり27氏の粘着説と霊気説が最も光ってる。
86Ledの教え子:01/09/11 00:17 ID:???
ってか、答えの出せない者同士が相手の意見を批判しあってるだけだと思うぞ!

# もう上げないほう良いんじゃないの?
87ご冗談でしょう?名無しさん:01/09/11 01:15 ID:???
>>86
いや、問題となってるのは元スレ27氏が答えが出せると言い切ってしまった
事だと思うぞ。
そして現段階では彼の説明の不明瞭な点について説明待ちという訳。
88ご冗談でしょう?名無しさん:01/09/11 08:30 ID:???
>>87
何が不明瞭なのか質問してみたら教えてくれるかもね。
「充分な説明が無い」って、知識不足で理解できない状態の責任転嫁だと思う。
一番最近の説明>>76は読み手に考えることを要求しているのは確かだけど、
これは>>58あたりを読むとわざとじゃないかな。
89ご冗談でしょう?名無しさん:01/09/11 10:04 ID:AdE/gLyY
自信があるどころか、自分でも疑わしいと思っているから
最後を疑問文の形にして最終的には信じた者に責任を転嫁させようという
手段が皆に見抜かれているところが、実にイタい。(藁
90ご冗談でしょう?名無しさん:01/09/11 11:04 ID:???
本当は分子論的な説明を理解したいのだが、変なプライドがあって
質問できないから(多分あの大学)、煽りで教えてもらおうという
手段が見抜かれて答えてもらえないところが、実にイタい。(藁
アゲてるところでバレバレ。
91ご冗談でしょう?名無しさん:01/09/11 17:01 ID:???
>>88
87でも89でもないが、相手するのもいい加減疲れて来たん
だけど、>>61,>>43の質問に対する 「元スレ27」の分子論
説明待ちなんだよ。質問してるのに答えてないだろ。

すなわち、「元スレ27」が答えてない質問は、
1. 「どうして翼上面に低圧部が発生するんですか?」
2. 「どうして翼上面の空気の速度が上がるんですか?」
3. 「空気分子のバランス(平衡状態)を、翼により
  どうやって(+どういう風に?)崩しているんですか?」
92ご冗談でしょう?名無しさん:01/09/11 20:21 ID:???
ウソと煽りしか書けないコドモ達が、スレを無駄に消費してるなぁ。
これも嵐か?

>>61の質問は>>62で応答待ち。だけど>>76に答えが出てるね。
で、>>76>>43に対する回答にもなってるね。
9388:01/09/11 20:41 ID:1jgQTuOY
>>89
れれ?
元スレ27のこと言ったわけじゃないのに(藁
9489:01/09/11 20:42 ID:1jgQTuOY
ゴメソ
>>90
へのレスでした。
95ご冗談でしょう?名無しさん:01/09/11 22:23 ID:tKp/nkfU
>>92
61ですが、私の質問は>>91で記していただいたように、

1. 「どうして翼上面に低圧部が発生するんですか?」
2. 「どうして翼上面の空気の速度が上がるんですか?」

です。これに関しては、一切回答いただいてないと思うのですが。他
の方もおっしゃってますが、あなたの回答待ちの状態です。あと、
正直言うと、>>76も何の説明にもなってないですよね。空気の塊が
翼に沿って流れる事をあたかも分子論で説明できたかのように言って
いるだけですから。後半言いすぎたかもしれませんが、とにかく、
ここで再々掲した質問の回答をいただけないですか。
96ご冗談でしょう?名無しさん:01/09/11 22:30 ID:gGkX/3gE

飛行機があれだけ高く飛べるのは、

凄まじいばかりの風の抵抗があるからなのだ。

by 逆境ナイン
97ご冗談でしょう?名無しさん:01/09/11 23:11 ID:???
>>95
キミは第1スレにいた kitty にソックリだなぁ。
まぁ、ホントに知りたいなら、これから質問することに答えてね。
それがイヤなら他の人に聞けばいいよ。

まず、>>61>>95の後半に書いてあることはイケないな。
気流を持ち出すと「分子論」じゃないと思うのがダメ。
注目する空気分子の平均速度がゼロじゃない状態が「気流」なんだが、
これは理解できるかい?
98ご冗談でしょう?名無しさん:01/09/11 23:23 ID:zmzo9M4o
>>97
確認しときたいんだけど
>>23
>引力が強くなってるわけじゃないよ。
>翼の上面に沿った流れは、上昇気流→下降気流と変化するでしょ。
>つまり鉛直方向の運動量が大きく変化したわけで、大きくなった
>のは翼が空気に対して作用した力積。力そのものではない。

って言ってたのと
>>76
>で、気流が上昇→下降と変化するところで作用するのは引力でしょ。
>分子間に働く力は接近すれば斥力、離れていれば引力。
>翼が静止していれば平衡状態にあるんだな。

って言ってるのはどっちがホント?それとも別人?
あと、あなたの意見だと揚力とはつまり空気が分子間力で翼を引っ張ることによる
という理解でいいのだろうか?
>>91の後でもいいからよろしく。
99ご冗談でしょう?名無しさん:01/09/12 01:32 ID:XUb.qfzI
今、アメリカで起こっていることを見る限り、飛行機はやはり
基本的に落ちるものであり、飛ぶというのは幻想のようだ。
100ご冗談でしょう?名無しさん:01/09/12 01:44 ID:???
飛行機って、特攻に使うんでしょ?
101ご冗談でしょう?名無しさん:01/09/12 01:51 ID:mRjsUquE
ビルの解体工事に使うんだよ
102ご冗談でしょう?名無しさん:01/09/12 01:57 ID:NE72VC5c
>>101えらい高い費用がかかりそうだな。
いったい今回のでどのくらい損失があるのか計算不可能だな。
103ご冗談でしょう?名無しさん:01/09/12 06:52 ID:9GvtsUUQ
こんなところに書き込みしている場合ではないような気もしますが、

>>97
>他の人に聞けばいいよ
無茶を言わないで下さい。今のところあなたしか分かっていないみたいですから。

非常に多くの数の分子が、いろんな速さでいろんな方向にランダムに飛びまわっ
ていますが、全体としてある方向の速度ベクトルが大きくなればそれが気流と
なるのはもちろん理解できます。

あなたが>>39でそうしろといったように、静止系で考えて、分子がランダムに
飛びまわっている平均速度0の分子の海に翼が通りすぎ、その結果として、平均
として下方向の速度を持つ分子の流れができるとして説明していただいても良い
です。あるいは、風洞実験のように、翼が静止しており、ランダムに動き回って
いるものの平均として水平方向に速度Vを持つ分子の海がやってきて、翼によっ
てその運動が変化させられるとしても良いです。とにかく禅問答みたいな事はや
めて下さい。質問は下記の2つです。

1. 「どうして翼上面に低圧部が発生するんですか?」
2. 「どうして翼上面の空気の速度が上がるんですか?」
104ご冗談でしょう?名無しさん:01/09/12 08:31 ID:???
>>103
次に翼上面の前側の斜面における空気分子の運動を考えよう。
静止している球に斜面が接近して衝突すると、球はどっちに動く?
まず、斥力の相互作用だけの場合ね。

あとこれは「禅問答」じゃないよ。物理の問答だよ。イヤならやめるよ。
105ご冗談でしょう?名無しさん:01/09/12 13:31 ID:qYY6FCU6
>>103
球は上方の運動量を得ます。

斥力というより弾性衝突のイメージがありますが、そのように理解
して良いですか?あと「球」と言っている事はモデル化して考えて
いて実際の分子では考えていないということですね。質問を整理す
ると、
  ・ 互いに相互作用する「球」モデルで考えていますね?
  ・ 「球」同士の相互作用はレナードポテンシャル型ですね?
  ・ 「球」と「壁面」の相互作用はいかなる形ですか?
      この点、今までに明言がないと思います。また
      通常の分子論では弾性衝突と考えるのですが、
      そうではないという事ですね。このようなモデ
      ルは初めて見ました。

物理問答にしても、あまりこのような形は好みません。私も暇で
はありませんし、何故あなたが質問の答えをすぐに答えないか
(というより、分子論を唱える以上、説明して当然の事を説明
しないから訊いているだけですが)理解に苦しみます。やめても
私は構いません。あなたが逃げていると評価される(既にそう
評価されていますが)だけですから。
106ご冗談でしょう?名無しさん:01/09/12 16:09 ID:???
>>105
そう、上方の運動量を得るよ。
平均として静止している空気分子中を翼が進む場合には、空気分子は
平均として上方の運動量を持つ、すなわち上昇気流ができる
ってことはわかるかい?
「気流で説明したら分子論じゃない」って、もう思わないよね?

次にこの気流が翼に沿って流れることを理解しようと思うのだが、
どーする、やめるかい?

それから、相互作用に関するイチャモンは、完全にキミの無知。
ソフトコア・ポテンシャルって調べてみな。
今の例ではポテンシャルの形は重要ではなく、相互作用が斥力で
あればそれでオッケーだ。
(「レナードポテンシャル型」ってのはイタすぎ。もっとお勉強してね)

問答形式はお望みの充分な説明をするためだぜ。
キミの無知ゆえに理解できない部分がすぐにわかる。

あと、オレに「充分な説明」は求めるけど自分は暇ではないって、
自己チュー過ぎだぜ。忙しい割にはレスは長めだけどな。(藁
こんなコドモの知識欲を満足させる義務はオレにあるのか?
107ご冗談でしょう?名無しさん:01/09/12 16:22 ID:ekc4s3bs
>>106
>相互作用が斥力であればそれでオッケーだ。

球同士では非常に近距離では斥力、遠距離では引力は分かるんですが、
球と壁面にも「同じ形」のポテンシャル(微分すれば力)が働くと
想定してるんですか?それは物理的に事実なのですかそれともモデ
ル化なのですか?

繰り返しますが、別にこの問答形式に沿ってお答えいただなくても
結構です。(というより私はこんな形式を望みません。下記の2つ
の質問にストレートに答えていただければそれで結構です。私が理
解できなくても他の人が理解できれば良いのでは?)説明を止めて
いただいても結構です。自分でどうしても説明したくないらしいの
で、焚き付けて説明してもらおうとしてるだけですから。

1. 「どうして翼上面に低圧部が発生するんですか?」
2. 「どうして翼上面の空気の速度が上がるんですか?」
108ご冗談でしょう?名無しさん:01/09/12 17:00 ID:???
>>106
・球と壁面の相互作用のモデリングについては確かに聞いたことがないと思われ。
>>105はレナードジョーンズ型ポテンシャルの事と思うが、これもソフトコアポテンシャルだと思われ。

>>105
・ほとんどが君の味方だから、困ったときは助けてもらえると思われ & vice versa
109ご冗談でしょう?名無しさん:01/09/12 17:11 ID:???
>108
質点と界面/表面の相互作用を扱うモデルもある事はあるよ。
ただ、難しいので通常はあまり考えないし、使わないけどね。
漏れも、元スレ27が質点と壁面(翼面)にどんなポテンシャ
ル、あるいは拘束条件を想定しているのか興味あるな。
(前スレまでの話だとどうもおかしいし)
110ご冗談でしょう?名無しさん:01/09/13 03:18 ID:???
>>107
なんだぁ、やめちゃうの? オレは説明したいんだけどなぁ。
物理はたいしたことないのにイチャモンだけはリッパなキティを
ボコボコに叩きながらとことん理解させてやるつもりだぜ。(藁
で、次の質問だ。
「粘着の条件」は分子レベルでみるとどのような状態だと思う?
つまり、翼の表面近傍では空気分子は翼からどのような力を受けている?
111ご冗談でしょう?名無しさん:01/09/13 03:27 ID:vJzFzbFM
>>110
107ですが、別にこちらにそんなこと質問しなくても説明したければ勝手に
どうぞ。私はあまり興味ありませんが変なところあればいろんな方から突っ
込んでもらえるでしょう。ただ、何度も繰り返しますが、下記の現象の分子論
的説明をしてもらえれば良いだけです。何をごちゃごちゃ言ってるんですか?

1. 「どうして翼上面に低圧部が発生するんですか?」
2. 「どうして翼上面の空気の速度が上がるんですか?」
112ご冗談でしょう?名無しさん:01/09/13 03:34 ID:???
>>108
レナード=ジョーンズ型は近くで斥力遠くで引力、ソフトコアは斥力だけ。
これギョーカイの常識。知ったかぶりで間違いを書くのは理解の妨げになる。
迷惑だぜ。

>>109
粘着の条件に絡めて説明しようと思っているよ。>>107の反応次第だけどな。
113ご冗談でしょう?名無しさん:01/09/13 03:35 ID:vJzFzbFM
>>110
あっ、そうそう補足ですけど、説明するなら空気分子と翼面に働く力を
きちんと仮定してから説明してくださいよ。分子/分子間に働く力と分子
/壁面に働く力を「全く同じもの」としているのか否かをはっきりさせて
ください。
114ご冗談でしょう?名無しさん:01/09/13 03:39 ID:???
>>111
オレには質問しておいて、自分が質問されるのはイヤなのかい?

今はキミが答えてくれないと先に進まない状態だぜ。頑張ってくれ。
115ご冗談でしょう?名無しさん:01/09/13 03:43 ID:???
>>113
違うものだよ。前にも書いたけどな。
さぁ、今は暇そうだから、>>110の質問に答えておくれ。
116ご冗談でしょう?名無しさん:01/09/13 03:47 ID:vJzFzbFM
>>114
111だけどアホらしい。

1. 「どうして翼上面に低圧部が発生するんですか?」
2. 「どうして翼上面の空気の速度が上がるんですか?」

ってのは質問ってよりも、その説を検証するための必須の説明じゃない
のか?勘違いするなよ。説明できないからって難癖つけて説明やめるつ
もりかい?別にいいけど、これで「元スレ27」の化けの皮が(まあほと
んど剥がれていたけど)剥がれたってことか。
117ご冗談でしょう?名無しさん:01/09/13 03:52 ID:???
>>116
だから説明をするための準備をしてるんだろ。
だいたい>>110みたいな簡単な質問を難癖だと思うのが変。
答えられないわけじゃないだろ?
118ご冗談でしょう?名無しさん:01/09/13 03:57 ID:vJzFzbFM
>>117
116だけど、準備なら勝手にやってくれ。
別に俺は分子論説明派じゃないから、自分のことは自分でやってくれ。
俺は別に分子論での説明の試みを否定している訳じゃない。下の2つ
をうまく説明できるかどうかでその説の妥当性が判断できるんだろ。
そろそろ観念して説明してくれよ。

1. 「どうして翼上面に低圧部が発生するんですか?」
2. 「どうして翼上面の空気の速度が上がるんですか?」
119ご冗談でしょう?名無しさん:01/09/13 04:10 ID:???
>>118
あのねぇ、オレには説明する義務なんて無いの。でも皆が知りたがっているから
説明しようと思っているんだけど、>>116みたいなキミの書き方はイヤなんだな。
だから、キミの反応次第で説明を進めるってことにして、責任をとらせてるの。
そろそろ観念して>>110の質問に答えてくれよ。
120ご冗談でしょう?名無しさん:01/09/13 04:19 ID:vJzFzbFM
>>119
118だが、「勘違いするなよ」と繰り返さざるを得ないな。
今頃気づいたんだが、真性キティを相手にしている俺が馬鹿なのか...。
121ご冗談でしょう?名無しさん:01/09/13 04:37 ID:???
>>120
またまた、そんなこと書いちゃって。勘違いしてるのはキミ。
早く答えてくれないと説明するのが遅れるぞ。キミの責任だ。
そんなに>>110の質問は難しくないだろ。
122Ledの教え子:01/09/13 05:37 ID:???
>>120
もう相手するの止めたほうが良いと思うよ。
勿体付けてる奴にマトモな回答が出来るとは思えないしね。

はい、終了♪
123ご冗談でしょう?名無しさん:01/09/13 08:39 ID:???
↑やっぱそうだよね。終了かどうかはさておき、その他はおっしゃる通り。
124ご冗談でしょう?名無しさん:01/09/13 09:38 ID:???
>>123
そんなに居心地のいいレスつけると常駐するかも…
つーか、夜中に言い争うくらいなら質問に答えて
先に進めたほうがよかったとオモワレ。
125Led:01/09/13 10:06 ID:???
>>122
もう物理板に出入りするの止めたほうが良いと思うよ。
勿体付けてる奴にマトモな回答が出来るとは思えないしね。

はい、死ね♪
126 :01/09/13 10:44 ID:???
世界平和は遠いな・・。
127ご冗談でしょう?名無しさん:01/09/13 19:23 ID:54fPSPY6
なんか荒れてんな…。進行しないので横レス。
>>121
>>110に関しては>>9で電子の交換相互作用によって
吸着してるって書いてあるよね。
俺にはよく分からんのだが翼と空気分子が結合してるって事?
128ご冗談でしょう?名無しさん:01/09/13 21:25 ID:???
>>124
いや、答えなくて正解だろう。
って優香、その回答って分子論で説明するなら結構デリケート
な内容になると思うぞ。「元スレ27」に説明させれば宜しい。

「元スレ27」の戦法としては、相手に回答させてあいまいな
点を罵倒して馬鹿には教えないぞと吐き捨てて話しを途中で
終わらせるか、あるいは、相手に回答させて話しを別の方向
に持っていくか だ。いづれにしろ、話しを進めるつもりは
無いらしい。スレ1と2を見ればよくわかる。

今回は翼上部の低圧部と速い流れという元スレ27の答えられ
ない話題から、頑として話しを逸らさせまいとした>>120
対して、「元スレ27」が喚いているだけだろう。
129ご冗談でしょう?名無しさん:01/09/14 02:20 ID:???

ドキュソ発見。
130ご冗談でしょう?名無しさん:01/09/14 10:27 ID:rt5BUiKg

ドキュソ発見。
131ご冗談でしょう?名無しさん:01/09/14 11:02 ID:???
おぃおぃ、一応言っておくけど、>>130は漏れ(128)じゃないぞ。
>>129は「元スレ27」だと思うが...。
132ご冗談でしょう?名無しさん:01/09/14 15:05 ID:???
>>131
124だけど>>128では、分子論の説明はデリケートなので「元スレ27」に
答えられないと決め付けていますよね。その態度に問題があるとオモワレ。
流体力学的な説明が停滞(破綻?)しているので、今は分子論的な説明を
議論するタイミングだと思うけど、もともと分子運動論はもっと普遍的な
わけだから、「元スレ27」が出てこなくても話しは進められるはずですね。
相互作用の話しをもう少し聞きたいので、個人的には>>110からの再開をキボン
ですが無理っぽいので、スレの正常化を願って寝たふりします。
大気圧で平衡状態にある分子運動が翼の運動で低圧側にずれるのは、上面側で
上昇気流になっていて、平均として翼から遠ざかるからだと考えていますが、
相互作用のことを考えるとそんなに単純じゃないですよね。
133ご冗談でしょう?名無しさん:01/09/14 20:22 ID:???
>>132
おっ、「元スレ27」登場か。
説明したくてうずうずしてるみたいだから、きちんと全部
自分ですればどう?分子運動論においては、翼前縁でアップ
ウォッシュがおきるために低圧になるってのは初めて出てき
た概念だね。(流体の説明では既に出てきていたけど。)
その調子で最後まできちんと自分で説明してくれぃ。
(いいヒントくれたね。流体でうまく説明できそうな気がしてきた。)
134ご冗談でしょう?名無しさん:01/09/14 20:31 ID:???
>>132
133(=124)だけど、答えるも何もできないよ。空気分子と翼の相互
作用たってせいぜい2〜3原子距離しか働かないんじゃないか。
デリケートも何も、分子論での説明は漏れにはうまくできない。
君は答えられるのか?結局よく判らないような事を言っている
だろう。だから、張本人「元スレ27」に説明を請うているんだよ。
135ご冗談でしょう?名無しさん:01/09/14 23:38 ID:cE7aZAso
>>132
>大気圧で平衡状態にある分子運動が翼の運動で低圧側にずれるのは、上面側で
>上昇気流になっていて、平均として翼から遠ざかるからだと考えていますが、
なんで上昇気流になるかが問題ではないかな。
例の参考リンク
http://www.allstar.fiu.edu/aero/airflylvl3.htm
のFig2を見るとかなり下の方の流れまで上面側に流れてるし。
これは流体にせよ分子論にせよ一筋縄では行かない問題だと思う。
136ご冗談でしょう?名無しさん:01/09/15 03:15 ID:???
流体か分子論かっつー、不毛な議論が続いているので横レス

分子論っつーのは、流体を完全な連続体とせず、内部に空洞(密度=0)
を含んでも良いという考えから出てきたもの。
特に密度がδ関数の和で表されるような場合が質点系に対応するもの
と考えられる。
つまり、流体力学は、質点系の力学をある程度含んでいる。それもその
筈で、Newton力学の一般化なんだから。

離散的な質点系と、連続体との区別は実はその相違を真に引き出すのは非
常に難しい。離散的なモデルからは、再帰性が、連続体からは消散性とい
う性質が説明しやすいとされているが、数学的な根拠は曖昧で、視覚的な
イメージに訴えやすいからそう思われているとオモワレ。
両方とも証明されてないし、排他的に一方だけが起こるということなんて
ことも勿論証明されていない。

何が言いたいかというと、普通の考え方じゃ、流体的方法と質点(分子
運動論)的方法との相違は実は大差が無く、一方で得られる結果は
原理的には必ず他方でも得られるということ。(数学技術論は無視)
勿論、分子運動論は、時間統計/空間統計をして流れと解釈している
のだが、これは分子の流れを統計的に解釈するという手法であり、
統計流体力学とも解釈できる。(流体力学を一段高く統計的に見ても
やはり流体力学。)

「翼表面と空気分子との特殊相互作用の時間統計・空間統計が気流の
下向き運動量として現れる」という主張は、「非常に特殊な境界での
外力作用を持つ流体の統計的な振る舞い」という*流体力学の*言
葉で書くことが出来る。だから、流体だ、分子運動論だって張り合う
のは無意味。消散性や再帰性の議論以外は、片方がもう片方を説明す
る。祖先は共に同じだから。でも、統計流体力学は、恐らく流体力学より
ずっと難しいだろうから、流体力学的説明の停滞がその基礎方程式の
困難性にあり、というのは、同時に分子運動論についてはもっと当ては
まる。

ただ一つ、質点というものを剛体と解釈し、ジャイロ効果とかそういった
剛体特有の振る舞いを、分子の本質的振る舞いとして組み込むと、話はガ
ラリと変わってくる。この場合、剛体力学と流体力学がミックスされた、ム
ツカシ過ぎる話になってくるが、分子運動論が、流体力学より大きな系に
なる可能性があり、意味のある相違が発生する可能性がある。

元スレ27氏は、この当たりの可能性を「デリケート」な話と言いたい
のではと想像しますが、如何なものでしょうか。
>>136
「デリケート」な話と言っているのは反「元スレ27」
138中学生:01/09/16 00:40 ID:zI6yWlaA
****終了****???
白熱の議論ようやく終了した模様ですね。
みなさま、お疲れ様でした。

元スレ27さんによると、空気と翼のジャイロ効果的粘着効果理論が空気を確実に
下向きにし、翼の上の圧力を下げると同時に加速する粘着の反作用が、揚力
であるとしてますけど、
これに意見が集約されたような感じですが、これが結論でよろしいのでしょうか?

難しい理論から、翼の上の空気が粘着し、決して剥がれて流れず、その上の空気
が表面の空気をムリに引き剥がそうとして、余計に強く翼を上に引っ張ることが
揚力の本質である
と理解して良いのでしょうか?

ベルヌーイの定理による説明は、このあたりの事情に触れていないので、正しい
とは言えない
という理解でよろしいのでしょうか。
最後に元スレ27さんのまとめで閉めたいと思いますが、元スレ27さん、
よろしくお願いいたします。
(1001まで一気にカキコして下さいな。)
139ご冗談でしょう?名無しさん:01/09/16 00:48 ID:qFLPQ98k
飛べなければ飛行機という表示が不当になってしまうから。
140ご冗談でしょう?名無しさん:01/09/16 01:44 ID:D4UaFi.A
ただ27氏を待ってるのもなんだから流体力学的な説明を考えてみない?
俺としては出発渦による説明が一番無難だと思うんだけど。
というか翼の後縁の働きを考えずして揚力は語れないと思うが?
141ご冗談でしょう?名無しさん:01/09/16 04:04 ID:7n8GLDb2
当方、流体力学を学んでいないせいだと思うが、渦と
いう概念が今一つわからない。それは良いとして、
出発渦による説明ってのは最初からストーリ展開的に
説明するとどうなるの?
142ご冗談でしょう?名無しさん:01/09/16 16:21 ID:.XZKAPU2
>>141
出発渦は教科書だとこんな感じに説明されてたと思う。

出発するときにはまず非粘性流れに近いような流れが現れる。
するとこの時、翼の後縁では流れが鋭角を回り込むことになり
剥離してしまう。このときに翼の上面と下面から流出する流れの
速さを比較すると(非粘性流で考えると分かると思うが)
上面<下面となっていて翼から渦層が出ていることになる。
これが出発渦。
渦を出した分だけ翼周りにはそれと逆の循環が出来ていき、
翼の上面と下面から流出する流れの速さが等しくなるだけ循環ができたら
渦の放出も止まる。これでKutta条件が満たされ揚力発生。

流体に詳しい人フォローよろしく。
143ご冗談でしょう?名無しさん:01/09/16 22:14 ID:AsAh9aEo
>>142
どうも、ありがとう。
説明自体は分かるし、間違いは無いんだろうけど、なんか
しっくりこないんだよね。上面と下面の流れの速さの違い
や翼上面の低圧部の発生ってこの説明から必然的に導かれ
る?

そういう点では下向き流れの反作用(合理的で正しい説明)
で揚力を説明している場合と同じではないのかな?

(出発渦とKutta条件)
ttp://members.tripod.co.jp/uirs/air/vortex.html
ttp://taku3.anc-d.fukui-u.ac.jp/~tak/project/vortexJ.html
ttp://www.ohashi.f2s.com/yacht/record/12sakurai.html
144ご冗談でしょう?名無しさん:01/09/16 23:54 ID:FsvjmdZg
>>143
循環を示せれば上面と下面の流れの速さの違いは必然的ではないかと。
というか循環の定義からそのまんまじゃないかな。
これから例のベルヌーイを使って翼上面で低圧ってなるよね。
まあ循環から説明するのは直感的ではないとは思う。

>そういう点では下向き流れの反作用(合理的で正しい説明)
>で揚力を説明している場合と同じではないのかな?
同じだと思う。
ところで下向きになる仕組みって分子論のやつじゃなくて流体で
説明できたっけ?
145ご冗談でしょう?名無しさん:01/09/17 00:38 ID:sbfntfUg
>>144
う〜ん、やっぱり何となくしっくりこない。
でも言ってる事は分かるよ。「循環の定義自体が
流れが曲がる際に翼の上下面で速度差が付くこと
を要求している」ってことだよね。

でも仮に翼上面と下面の速度差がつかないとした
場合、後縁で同時到着する必要がないから、流れ
の向きは変わることはできるんじゃないのかなと
も思うけど...?
(別にからんでるわけじゃないんで荒れないでね...)

つまり、流れが曲げられる際にどうして翼上下面
で速度差が付くかっていう問いに対する明確な答え
ってのはなんだろうね?

あと、流れの向きが変わるのは分子論説明とは直接
関係無いよ。こっちの方が循環より流体的に直感的
に判りやすんじゃない?流れに対して斜めに板を置
けば板に沿った方向に流れが変わるって事でしょ。
146ご冗談でしょう?名無しさん:01/09/17 03:08 ID:ZaFXDKDA
循環保存による、循環不滅から揚力の不滅を導くやり方自体は
解らなくも無いが、飛行機の翼だからね。
翼と空気以外にも、さまざまな妨害要因があるから、最初に
循環があるから、循環は保存され続けなければならない、という
主張はどうも頼りない主張に聞こえてしまうのだよ。
ちょっとしたそよ風ひとつで、飛行機が飛んだり飛ばなかったり。
そんな感じを受けるのだが。
147ご冗談でしょう?名無しさん:01/09/17 23:45 ID:9Tzt1fCs
そよ風ひとつで飛ばないって事は無いだろ。
うしろから突風が吹いたらヤバイかもしれんが。
ところで
>翼と空気以外にも、さまざまな妨害要因があるから
とは例えば何?
148146:01/09/18 03:46 ID:ZCTLC8V2
循環という量は、非常にデリケートだからじゃないかな?
誰かが前カキコしてくれたみたいだけど、1%とかその程度の速度(絶対値だから
速さかな?)が変化しただけで、循環が大きく変動するとか。
どういう例だったかな?確か2次元で(r,Φ)の位置にある速度ベクトル

(rcosΦ,rsinΦ)+1/r(cosΦ,-sinΦ)となる奴。第2項が循環に寄与するけど
r=100の時、循環に寄与する成分は、他の成分の1万分の1以下の大きさに
なる。
僅かなそよ風程度でも、循環が大きく変動する可能性ありとみたが、如何?
149ご冗談でしょう?名無しさん:01/09/18 10:39 ID:K9zB8ENI
145だけど、いろいろ考えて「流れが曲げられる際にどうして
翼上下面で速度差が付くか」の答えらしきものを見つけた。
(と思うので、皆さんのご意見聞かせて)

以前からちょっと出てたと思うけど、やはり「慣性」と「粘性」
が本質的な原因だと思う。板で流れが曲げられる際の外側の流
れを考えると「慣性」のため真っ直ぐ進もうとするけど、同時
に「粘性」のために板面に沿って流れようともする。この2つ
の性質のために流体(水,空気など)は圧力(静圧)が下がるん
じゃないかな。ただし、ここで膨張するわけではなくて、圧力
が下がった分だけ流体の速度が上がるってこと。圧力と流体の
速度にはある拘束関係(その特殊な場合の近似式がベルヌーイ
の定理)が推定されるってことで良いよね。どう?

--- 続く ---------------
150ご冗談でしょう?名無しさん:01/09/18 10:40 ID:K9zB8ENI
--- 上の続き -----------

ちょっと話しを分かり易くするために連続カキコするけど、
「飛行機って何で飛べるんですか?」という問いに対する、
理解しやすくて間違いのない説明はやはり、「翼によって空気
の流れを下向きに曲げ、その反作用として翼が上向きの力(=
揚力)を得ているから」だと思う。これで質問に対する答えは
完結。この説明により多くの小中学生から大人までが直感的に
理解できると思う。

ただし、「何で飛べるか」とはちょっと別に、揚力が発生して
いる時に翼の上下面で圧力差が発生しており(その圧力差が揚力)
且つ、翼上面の流れが速いという事実が存在している。

で、「その圧力差と翼上面の速い流れはどうして生じるのか?」
という別の質問(と敢えて言うが)の答えが>>149 ってわけ。

---- 続く ------------
151ご冗談でしょう?名無しさん:01/09/18 10:41 ID:K9zB8ENI
---- 上の続き --------

もちろん、「飛行機って何で飛べるんですか?」という問いに
対して、>>149の説明をして、その結果として生じる翼上下面の
静圧の差により揚力が発生して飛べるんですよ。といっても良い
と思う。こちらが所謂、標準派(ベル派)の立場から見た説明の
仕方なのかと思う。

飽くまで「慣性」と「粘性」で静圧の上下差の発生の説明をして、
「静圧と流れの速度の拘束関係」で翼上面の速い流れを説明する
ってこと。この観点は今までの標準派(ベル派)には無かったん
じゃない?

ただし、繰り返し言うけど、個人的には>>150のように反作用と
説明された方がよっぽど合理的で直感的に分かりやすいと思う。
152ご冗談でしょう?名無しさん:01/09/18 11:01 ID:kfHFQhPo
「静圧と流れの速度の拘束関係」で翼上面の速い流れを説明する
端的に言えばベルヌーイの定理ってことね。
153ご冗談でしょう?名無しさん:01/09/18 11:06 ID:K9zB8ENI
>>152
「静圧と流れの速度の拘束関係」の内、実在しない
特殊な場合の式(=近似式)が「ベルヌーイの定理」
だね。

ただ、ポイントはそんなところじゃなく、標準派(ベル派)
は速度が速くなる原因、あるいは低圧になる原因を今まで
説明できていなかったでしょ。それじゃイカンという事。
154ご冗談でしょう?名無しさん:01/09/18 20:19 ID:6ToRSE22
>>148
循環という言葉にとらわれすぎると本質を見失うのでは。
前スレでも出てたけど循環を計算するときのパスはよく考えて
設定する必要がある。
そよ風と一様流の間には渦層が存在してるわけでこいつまで含んで
循環を計算するのはまずい。
極論するとそよ風なんて考えなくても出発渦まで含めばどう考えても
循環はゼロになってしまう。
大事なのは翼近傍の循環だよ。
155ご冗談でしょう?名無しさん:01/09/19 00:44 ID:H3IvZXbk
>>150
>「飛行機って何で飛べるんですか?」という問いに対する、
>理解しやすくて間違いのない説明はやはり、「翼によって空気
>の流れを下向きに曲げ、その反作用として翼が上向きの力(=
>揚力)を得ているから」だと思う。これで質問に対する答えは
>完結。この説明により多くの小中学生から大人までが直感的に
>理解できると思う。

揚力が流れを曲げる反作用ってのは確かに直感的で正しいんだけど
これだけで完結とはいえないのでは無いかな?
なぜ流れが曲がるのかって事に対して論理的に示そうとすれば
結局は>>142のような筋書きを示さないといけない。
まあ小中学生相手なら仕方ないかもしれないが大学生相手に
流れが曲がることぐらい直感で分かるだろ!って訳にもいくまい。
156ご冗談でしょう?名無しさん:01/09/19 01:07 ID:???
反作用という言葉をどう解釈するかってことかな?

物理学が確立した大きな原動力はAtomicな考え方と、幾何学的な
単純性。Newtonの作用・反作用の法則は、Atomicな考え方と幾何学
的な単純さで受け入れられた。
「揚力が流れを曲げる反作用」という考え方は、Atomicとは思えない。
幾何学的にも、(揚力は厳密には、完全に下向きではなく、抗力の
成分も持っているから)、簡潔とは言えない。

因果関係として、「流れが下に曲がるから、翼はその反対に上がる」
という言明は例え正しくても、少なくとも近代以降の物理学では
少し語弊があるんじゃないのかな?

広い意味で反作用という言葉を解釈すれば、そう言える部分もある
かも知れないが。

応答という言葉を使ったら?
157ご冗談でしょう?名無しさん:01/09/19 10:12 ID:5bhyYjBg
>>155
>なぜ流れが曲がるのかって事に対して論理的に示そう...

「空気には粘性があるため、流れに対して傾いて位置する翼に
沿って流れる。そのため、結果的に翼によって空気の流れが
下向きに曲げられ、その反作用として翼が上向きの力(=
揚力)を得ているから」でいいんじゃない。

>>142って、圧力差や翼上面の速度の増加の説明にはなって
ないんじゃないかと思うんだけど...。

>>156
なるほどそうなの...(?)。
まあ、漠然と広い意味で反作用と言ったと思ってください。
158ご冗談でしょう?名無しさん:01/09/20 02:54 ID:iLmugX6.
>>157
粘性があるから沿って流れるって理屈がわからん。
もしかして普通と違う意味で粘性って言葉使ってる?
159ご冗談でしょう?名無しさん :01/09/20 03:44 ID:g/97p4tU
>>158
それって第2スレでガイシュツじゃなかったっけ?
粘性があるっていう事はせん断応力が働くって事でしょ。

その時の板表面の速度分布を考えると、板表面で速度0
(と見なすかな?)、板表面から離れるに連れて速度上が
ってくる。その速度分布をイメージできれば、流れが
湾曲に沿って流れるのがイメージできるんじゃない。
160ご冗談でしょう?名無しさん :01/09/20 03:50 ID:g/97p4tU
第2スレ、もう見る事ができるみたいね。
このスレの >>2 にリンクがあるけど、一応ここにも再掲。
http://cheese.2ch.net/test/read.cgi?bbs=sci&key=992901156
161ご冗談でしょう?名無しさん :01/09/20 04:20 ID:/6ptyk2A
>>158-159
何度も出てるが、粘性の有無による流れの違いは
模式図で書くとこうだね。
http://journal.msn.co.jp/worldreport/s_suzuki_fig6.gif
     ↑
このページの図6
http://journal.msn.co.jp/worldreport.asp?id=010710s_suzuki&vf=1
162ご冗談でしょう?名無しさん:01/09/20 18:50 ID:eZF0NmMA
>>161
正確には循環の有無だよね。
粘性があるとクッタ条件を満たすような循環が無いといけないと言うのが
流体力学的な説明で、確かにそうなんだけどじゃあなんでその循環が
できるの?それを説明してよっていうのが元スレ27氏が言ってたことだったと思う。
彼は分子論的にそれを説明しようとした訳だけれど、まあ難しいんじゃないかな。
分子間力で流れが曲がることがまあ説明できたとしても、それによってできる
循環がちょうどクッタ条件を満たすってのは都合がよすぎるしなあ。

>>159
粘性がある場合の速度分布ってのはReに依存するからその説明だと
粘性の大きさによって流れが翼に沿えたり沿えなかったりするかも
しれないと思ったりした。
163ご冗談でしょう?名無しさん :01/09/21 01:11 ID:6gYclAZc
>>161
循環なんてものは、流れや揚力等の実際に起きている現象
をより詳しく解析するために導入した「概念」にすぎない
んだから、あまり拘る必要はないんじゃないか?

自分自身うまく説明できないけど、クッタ条件を満たすの
も「粘性(適当な値の粘性といった方が良いか)」がある
からだよね。

「粘性」の無い流体、例えば超流動ヘリウムなんてのは、
どんな小さな孔や、小さな曲がりくねったところにも入っ
ていくし、ビーカーの壁面を自分で伝って出て行くんだ
よね。(といいつつ自分で扱った事ないから知らないけど)

つまり、「粘性」がなければ、翼下面の流れは後縁で周り
こんで、下図6の向かって左の状態になると思うけど。
http://journal.msn.co.jp/worldreport.asp?id=010710s_suzuki&vf=1
164ご冗談でしょう?名無しさん:01/09/21 01:35 ID:gg7bhaeo
>>163
粘性が無きゃそもそもKUTTA条件なんて考えないでしょ。

ところで素朴な疑問なんだが粘性があるから流れが表面に沿うって
説と>>163で言われてることは矛盾してないかな?
165ご冗談でしょう?名無しさん:01/09/21 07:55 ID:???
>>164
粘性が無くても流れは表面に沿うでしょ。問題はその沿い方。
円柱を非粘性流体中に置くと上流側と下流側で表面に沿う流れは
対称になって抗力は無くなるけど、粘性流体中では非対称になり
抗力が発生するよ。
166ご冗談でしょう?名無しさん:01/09/21 08:04 ID:6gYclAZc
>>164
165で既にどなたか答えていただいたが、沿い方だと思う。
せん断応力を発生させつつ曲面に沿って流れるのか、単に
空間を埋めるべく入り込んでいくかの違い。
167ご冗談でしょう?名無しさん:01/09/22 02:33 ID:izti8Tm2
ずっと前から聞きたかったんだけど、前スレの10番の方法で揚力は得られるのか?
或いは揚力はどのように変わるのか?粘性で流れが曲がるという人に質問

前スレのURLは
http://cheese.2ch.net/test/read.cgi?bbs=sci&key=992901156
前スレ10を引用。(させてもらいました)
----------------------------------------------------------------
10 :ご冗談でしょう?名無しさん :2001/06/19(火) 21:53 ID:NmnrJgm2
もんだい

今、断面が非常に薄いベルト状の金属テープがあるとします。
両端は繋がっており、ちょうど女性が使うガーターベルトみたい
なものと想像して下さい。
これを翼に何本も巻き付けてそれを覆います。
ただし、ベルトの表面は十分滑らかで、ベルトが露出している
のは、翼上表面だけで、下半分のベルトは、翼の中に入り込んで
隠れているものとします。
下手な図ですが、一応この状態での翼の断面図を

           ↓ベルト
     ┌──────────────────────────┐
     │   ──   ↓翼上面(のつもり)                 │
     │ ──   ──────              │
     ├─           ─────         │
   ──└──────────────────────────┘
 ──                          ──────
─────────────────────────────────────
↑翼下面(のつもり)

それを翼の前後方向に、何枚も並列に並べて覆います。
上から見ると、翼は縦に沢山のベルトで覆われているように
見えるでしょう。
ベルトは、翼前端で翼の中に入り、翼後端で、翼から出て
再び上面に表れます。(ベルトは十分弾力があるとし、弛ま
ないとします。かつ表面も十分に滑らか)
今、ベルトは翼内部のモーターによって高速回転出来るように
なっているします。つまり、翼上表面は、静止状態でも任意の
速度で(表面だけが)動くことが出来るようになっているというわけです。
つまり、表面だけ動かすことが出来る翼(藁)を作ってみたというわけです。
翼の表面にベルトコンベアを何ラインも設置したということです。

境界層の理論から言って、ベルト表面の空気の速度はベルトの
移動速度と同じになる筈です。(空気が翼全体の移動速度を
認識できるというのなら話は別になりますが、この場合、明らかに
物理的証明が必要になります。空気はヴァカでしょうから、目についた
表面だけの速度を認識する筈です。)
飛行中にベルトを適切に動かすと、空気力学的には、より高速に飛ぶ
ことと同じになるでしょう。
この時、果たして循環は増大するでしょうか?
下降気流の増大についてはどうでしょうか?
揚力の寄与はどうなるのでしょうか?
もし寄与すると言うのならば何故(比較的ローコストで可能な
技術であるのにもかかわらず)実現されていないのでしょうか?
168ご冗談でしょう?名無しさん:01/09/22 09:37 ID:O.CJuNmY
>>167の派生問題に移る前に、3スレに渡って論争が続いて
来た「飛行機って何で飛べるんですか?」っていう問いには
空気流れの「慣性」と「粘性」で説明できた(>>149-151)
って事で良いのかな。とすれば、めでたしめでたしでひと
まず閉幕ということですか。
169ご冗談でしょう?名無しさん:01/09/22 15:11 ID:???
その答を出す前に、派生問題について考えてみようということでしょ。
結論は、最後になっても良いんじゃないかな?
170ご冗談でしょう?名無しさん:01/09/22 16:28 ID:???
>>169
結論も何も、「飛行機は何故飛べるのですか?」と派生問題
とは直接関係ないと思うんだが...。「粘性+慣性」の説明
でうまく行かないって言うんなら、派生問題に移る前に白黒
はっきりさせるってのはどう?
(というより、現状、その説明の否定材料は無いんじゃないか。)

ひょっとして君は亡霊「元スレ27」か?
171ご冗談でしょう?名無しさん:01/09/22 17:23 ID:???
>>170
「元スレ27」(>>2参照)
は、粘性が慣性より重要ということを言っていたね。
慣性も粘性も、レイノズル数を決める重要なファクターだから、
それを踏まえた上でどちらが重要であるかの違いを挽き出すのが
>>168さんが再引用してくれた前スレの10さんの問題。

慣性と粘性が重要ってのは、最初からわかってるんじゃないの?
172ご冗談でしょう?名無しさん:01/09/22 19:56 ID:???
>>170
「粘性+慣性」の説明は黒。

慣性とは、外力が作用しなければ運動量は保存するということ。
だから
「粘性で引かれるが慣性のために気圧が下がり流速が増す」とは
「外力が作用しても運動量は一定であるため静圧が下がり速度が増す」
というドキュソな主張になる。

応答のことを書いたレスもあったけど、実際の気流は翼に沿って
流れているわけだから、低圧になる根拠として「慣性」は不可だね。

レベル低すぎ。「元スレ27」も成仏できないぜ (藁
173171:01/09/22 22:58 ID:???
171!=172だからね
169=171だが。
174ご冗談でしょう?名無しさん:01/09/22 23:27 ID:???
あぁ、また気違いが戻ってきた・・・。
175ご冗談でしょう?名無しさん:01/09/22 23:35 ID:2/z0jMh6
「粘性+慣性」説について2点ほど気になった。
1)この説だとKutta条件が満たされる必要は無い。(>142の説明はこの点を満足している。)
2)「粘性+慣性」説だと粘性が大きいほど揚力は大きいことになってしまう。
176ご冗談でしょう?名無しさん:01/09/22 23:36 ID:idAhSo.E
まだ27ってぐぁんばってるの?

翼に沿う流れに、慣性によって翼に沿わない方向の力が作用し、沿わない
流れ成分が生じる。ー>翼表面の空気密度が下がる?
という主張なのかな?
177ご冗談でしょう?名無しさん:01/09/23 00:26 ID:YAKP26q.
>>172
電車が急ブレーキかけてロックして、車輪とレールの摩擦のみ
で止まる時って、すぐには止まらないんじゃない?これって
慣性じゃないの?外力(摩擦力)は働くし、運動量は減ってくよ。

>>175
1)については、翼上下面からの速度が一緒になった時が定常
状態だという事だよね。(適当な)粘性があるから流れの回
りこみが起きずに、これが定常状態となるんでない?
2)そんな事ないんじゃない?速度、粘性、流体密度の兼ね合
いじゃないかな。

>>176
全く君は....。
178ご冗談でしょう?名無しさん:01/09/23 00:27 ID:???
>>167
答えは簡単で、循環は増加するよ。だから飛んでるときにベルトを回せば
揚力も増加する。実現されないのは「ローコスト」じゃないからでしょ。
翼の断面の形状は揚力の重要なファクターだから、それを崩さずに
高速にベルトを回転させるのは難しいよ。ローラーでしなやかに回転
できて、かつバタつかないで形を保つためには、どんな材質が最適なのか
ちょっと想像できない。

先日767−300に乗ったときに離着陸時の翼の形状を見てたけど、
前縁側でも翼を斜め下に伸ばしてキャンバーを大きくしてた。
圧力差の大きい前縁側も可動になったのは、技術の進歩なのかな。
179ご冗談でしょう?名無しさん:01/09/23 00:48 ID:???
>>177
電車がすぐに止まらないのは、ブレーキの摩擦力が小さすぎるので
時間を長く作用させて力積を大きくしないと、運動量をゼロにでき
ないから。慣性とはチョト違う。

翼周りの気流も、翼の形状に沿わなかったり、「遅れて」沿うなら
電車は急に止まれない場合に相当するけど、流線は翼に沿ってる
から違うでしょ。
180ご冗談でしょう?名無しさん :01/09/23 01:10 ID:YAKP26q.
>>179
177だけど、正に「遅れて沿う」イメージなんだと思う。
だから、こういう説明をしたんだけど。流線についても、
翼が迎角持っていて且つ曲表面を持っているから、表面
に対して常に平行に沿うことはあり得ないよね。
(ちゃんとまともな事書いてるんならsage止めようよ)
181ご冗談でしょう?名無しさん:01/09/23 01:34 ID:???
>>180
流れが最も翼に沿っていないのは前縁部で接平面が流れに対して
垂直になるところでしょ。この部分の圧力は高くなっているね。
コンシステントな説明にはなってないよ。
さらに、運動量は動圧と等価だから
「外力が作用しても動圧が変化するのに時間がかかるため
 静圧が下がり、その結果動圧が上がる」
って主張になるけど、これは変だと思わない?
182175:01/09/23 01:50 ID:BptHto/E
>177
>1)については、翼上下面からの速度が一緒になった時が定常
>状態だという事だよね。(適当な)粘性があるから流れの回
>りこみが起きずに、これが定常状態となるんでない?
この翼の後縁に注目するのはむしろ>142の説明だよね。この考え方なら
納得がいくんだが「粘性+慣性」説では速度変化(つまり循環の変化)を生むのは
翼上面での粘性と慣性という事になっていて、これでも循環は説明できそうだけど
この循環がKutta条件を丁度満たすと考えるのは少し楽天的過ぎる。

>2)そんな事ないんじゃない?速度、粘性、流体密度の兼ね合
>いじゃないかな。
「粘性+慣性」説では当然そうなるわけだけれども、揚力はご存知のとおり
  L=流体密度*流速*翼回りの循環
となるわけで基本的には粘性には依存しないです。
183175:01/09/23 02:01 ID:BptHto/E
>>181
>>149-151を読むと前縁部で圧力が高くなることは問題ない。
運動量と動圧のくだりもちょっと読み違えがあるんじゃなかろうか。
184ご冗談でしょう?名無しさん:01/09/23 02:56 ID:GSI.TNag
>>182
177だけど、良く分からないが、適当な粘性があるから
後縁では流れの回り込みが起きないってのは説明に
ならないんだっけ?

あと揚力の大きさだけど、「粘性+慣性」の説明でも
粘性だけが大きくなっても揚力は増えないんじゃない?
ってゆうか、どうして揚力が増えると考えられるの?
185ご冗談でしょう?名無しさん:01/09/23 02:57 ID:???
>>1
飛行機と名乗っていて飛べなかったらおかしいだろ?
もし飛行機の名が陶磁器とか歩行器とかだったら飛べないけど、なんたって飛行機だもん。
そりゃ飛ぶさ。
飛行機は飛行機だから飛べるんだ。
というわけで以上で終了な
186ご冗談でしょう?名無しさん:01/09/23 03:00 ID:GSI.TNag
184に付け足し。
飽くまで「粘性+慣性」の説明では、とりあえず定常
状態を考えているから、循環が発生している(上面の
速度が速くなっている)ことも、後縁で回り込みが
起きていないことも定性的概念としては説明できてい
る気がするんだけど。
1871:01/09/23 13:15 ID:???
>>185
だから、その手の回答は、激しくガイシュツで、元スレの最初の方で
既に出ている。(>>2参照ね)
元スレ・2スレのURLは、このスレの途中で出てるから、調べて。
もし続くようだったら、>>200当たりでもう一度書くよ。(覚えやすい番号に)
188******結論******:01/09/23 15:36 ID:XEpE0D3E
パイロットがいるから飛べるのだ!
189ご冗談でしょう?名無しさん:01/09/23 15:50 ID:yax2CRn2
>>186
175が聞いてるのは何故そのKUTTA条件を満たす定常状態に
落ち着くのかということだと思う。
それと「粘性で引かれるが慣性のために気圧が下がり流速が増す」のであれば
粘性の大きさが気圧低下の大きさに影響を及ぼすと考えるのが自然では?
190ご冗談でしょう?名無しさん:01/09/23 16:33 ID:.zN3Fd0Q
いやさ、話が戻ると言うか、既出なことを持ち出したりすると、呆れられる
かと思うけど。
例えばの話で恐縮ですが、例えば電流というものを説明するのに、電子の
移動がどうとか説明するよね。
ある意味では間違ってない、真空内、あるいは、半導体内部では電子、あるいは
そのない部分(ホール)が電流の流れに寄与してる。しかし、現実の多くの
導体(あるいは、空間)においては、磁界と電界の相互作用によって、電磁
エネルギーとして電流、電圧の伝送が生じる訳ですよ。
だからといって、送電線の電流の流れ、回路網の電流の流れを電子の移動で
あるかのように説明しても誰も目くじらをたてないでしょう。

だから、その辺の区分けというか何かがあってもよさそうな気がする訳。
それは間違いとか基地街とか決め付けるんじゃなくて。説明として、それは
それで良い、でも、本当はこれこれだよ。
というような方向への進展があっても良いかと思われ。
191-:01/09/23 17:03 ID:LrhlATEc
その画像ならココ
http://members.tripod.co.jp/bingo852/
192ご冗談でしょう?名無しさん:01/09/23 21:51 ID:???
>>190
電磁波と電流の区別がついてないとオモワレ。
193飛行機マニアだが・・・:01/09/23 22:17 ID:kLPlvRfM
超音速飛行中の飛行機は、なぜ、墜落しないんですか?
超音速飛行中は、どんな翼形でも揚抗比はほぼ同じですか?
194ご冗談でしょう?名無しさん:01/09/23 22:20 ID:???
実のところ、飛行機は飛べません。

飛行機はアホだから自分が飛べないということに気づいていないのです。

まれにそれに気づいてしまう飛行機がいます。

そして気づいてしまった飛行機は墜落してしまうのです。
195ご冗談でしょう?名無しさん:01/09/23 23:05 ID:G.1QT1f.
>>193
ちなみになんで墜落すると思うんでしょうか?
196193:01/09/23 23:35 ID:JVwMlQF6
ショックストール?
翼の上下の気圧差が吹き飛ぶのでは?と言う疑問。
単に風圧で浮いてるんですか?
197ご冗談でしょう?名無しさん:01/09/23 23:56 ID:sjHwUcVA
>>196
下面は衝撃波で圧力上昇・上面は膨張波で圧力低下。
198193:01/09/24 00:11 ID:iX1j148Q
おおなるほど。
199ご冗談でしょう?名無しさん:01/09/24 20:18 ID:9uQaqiK.
答えが出たようなので終了
粘性よりも、慣性が決定的理由。
粘性は、圧力勾配による加速が、速度分布によってズレるだけ
の話し。本質は圧力勾配

以上
200ご冗談でしょう?名無しさん:01/09/24 20:24 ID:Sm4.BGdw
>>199
186だけど、>>189にレス付けそびれちゃった。
粘性と慣性のどっちが本質というのはあまり意味
無くて、両方必要という事だと思うけど....。
まあ、いいや、最後にまとめてよ。

「飛行機って何で飛べるんですか?」
201ご冗談でしょう?名無しさん:01/09/24 20:26 ID:???
>>199
基地外
202ご冗談でしょう?名無しさん :01/09/24 20:28 ID:Sm4.BGdw
186だけど、>>189にレス付けそびれちゃった...

>>199
粘性と慣性のどちらが本質というのはあまり意味が
無い議論で、両方必要だという事だと思うが...。
まあいいや、で結局、最後にまとめてよ。

「飛行機って何で飛べるんですか?」

小中学生でも分かるような説明キボンヌ。
203ご冗談でしょう?名無しさん:01/09/24 20:30 ID:Sm4.BGdw
202だけど二重投稿ゴメソ
204ご冗談でしょう?名無しさん:01/09/24 23:44 ID:dr9DuFUw
粘性が極めて小さいとき揚力は発生しないわけだけど
粘性と慣性で説明しようとするとこれっておかしくない?
それに分子論で説明しようとしても難しそうだね。
要するに揚力発生の仕組は一筋縄では行かないみたいね。これが結論?
205ご冗談でしょう?名無しさん:01/09/25 03:06 ID:c9aDHqUQ
>粘性が極めて小さいとき揚力は発生しない

これ、嘘。
粘性があろうと無かろうと、揚力を物体にかかる圧力の総和であると定義する限り
少なからず、揚力は発生する。
定常状態で、揚力が発生しないものがある(というか、そういう定常状態
しか有り得ないのでは?というのがギョーカイの中心説。
それに対してジューコフスキーの流れ(円の周りの一様な流れを、ジューコ
フスキー変換でMapした奴)は、揚力が発生する定常状態が有り得るという反
例みたいなもの。(オイラー方程式を弱い意味で解釈すると、こういった解も
認められる。古典解ではないけど。)

粘性が小さいと、最初は揚力があっても、やがてスーっと抜けていくみたいに
それが無くなるってことじゃないの?
流れが定常(各点での速度ベクトルが時間と共に変わらない)になるまでは、
揚力があってもおかしくは無い。
気体分子論で説明出来なくも無いが、数学的に厳密なモデルを作るのはすごく
面倒だし、説得力があるかどうか。
分子間力が同じでも、分子が点か、剛体か、それ以外かで、話が大分変わってくる

分子が剛体であるとしたら、そのジャイロ運動を取り入れたりして、幾らでもヘ
ンなことでっちあげられるんで、(分子論での説明は)あんまりやらないほうが
良いと思うよ。
206Ledの教え子:01/09/25 03:20 ID:???
はーー!(汗"
皆さんまだやってたのですね♪
「粘性」ごときに拘っててはまともな論理は得れませんよ。
所詮は、慣性だけで理由付け可能だしね。

# それにしても時間が掛かったこと...(笑
207ご冗談でしょう?名無しさん:01/09/25 11:22 ID:YDLvAX42
>>206
それでは、「飛行機が飛べる理由、つまり翼が揚力を受ける理由」
を簡単に100字以内で記述してくれることをキボンする次第で
ございます。
208ご冗談でしょう?名無しさん:01/09/25 23:22 ID:sUWtXv5M
>>206
慣性だけで説明出来るってのはどうしてですか?
圧縮性や粘性、循環との関係が今一つわからんのですが、教えて下さい。
209ご冗談でしょう?名無しさん:01/09/26 00:41 ID:xBbW15Vw
>>208
Ledはほっとくとして圧縮性は置いといていいんじゃないの?
液体でも揚力は発生するし。
210ご冗談でしょう?名無しさん:01/09/27 04:20 ID:sNbUUf32
かつての白熱議論も冷めて、60という深さに沈んだこのスレ。
レスも途絶えたと見えるので、まとめ。

正常神経波:揚力は圧力差であり、その圧力差が安定して維持されるのは、翼が
横に長いことと、空気慣性によるもの。粘性効果は、空気の気流内部での速度差が
圧力に若干影響を与えるという程度で、揚力の存否には直接関係無い。安定性には
重要な役割を果たしているだろうが。(詳細は実は今でも精力的に研究されている
らしい)

二次元至上主義派:何が何でも、2次元翼理論を基本として理解しようとする。
循環という量が非常に好きで、これにこだわる。ジューコフスキー翼周りの流れに
神秘を感じる。

分裂派:いわゆる、粘性粘着派。揚力発生を、翼と空気
の間の他に還元することが出来ない基本原理とみなし、物理学とのつながりは
ニュートン力学における作用反作用則のみとすると同時に、それは空気分子が
剛体であることによる、ジャイロ効果(現象)に起因する、特異的翼粘着作用
と分子間力による粘性発現によるものとする。
剛体分子に還元できるものは認めるが、それ以外はすべての諸事象は還元不能
であるという、還元主義とも反還元主義ともとれる、分裂病的な考え方。

この3つの考え方のバトルでした。傍観者として結構面白かったことを
感想として述べさせて頂きます。
211Ledの教え子:01/09/27 05:00 ID:???
>>208
> 慣性だけで説明出来るってのはどうしてですか?
> 圧縮性や粘性、循環との関係が今一つわからんのですが、教えて下さい。

慣性なけりゃ衝突による反作用は起こらんよ。
基本的に衝突現象であることを理解すべきだな!(笑
212ご冗談でしょう?名無しさん:01/09/27 09:33 ID:l5t3onEA
>>210って、かなり偏見に満ち溢れてないか?
単なる傍観者がそんな見解しか得られないのだとすると、
説を述べている者が悪いという事か...(?)

論理的に考えれば、>>149-151 と>>142が合理的説明
じゃないの?つまり、いろんな見方があって、
 1)流れを下向きに曲げる反作用での説明
 2)出発渦の反作用により発生する循環での説明
 3)流体の粘性と慣性による説明

どれも正しいと思うよ。お好みでどれでも良いんじゃない?
3)での説明が筋道が通っていて且つ翼上面の圧力差を直接
説明するから分かりやすいかもしれない。(いわゆるベル派
の説明では無い事に注意) 1)の説明は感覚的に理解しやす
く、小中学生及び文系向きかもしれない。 2)(=>>142)の
説明でも良いかもね。(ただし、上下面の流れの速さが同じ
になり渦の放出が止まるのも、循環が翼周りに常に固定され
るのも「粘性」+「慣性」のおかげじゃないかな。)
213ご冗談でしょう?名無しさん:01/09/27 09:42 ID:l5t3onEA
212だけど、補足。

上はいずれも流体での説明ね。分子論で説明できれ
ばそれは間違いではないと思うが、それはビリヤード
球を棒で突く時の運動を、ビリヤード球が相互作用を
有する非常に多くの原子の集まりであると考えて、
運動を解析するようなものだと思う。

事実、揚力に関しては、多分世の中ではそんな事やら
れてないし、この3つのスレでも挑戦した者もいたが
うまくいかなかった。(間違いだとは言ってないよ。)
214ご冗談でしょう?名無しさん:01/09/27 09:48 ID:l5t3onEA
212だけど、ゴメソ更に補足。

分子論で説明を試みるとすると、>>212の1)で説明
する方法と3)で説明する方法の2つがあると思う。
( 2)でもできるかもしれないけど、イメージできない)

つまり、分子の速度の下向きの平均値が翼の後ろで
(望ましくは機体重量を支えるだけ)増える事を示す
か、あるいは翼上面の圧力が低下しかつ翼面に水平方
向の速度が大きくなる事を示すか、のいずれかという
事。まあ、いづれにしろ、あまり意味のない事かとは
思う。
215ご冗談でしょう?名無しさん:01/09/27 19:54 ID:DuHteGMo
はぁ〜。
循環という物理量は、保存量を示すことはあるけど、揚力と明確に結びつく
のは2次元だけでしょ。
そして、渦と循環を混同している。
だから2次元固執と言われるんだな。
216ご冗談でしょう?名無しさん:01/09/27 23:51 ID:Md/qidWI
>>215
2次元で考えないとややこしくてしょうがないだろ。
ところで誰に言ってんだ?
217ご冗談でしょう?名無しさん:01/09/28 03:47 ID:WO196hVU
>>216
2次元の理論と3次元の理論の間には、想像もつかない隔たりがある。
例えば、音の伝播。音源がごく短い衝撃音をたてたとして、
二次元平面では、音源付近の音はなかなか減衰しない。一方3次元で考えると
音源付近は、音が到達した後はすぐに減衰する。波の伝わり方は全然違う。
次元の違いが本質的に現れる例。
物理現象というよりは、数学現象といったところかな。
だから、2次元に固執すると、ややこしいとか簡単という次元の問題でなく、
本質的に小さな世界のみで考えることになって大きな間違いをすることになる...
らしいね。聞いた話では。
218ご冗談でしょう?名無しさん:01/09/28 07:33 ID:svK.zURg
>>215
212だけど、書いてる本人は渦と循環を混同している
つもりはないんだけど・・・。まあ、いいや。別に
数学的に証明してるわけでなく、揚力発生のコンセプト
を示してるだけだったんだけど、3次元だとこの
コンセプトは使えないということかね?

まあ、どうでもいいや。分かりやすいように、>>212
1)と3)だけにしておくよ。>>149-151に詳細があるけど、
どっちの説明でもいいんじゃない。もちろん、この2つ
以外にも(+渦での説明以外にも)説明の仕方はあるか
もしれないね。
219ご冗談でしょう?名無しさん:01/09/29 03:13 ID:9fZaz60w
212にケチつけるつもりじゃないが、1)の説明は厨学生や、高校性や文型の人には
解りやすいというより、不安感と誤解を与える説明じゃないかな?
空気と翼がぶつかって....というんじゃなくて、引き合って反動でって奴だから
な。
3)についてだけど、それこそ分子論レベルで考えると、(分子の内部状態を
考えると)、簡単に慣性や粘性だけで因果関係を言い切るのは、これまた誤解を
与えると思うが。(こちらは、クロートっつーか、セミプロ・マニア筋に)

揚力ってのは、起こるときは起こるし、起きない時は起きない。起きる時はその
理由を特定できるが(それこそ慣性や粘性が濃厚だろう)、その逆が言えない。
だから、ベルヌーイの公式のような、因果関係をぼかした、論理関係で説明する
のが無難じゃないのかな?

曰く
揚力が起きているとするでしょ。この時、翼の上の流れはベルヌーイの等式を満たしてい
るんだ。気流の水平速度は、(別に翼から水平成分の力を受けるようには思えない
から)余り変わらないんだ。だから上下の流れは同時到着する。しかし、この
場合、上の流れの速度は経路が長いから、速く流れなけりゃならない。
そうしなければ、水平成分の速度を変えなければならなくなって、空気の流れが
調和しない。だから、ベルの等式から圧力が下がって流れなければならない。
揚力が起こらない場合は、水平成分に乱れが出る場合で、この場合が失速。
どっちになるかは、神のみぞ知るが、経験的に揚力は起こる。

って具合かな?(行間に込められた意味は深そうだが)
220ご冗談でしょう?名無しさん:01/09/29 04:03 ID:p5BZht.Y
>>219
言いたい事は分からんでも無いがさすがに同時到着はまずかろう。
221ご冗談でしょう?名無しさん:01/10/01 10:14 ID:???
はぁー!
ようやく終わったみたいね。
中学生さん、余り大人を困らせること聞いちゃだめだよ。
にしても、凄まじい白熱議論でしたね。

ー理想気体に粘性がある。
ー空気分子のジャイロ効果で、翼に粘着する。

発想の豊かさには実に驚かされました。
ま、ここで叩かれたことを踏み台に、一つ大理論でも建設して下さいな。藁)


-------------このスレは無事終了しました。-------------
222ご冗談でしょう?名無しさん:01/10/09 03:22 ID:C5ZD2Oog
age
223ご冗談でしょう?名無しさん:01/10/14 01:07 ID:IUaGaUMI
リンクから辿って、このスレに到達して、拝見させて貰いました。
最初から、尾翼失速のこと忘れているのか、知らないのか。
ダウンウォッシュって、尾翼にとっては、恐ろしいもの。
ダウンウォッシュ発生=尾翼が聞かなくなる(ピッチコントロールが効かなくなる=一般に機首が下がる=墜落する)ですね。
アンダーソン氏ってのは、もしかして、今はやりの「ある会だ」関係者かも
と思った次第です。
横レス&蒸し返し失礼
224>223:01/10/14 01:20 ID:???
もうあげるな
殺すぞ?
225ご冗談でしょう?名無しさん:01/10/14 01:47 ID:???
>>223
ageるなら、まともな事書けよ、全く。
物理というより、もうちょっと航空力学の基礎を
かじってから書きなさいって。
(このスレに続けないで、航空・船舶板へ逝きたまへ)
226ご冗談でしょう?名無しさん:01/10/14 04:16 ID:???
あげるのは、自由とちゃいまっか?
倉庫入りしなけりゃ、書き逃げ自由はここのルールやろ!
227ご冗談でしょう?名無しさん:01/10/14 05:33 ID:???
>226
まぁね。同様に「あげるな殺すぞ」と書くのも自由。
しかし私の見たところ、このスレをできればageてほしくないと
思ってる住人は多そうだ。当然私の主観だから、判断は御自由に。
228ご冗談でしょう?名無しさん:01/10/14 06:07 ID:???
ageるにsageる方法=レスしない。
でも、別にこのスレそんなにマジ過ぎたりしてるわけじゃないし、
いいんじゃないの?ageられても。
誰かにとって都合の悪いことでもかかれているのかな?
229ご冗談でしょう?名無しさん:01/10/14 14:26 ID:S+OY6tTE
ageると主に怒られそうだから、書き逃げだけど、
このスレで出てきた揚力の大体3つ位の性質
1.作用・反作用
2.幾何学的束縛力(斜面の原理)
3.圧力差(不安定な要素を持ち、何時でも、そして何時までも働くとは限らない)
ってのは、どれも揚力の本質を衝いている気がする。そしてこいつらをすべて
認めなければ揚力を理解したことにはならないって感じ。
1.は短い時間のことを説明しやすい。
2.はエネルギーや仕事のことを説明するのに適当
3.は揚力が1.2.だけでは一筋縄で説明出来るものじゃないってことを
教えてくれる。
三位一体の原理ってところかな?
230ご冗談でしょう?名無しさん:01/10/14 16:32 ID:???
3種の神器
231 :01/10/18 11:49 ID:lYlNkIgH
age
232ご冗談でしょう?名無しさん:01/10/22 18:49 ID:vBb53w6S
age
233ご冗談でしょう?名無しさん:01/11/04 16:51 ID:2aQiwR6O
「酸」と関係ないのになぜ「酸素」? http://nara.cool.ne.jp/mituto
234 :01/11/28 16:54 ID:b6BU3RAQ
age
235\\\:01/11/29 18:18 ID:0ia9cedD
「飛行機って何で飛べるんですか」というスレなのに
揚力の発生に関することしか書かれてないな。

どうせナウシカのガンシップが理論的に飛ばないことなど
分からない連中ばかりだろ。
236ご冗談でしょう?名無しさん:01/11/29 20:12 ID:???
飛べるから飛行機って言うだけのことじゃないの?
237ご冗談でしょう?名無しさん:01/12/03 01:00 ID:???
>>235
ココは物理板。ガンシップの話は航空板でどーぞ。
個人的にはメーベが欲しいと思ったことがあるけど、
良く考えてみるとコワくて操縦できないな。

このスレの命題を「リンゴは何故落ちるのか?」に置き換えてみると、
 「地球が引っ張る」派
 「万有引力は質量の積に比例して距離の2乗に反比例」派
 「重力の本質は、超紐理論による発散の無い重力場の量子化で…」派
の議論だったのでしょうか。
前の2つはわかり易くてちゃんと計算もできるけど現象論的で、
「何故」に対する答えにはなっていない。最後のは難しくて
理解できない。
238   :01/12/26 22:50 ID:UXJV38FM
あげ
239  :02/01/24 03:12 ID:q6ZXinx4
あげ
240ご冗談でしょう?名無しさん:02/01/31 16:34 ID:UEnL6efF
7次元空間の巨人が手で持って飛ばしている
241ご冗談でしょう?名無しさん:02/01/31 18:54 ID:???
ガンシップが飛ばない?
ロッキードのFBW屋に手伝ってもらえばよかったのに
242ご冗談でしょう?名無しさん:02/01/31 22:44 ID:ZUT9aqc4
ファンは空気を動かすように作るが、プロペラは、空気をなるべく動かさない
ように作る。このどちらの原理で現在の飛行機の翼が設計されているかって
ことを、長々と論じていたのがこのスレ。
でも、ファンとプロペラなんて厳格には区別出来ないでしょ。
243ご冗談でしょう?名無しさん:02/02/01 03:06 ID:RhzWqJms
揚力    以上
2441年生:02/02/01 22:13 ID:uEf/GxY4
それまで通り普通に講義出てたら、わけわからなくなって落とした。
なんか今までの講義とは違うって感じ。やっぱ演習いっぱいしないとダメなんかな。
>>28
T、U、Vと基礎から順に積み上げる感じじゃないんですか?
Tが分からなくてもUがとれるの?
>>31
大学によるでしょ。
必修かどうか、留年があるかどうかとか。
2451年生:02/02/01 22:14 ID:uEf/GxY4
すまん、誤爆
246ご冗談でしょう?名無しさん:02/02/02 00:08 ID:+z6Z6MAf
>>243
運動学的揚力は重力が無くても働く。空気といっしょに落下運動している
飛行機は(妙なシチュエーションだが)、大して上昇速度を得ない。
揚力だけで浮いていたら、G(重力加速度)分の落下速度差がついていく
筈でしょ。
空気が地面に「固定」されていて、圧縮された状態になっている
場合、運動学的揚力以外の力が発生している証拠。
それが圧力差。
圧力差+運動学的揚力=揚力だが、揚力、以上じゃ答えにならん。
247ご冗談でしょう?名無しさん:02/02/10 18:42 ID:jzA11g/H
最近トイレに良くある、ジェット式手乾燥機、あれに手をかざすと良くわかるが
翼の下面に当たる風圧も相当のもの。
手を翼のように曲げても、あまり力を感じない。
負圧なんて発生しているように思えない。
同様に、下降気流なんてまったく発生していない。
さぁーどう説明する?
やっぱり下面の風圧が大事なんじゃない?
248sage:02/02/10 18:59 ID:tPG3QVnA
船が水に浮くのと大して変わらん。
249ご冗談でしょう?名無しさん:02/02/12 13:29 ID:???
>>246
圧力差の原因が「空気バネ」か...デムパ?

>>247
これは手を翼のように曲げただけで、手を翼にしたわけじゃないだろ。
翼の下面に流れが当たって揚力を得ても別に問題無いよ。抵抗が大きいだけ。
抵抗最小、揚力最大にするのが翼のあの形状でしょ。

>>248
そりゃ、飛行機じゃなくて飛行船。でも、空気の連続体近似が「真実」だって
信じて疑わない人(工学系に多い?)は>>248みたいに思い込んでる場合があるね。
250ご冗談でしょう?名無しさん:02/02/13 02:17 ID:P2hvpZcQ
>>246
運動学的揚力の意味がいまいち曖昧だが、気持ちはわかる。

>>247
手をジェット流に当てただけでは、翼のモデルにはならないということは>>249
の言う通り。
手を翼のような形状に曲げても揚力を得ない理由は正に、翼のミニチュアモデル
になっていないから。手の後ろの空気は動かない。

>>249
そりゃ逆だって。工学系こそ離散的に理解してる奴が多いと思うよ。
理学系は連続系で考えて、離散性はフーリエ成分に分解した領域で考えてる。
要するに使い分ける。
飛行機の揚力を理解するのには、別に空気を連続体と見なしてもそう問題は
無い筈。

251ご冗談でしょう?名無しさん:02/02/14 00:37 ID:???
>>250
離散的な理解ってなに? 連続体近似の意味を取り違えているのかな。
連続体近似では揚力が生じている事実は簡単に示せても、なぜ生じるのかを
ちゃんと示せないってのが、このスレのサブジェクトの一つだったと思う。
工学的な応用ならモーマンタイだけど、理学的な「理解」ではないでしょ。
252ご冗談でしょう?名無しさん:02/02/16 05:33 ID:???
>>251
対象を要素の集合体に還元して、単純化しそれを合成して結果を出す
考え方(還元法)のことが離散的な意味での理解なんでしょう。
多分ここではね。その方法が強力であるし、工学の基本原理中の基本原理である
ことは言うまでも無いけど、工学の場合、それが目的になっちゃうんだよね。
理学は、還元法はあくまでも手段に過ぎない。
還元法なんて所詮は方法論の一つでしか無いことを認め、むしろ離散的な性質か
ら導き出せる結論を完全に暴き出して実際と照らし合わせて、理論(還元法)から
は出てこない部分を暴き出そうというのが理学の本来の目的なんでしょうね。
もっともそこらへん勘違いしている奴も多いようだが最近は。
253ご冗談でしょう?名無しさん:02/02/19 22:47 ID:fVr3Sw5p
ageage
254ご冗談でしょう?名無しさん:02/02/19 23:55 ID:???
遅刻しそうなときに使うんだよ
255ご冗談でしょう?名無しさん:02/03/09 02:06 ID:/yAbvEFn
???
256ご冗談でしょう?名無しさん:02/03/28 03:59 ID:YqSeLP1z
>>255
?????????????????
257ご冗談でしょう?名無しさん:02/03/28 13:19 ID:DgF55vGi
「ライトフライヤー号の謎」発売age
(遅すぎ?)
258ご冗談でしょう?名無しさん:02/04/07 07:52 ID:WwcvI4rE
ベルヌーイの法則で説明された揚力って間違っているのですか?
ベルヌーイの法則で説明された揚力では、飛行機は浮くことは
できても、飛び上がる(上昇速度が+になる)ことは出来ない
のですか?
259ご冗談でしょう?名無しさん:02/04/07 17:18 ID:???
パート3か
飛行機ってみんなの憧れなんだな
とりあえず「狂っちゃいないぜ」を見て見よう
260ご冗談でしょう?名無しさん:02/04/07 23:05 ID:???
>>258
ベルヌーイの定理だけじゃ揚力の説明はできないだろ。
261ご冗談でしょう?名無しさん:02/04/08 00:49 ID:X4zn+PjQ
>>260
んじゃひとつ講義お願いし増田
262ご冗談でしょう?名無しさん:02/04/08 00:50 ID:???
飛行石が入っているからです。
ガイシュツでした?
263ご冗談でしょう?名無しさん:02/04/08 02:14 ID:???
飛行石???
264ご冗談でしょう?名無しさん:02/04/08 02:16 ID:???
知らないのぉ?
かっこわるーい!!
265テロリスト:02/04/08 06:55 ID:???
バルス!
266ご冗談でしょう?名無しさん:02/04/08 12:39 ID:znUWC8yD
age
267ご冗談でしょう?名無しさん:02/04/09 02:21 ID:???
>>261
揚力の大きさは計算できますけど説明はできません。それが何か?(某航空工学屋)
268ご冗談でしょう?名無しさん:02/04/09 02:54 ID:z0IgFthE
>>267
大きさの計算方法を説明して頂けませんか?
269ご冗談でしょう?名無しさん:02/04/09 03:03 ID:???
計算ソフトにぶちこむ
270ご冗談でしょう?名無しさん:02/04/11 00:54 ID:???
>>268
空気を連続体で近似すると微小部分の相互作用からナビエ‐ストークス方程式が
導けるので、それを解く。解析的に計算するには複雑すぎるのでいろいろ言い訳
つけてバッサリ近似する。数値的に解くときは空間、時間のメッシュのサイズに
気をつけて差分化すれば何とか解ける。でも揚力が生じる理由はよくわからん。
271ご冗談でしょう?名無しさん:02/04/11 01:33 ID:???
>>270
ありがとうございました。で、このスレの中心テーマであるところの
ベルヌーイの法則を使った説明である時によく引き合いに出される
翼、上下の速度差(同時到着原理)は出てくるのでしょうか。
出てきた場合、ベルヌーイの法則をそこに適用した場合、揚力の計算結果
と一致するのでしょうか。
272ご冗談でしょう?名無しさん:02/04/11 01:56 ID:???
>>271
同時到着はウソだけど、速度差は出るよ。圧縮性の粘性流体の場合には
ベルヌーイの定理みたいな近似は適用しないから、計算結果と一致しな
いんじゃないかな。比較したことないけど。
流体力学の何とかの法則って、ある条件を満たす場合の関係に言及した
ものがほとんど。だから気をつけて適用しないとすぐに成立条件を満た
さなくなってしまう。
273ご冗談でしょう?名無しさん:02/04/11 02:15 ID:E7O/yGWo
>>272
ありがとうございます。では、あなたの経験的なカンで敢えてお答えして
頂きたいのですが、揚力が発生する状況とはどのようなコンフィグレーション
なのでしょうか。逆にいえば揚力が発生しない状況(失速)との間には大局的に
見て流れの場のコンフィグレーションはどのような相違があると思えますか?
両者に一般に有意な差を見出すことが出来るでしょうか?数学的には困難でも
物理は直感でその困難さを乗り越えてきたと聞いております。

274天帝ユタ様 ◆UEhy5aKQ :02/04/11 02:29 ID:AECJXurg
揚力というのは俺的に言うとだな、翼の上下の曲線率は違っていて、曲線率が高いほど空気の流れが遅くなるわけなんだ。
つまり上下の空気圧が違ってくるわけで、/\まーこんな感じで風が→に吹くとしようや。
するとやな、\の部分では空気圧が下がってしまうんや。
翼の下はというとやな、_こんな感じで曲線率がないから、翼の上下では気圧差が生まれ、気圧のないほうにわずかに浮上する。
275ご冗談でしょう?名無しさん:02/04/11 02:36 ID:???
>>273
ダウンウォッシュの量じゃないの。
ヘリの回転翼で考えてみると、迎角を大きく取りすぎる
と「失速」状態になるね。このときダウンウォッシュが
減っているのは明らか。
276ご冗談でしょう?名無しさん:02/04/11 04:04 ID:E7O/yGWo
横レススマソ
翼:鉄とするね。
滑走路、電磁石。翼強い力で引っ張る。翼上昇出来ない。
翼、高さhのところ平行移動しか出来ないとするね。
この時でも、翼、速度つけば揚力「感じる」よね。(でも揚がらない)
この時、ダウソウォッシュって起こっているんですかにゃ?
277ご冗談でしょう?名無しさん:02/04/11 10:04 ID:???
起こっている
278何でこんなに下がるの?:02/04/16 00:34 ID:???
ダウソウォシュアゲ
279ご冗談でしょう?名無しさん:02/04/21 20:48 ID:WPGhPtrY
ビンラディンが捕まらないから?
280ご冗談でしょう?名無しさん:02/04/22 05:18 ID:???
そういや、ずいぶん昔の科学雑誌に
「昆虫は何故飛べるか」という記事があった。
結論は「根性」だったよ。
281ご冗談でしょう?名無しさん:02/04/22 21:57 ID:???
ユタ様チュウガクセイレヴェルダナ(w
282ご冗談でしょう?名無しさん:02/05/07 02:51 ID:???
ふーっ
過去ログ読み終わっての感想
揚力がstaticなのが主体かDynamicなのが主体かでこんなにもめるか?普通
staticな奴(圧力差ね)が主体じゃなけりゃ絶対飛べない。
Dynamicな奴(迎え角とって、速度殺して舞い上がる奴ね)補助的に使わなけ
れば不安定でまともに離陸できないし、着陸の時は神業以上のものが要求さ
れる。相当早く高く飛んでも、Staticな揚力は不安定(速度に鋭敏に依存。
時間的な変動かなりあり)
原因は、渦。しかしStatic揚力は強力で重量の大部分を支える。Dynamic揚力は
それを補うだけじゃなく、時間的な変動が少ないし、速度にそれほど鋭敏に依存
しないから、安定飛行には欠かせない。
離陸はともかく、降りるとき、迎え角使わないと、まともに速度を殺せず、
着陸地点を滑走路の縦横100m程度の範囲に絞るなんて神業になる。
283ご冗談でしょう?名無しさん:02/05/09 11:45 ID:???
>>282
もう一回読み直したら?
そんなことは大した問題にはなっていない。
要は圧力差で揚力を理解して良いのかって事だよ。

翼が空気から受ける力は空気からの圧力のみ。だから翼に上向きの力、揚力が
働くときには圧力差があるのは、ある意味当たり前のことで単に言い方を変え
たにすぎない。だが、揚力の場合と違って浮力の場合には圧力差がその本質的
な原因となるので、同じ上向きの力のためか混同して納得する場合が多い。

ここで問題になってくるのは、なぜ翼の周りで上向きの力=圧力差が生じる
のか、だが、流体力学的な方程式をいくらこねくり回したところで、圧力差が
生じている事実は示せても、その原因を明確に示すことはできない。

そこで、力学の基本法則に立ち返ってみれば、翼が空気に対して及ぼした作用
対する反作用がすなわち揚力になっていることは明らか。空気に下向の運動量
を与えた結果、翼は上向きの揚力を受ける。飛行機の翼から出るダウンウォッ
シュは想像し難いが、ヘリコプターのローターの下に入れば実感できる。

この考えがなかなか理解できないのは、翼の上面の空気に下向の運動量を与え
るところ。これは空気の粘性のためなのだが、流体力学の計算では粘性がある
と方程式が複雑になるので、しばしばこれを無視する。それで、実際に存在す
る空気の粘性も無視して良いものだと勘違いしてしまうから、わからなくなる。

「圧力差があるので揚力が生じます」では説明になってないから、もう止めようぜ。
284ご冗談でしょう?名無しさん:02/05/09 18:44 ID:???
>>283
物体に力を及ぼしたら、物体は必ず運動量を得るという考え方は正しいのですか?
私たちは、床に力を及ぼしている筈ですが、床は運動量を獲得してますか?
もちろん、私たちは、床から反作用として、私たちの体重に見合う力を受けている
ので、私たちは地中にめり込んでいくことがありません。
翼の上面の空気が下向きの運動量を獲得するのは、確かに粘性の作用と観測され
るでしょうが(粘性の定義→)、通常、流体を扱う場合、粘性を理論に組み込む
場合は、粘性として圧力勾配の無いところで測定されたものを採用します。
あなたの主張の後半は2つの文脈で使われる粘性を混同されているようです。
285ご冗談でしょう?名無しさん:02/05/09 21:35 ID:???
>>284
前半部分は「運動量の変化=力積」ってことだよ。
後半の「2つの文脈で使われる粘性」って何だ?
286ご冗談でしょう?名無しさん:02/05/10 02:56 ID:???
>>力学の基本法則に立ち返ってみれば翼が空気に対して及ぼした作用
>>対する反作用がすなわち揚力になっていることは明らか
つーか、この基本法則が正しいか否かを検証するのが「物理」の目的でしょ。
作用反作用の法則は、ミクロで成り立っても、マクロでも再現されるか?
ってのは一番面白い問題だと思うけど。
普通ミクロで成り立ってるからマクロでも成り立つと考えたいし、それを
検証しようとするのが物理理論。マクロで成り立つと頭から決め付けて
ミクロではそうでもないことを発見して、じゃ何故ミクロはマクロの法則を
認識してあたかも法則を知っているがごとく振舞うのかと思い悩むのは
オカルト系の理論
287ご冗談でしょう?名無しさん:02/05/10 08:38 ID:???
>>286
ニュートン力学を検証するのが「物理」か?

それにこの問題の場合、翼と空気との相互作用の詳細は関係ないよ。
空気の運動量が変化した事実があればそれで良い。だから、ミクロ
とかマクロなんて話にはならない。
「マクロ=ミクロの足し算か否か」ってのは別の問題。
288ご冗談でしょう?名無しさん:02/05/10 13:59 ID:???
>>287
流体力学的な方程式から何とか揚力発生の原因を突き止めてやろうという
考え方を敢えて「ミクロからマクロの法則を導く」とするならば、Newtonの
一般的な作用反作用の法則がどのようなスケールにおいても成り立つという
「公理」を頭から認めて考える方法を、(Newtonがこの法則を天体運動の解析
から確信したことに因んで)「マクロからミクロの法則を導く」と言うことが
できるでしょう。
大事なのは、矛盾が起こらないかどうかです。「ミクロ」で厳密に成り立って
いてもマクロではどうもうまくいかないということが起こるかも知れません。
というか、すべてのスケールで運動量保存の法則が成立するという公理を入れた
場合、これは現在の流体の方程式を扱う立場からは、もしかするとより強力な
制限を課することになるかも知れません。流体の運動はまだ未知な別の方程式
によっても同時に支配されるべきであるというような結果が出てくるかも知れ
ません。もしそうなると揚力の発生が流体の方程式から説明できるかも知れま
せん。勿論その時はその未知の方程式が物理基本法則として採用されるまでで、
運動量保存云々はエネルギー保存側と同様それから導き出される一般的法則と
しての価値を持つまでです。(ちょうどアインシュタインが光速度不変原理を
Newton力学に課して、Newton力学を書き直してそこから光速度不変原理を
導き出したのと似ています)
現実の世界の認識で、運動量保存則を一般的原理(確率論で言うなら大数の
法則に似ているか?)として無邪気に利用するのは....です。
ひとつ下のスケールでそれが厳密に成り立つことを仮定した上で現在のスケー
ルでも成り立っていることを認めてから使うのはOKだという気がします。
289ご冗談でしょう?名無しさん:02/05/11 07:53 ID:???
>>288
久々に「と」なレスを見たぞ(w
290ご冗談でしょう?名無しさん:02/05/11 12:09 ID:???
「と」は言いきれない。物理的に確信していても、数学的なモデルを立てて
追い詰めていくと矛盾が発生するかも知れない。
とゆーか、このスレでの流体は、高度な対称性を持った粒子の集まりで
複雑な相互作用をするものだからね(w
しかも重力で落下運動するが、地面で反射される(ww
運動量では普通押してけないよ。
291ご冗談でしょう?名無しさん:02/05/11 17:15 ID:???
今、翼が(地面に対して)静止しているとする。
翼の上の空気が何故自由落下しないのか?それは翼という障害物がありながらも、
翼の下の空気が、翼の上の空気の荷重を受け止めるべく圧力を発生しているから
ただし翼にも重量があるので、翼は落下する。
静止していない翼の上の空気は、翼が後部が下がっている為、自由落下を始める
翼の重量が下の空気の圧力を遮断するから。
この自由落下こそがダウンウォッシュの本質。この自由落下による翼上の空気が
獲得する運動量は、本来は翼が落下する時に獲得する運動量に等しいことは
ある程度正しい。ただ重力という地球がからむものも含んでいるからいきなり
作用反作用の法則を持ち出すのはやや短絡的。
292ご冗談でしょう?名無しさん:02/05/11 17:52 ID:???
>>290-291
おぉ、久々にKittyなレスを見たぞ(w
空気の自由落下がダウンウォッシュの本質だと(ww
気体の分子運動論くらい勉強してもバチは当たらないぜ(w3
妄想を垂れ流すのは止めよーな。
293ご冗談でしょう?名無しさん:02/05/11 17:58 ID:???
>>292
地球上にある空気の分子は自由落下しないのか?(W
294ご冗談でしょう?名無しさん:02/05/11 20:33 ID:???
空気の分子に重力は作用しているがそれがなにか?
Kittyちゃんの妄想をアップウォッシュに適用すると、
その本質は反重力か(w
295ご冗談でしょう?名無しさん:02/05/12 00:34 ID:???
飛行機の問題を考える限り、空気・翼・地球(重力)を考えれば十分だし地球は重力を提供は
するが重すぎて動かない(運動量変化が無い)と考えてもOK。となると運動量保存則は実質
空気と翼の関係だけになる。で、等速水平飛行してて、翼の運動量変化が無いとしてどうし
て空気に運動量変化が起こるのか?起こらないよね。Newton力学の基本原理からして。
あくまでも、これは翼の周りの空気の全体(地球全体の空気)の運動量変化が0なのであって
翼の周囲のある有限時間に起こる運動量変化が0であることは意味しない。
ただ粘性があると、むしろ粘性が強いと、運動量変化が局所的に起こらず、ずいぶん遠く広い範囲
まで引き摺られて起こってしまう。それも小さく分散した形で。
となると、翼周辺のダウンウォッシュ(下向きの運動量変化)やアップウォッシュ(ダウンウォッ
シュを相補する為に発生)の運動量収支だけをもってその力学的反作用が揚力の原因と断定するのは
説明不足なんじゃないかな?因果関係は認められるけどね(W
Newtonの作用反作用の法則を適用して...というより、この結果をもってNewtonの作用反作用の
法則の実証例とするほうが適切。
あくまでもダウンウォッシュの運動量にこだわるならば、むしろ粘性が無い時こそ、ダウンウォッ
シュの反作用=揚力としての説明がしやすいと思うのだが。
少なくとも、ダウンウォッシュの原因を粘性に求めるのはおかしい。
296ご冗談でしょう?名無しさん:02/05/12 01:09 ID:???
>>295
翼と空気だけの系で「運動量保存則」ねぇ。アフォですか?
風が吹き出している扇風機はどんどん後退してるのかよ(w

「作用反作用の法則の検証」なんて平気で書けるところも
「と」の臭いがプンプン。

>少なくとも、ダウンウォッシュの原因を粘性に求めるのはおかしい。

で、おなしなところは一つも書いてないぞ。
そんな気がするだけだろ(ww
297ご冗談でしょう?名無しさん:02/05/12 01:42 ID:???
>>296
後退しない扇風機の場合、大地と一体化しているとみなせます。
この場合、運動量保存則は一見すると成り立っていないように見えますが
どうなのでしょーか。通常は、空気が獲得した運動量と反対の向きの運動量を
大地が獲得すると考えるでしょう。(議論が必要ですが)

素粒子物理の世界では作用・反作用の法則を自明と決め付けて
はならないという話を聞いたことがあります。(ウェイク力だったっけな?
門外漢ですので、名前だけ聞きかじりで済みません)

おなしな?ところは確かにありませんが、運動量保存の法則を使って(作用反作用
の法則と実質的に同じです。)説明するという立場を取る限り、粘性は力の向きを
拡散させる性質がある以上、説得力を弱めます。翼とはずっと離れた位置では
上昇気流も発生し得るからです。局所的な運動量変化は粘性が強いと起こりにくく
なります。ダウンウォッシュの原因を別に求めて、粘性を敢えて無視したほうが
わかりやすい説明になると思います。
298ご冗談でしょう?名無しさん:02/05/12 03:20 ID:???
>>297
しょーがねぇなぁ。

扇風機は空気が獲得した運動量に相当する「力積」を受けるんだよ。
その力と扇風機の底の摩擦力が釣り合っているから動かない。
翼の場合は重力と釣り合っているから鉛直方向には動かない。
議論する余地は無いな。

で、粘性に関する記述を見る限り「よくわからないから無視したい」が
本音だろ。流体力学にヤラれちゃってるねぇ(w
空気を連続体近似した場合の粘性はなぜ生じるのかを考えてみるんだな。
299>>298:02/05/12 04:30 ID:???
扇風機は空気が獲得した運動量に相当する「力積」を受けるんだよ。
その通りです。しかし同時に大地から摩擦力という形で同時に力積を受けます。
両者は向きが反対で大きさが同じです。したがって受ける力積は0で、扇風機の
運動量変化はありません。ただし大地には扇風機から反作用として、扇風機に与えた
反対向きの力積を受けます。しかし大地の質量は極めて大きいので観測者の座標系に
対して静止しているとみなせます。この場合空気のみが扇風機から運動量を獲得して
いるように観測されます。
翼と空気、そして地球(=大地)の関係に戻ります。今、翼は大地に対して完全に
水平かつ等速であるとします。翼を原点に取った座標系は慣性系とみなせ、Newton
力学が適用できます。大地は重力という形で翼を引っ張りますが、翼は空気との相互
作用で重力と同じ大きさで向きが反対の力(揚力)を同時に受けており、単位時間に
獲得する運動量は0です。しかし大地は重力の反作用として加速度としては僅かですが
翼に向かう方向へ運動量を得ています。これは翼が水平に飛んでいるという仮定には厳
密には矛盾していますが、獲得している速度は僅かなので無視します。
ここで重大な仮定をしましょう。単位時間に翼が空気から得る力積は翼の上からであり
下からではないという仮定です。この前提が重要で、この仮定のもとでは確かに単位時
間に大地が獲得する運動量=−空気全体が単位時間に獲得する運動量という関係が成立
します。(DownWashの発生)
逆に、揚力は翼の下からであるという仮定の場合、空気が大地と一体化します。
つまり、大地と翼の間の空気の相互作用が発生し、空気が獲得する運動量は結
局0となります。(DownWash=0)
「DownWashは実際起こっているんだから、空気が翼の上から引っ張るんだ」という主
張は問題があります。翼と空気の間の相互作用の存在を暗黙の内に仮定しているから
です。近接したからといって相互作用は発生するとは限りません。相互作用が発生した
からといって、質点とみなすことが出来ない系の局所的運動を相互作用の帰結であると
することも出来ません。空気内部の運動であるかも知れないからです。例えば渦運動は
空気内部完結的な相互作用の結果としても起こり得ます。
300>>298:02/05/12 04:30 ID:???
>で、粘性に関する記述を見る限り「よくわからないから無視したい」が
>本音だろ。流体力学にヤラれちゃってるねぇ(w
>空気を連続体近似した場合の粘性はなぜ生じるのかを考えてみるんだな
運動量変化で揚力を説明したいという立場では、粘性の存在はむしろ邪魔になる
筈であると申し上げたつもりです。上で説明させて頂いたように、空気と翼の相互作用
が、翼の上で起こるのか、下で起こるのかは非常に重要で、両方同時に起こる場合は
運動量変化と揚力の関係がやっかいなものになり、説明力に不足します。空気の運動量
変化は、翼が本来重力によって獲得するものよりも、粘性がある場合は相当に小さく
なることが予想されます。これは粘性により、翼の周囲の空気が一斉に下に引っ張られ
大地と相互作用するからです。これはまた、空気を離散的な質点の集合体の運動と考え
た場合も同じです。離散的な質点の集合体の場合でも、粘性は定義でき、2質点を結ぶ
ベクトルと垂直な平面Dに成分を持つ力(質点間の距離・相対速度のD成分に関係する)
となります。
301ご冗談でしょう?名無しさん:02/05/12 06:24 ID:???
>>299-300
「翼の運動により単位時間に変化した空気の運動量=揚力」

この説明には空気の粘性は無関係だ。
どの部分の空気がダウンウォッシュになるのかを理解するときには粘性が大事。

ろくに勉強もしないで下らない屁理屈こね回しているのは「と」な連中と同じだぜ。
302ご冗談でしょう?名無しさん:02/05/12 12:26 ID:???
>どの部分の空気がダウンウォッシュになるのかを理解するときには粘性が大事。
うーん。粘性について理解不足なレス発見。
>>300が言っているとおり粘性は、局所的な運動状態を「瞬時に」全体に伝える
性質があり熱拡散とか濃度拡散と同じようにその伝わり方は相当早い。
大地付近まで粘性の影響が及ぶ。その部分のことを考慮するとダウンウォッシュ
の分布は粘性が大きいと大地と空気の間の相互作用が無視できなくなって、作用
反作用での説明には向かなくなる。むしろ粘性が無いほうが翼付近に集中し大地
との相互作用を無視しやすいので、作用・反作用で説明しようとするのだったら
そちらのほうが適している
>>300あたりで言っているようだがね
とっても勉強家で高尚な象牙を撫で回している非「と」系の人にはそう見えない
ようだ...
303ご冗談でしょう?名無しさん:02/05/12 13:14 ID:???
>>302
>粘性は、局所的な運動状態を「瞬時に」全体に伝える性質があり

どこかでデムパを受信してないかぁ(w

流体力学の講義では「粘性は周囲の流体の運動状態を均一化する性質」と
教えるんだがな。習ったことないのか? だから上空を飛行機が通過しても
粘性があるから「静止」している大気と均一化して、地上でダウンウォッシュ
を感じることはない。地上近くのヘリコプターからのダウンウォッシュを感じ
るのは、距離が短いので十分に均一化してないからだ。
つーことで「翼と大地の相互作用」とか書いているヤツはシロー「と」だな。

それにしても進歩してねぇーなぁ。勉強はキライか(w
304ご冗談でしょう?名無しさん:02/05/12 13:39 ID:???
>だから上空を飛行機が通過しても
>粘性があるから「静止」している大気と均一化して、地上でダウンウォッシュ
>を感じることはない。
そう。粘性があるから「静止」している大気は、ダウンウォッシュとやらと
調和しなくてはならず、その影響を受ける。その影響は次から次へと
より翼から離れた空気に影響が及び(音の伝播と異なり、有限速度伝播性
を必ずしも仮定しない)エネルギー保存則から、必ず遠くの影響は翼の近く
よりも小さくなる。これ当たり前。問題は大地にまで影響が到達した時
粘性を考えると、翼直下を含む相当量の空気にダウンウォッシュの影響が
拡散する。大地に空気は侵入出来ないから、結局空気は大地からも力を
受けることになる。空気の運動量変化が単純にダウンウォッシュの運動量と
結び付けられなくなる。
305ご冗談でしょう?名無しさん:02/05/12 14:51 ID:???
>>304
デムパだけじゃなくて、上空を飛ぶ飛行機のダウンウォッシュも感じちゃうのね。

揚力を得るために運動量を変化させた空気が、その後どのようになろうとも
翼に直接影響があるわけじゃない。戦闘機から発射されたミサイルが爆発して
も戦闘機の速度が変わるわけじゃないだろ。

では、間接的(とゆーか二次的)な影響はないのかというと、そーではない。
上空を飛行している場合には問題にならないけど、地表付近を飛行していると
「地表効果」で揚力が増す。ダウンウォッシュも地表で反射されてるだろう。
流体力学屋は「渦」とかで説明するけどな。

まぁ、>>304はこの地表効果のことを気にしていると、大目に解釈してやるから
そろそろ退散しな。
306ご冗談でしょう?名無しさん:02/05/12 16:49 ID:???
だ・か・ら、その後という概念が怪しいのよ。粘性があると、例え空気が独立
して運動できる質点の集まりだとしても、粘性という相互作用がある限り
音速よりもずっと早い速度で、影響が伝播し、大地と無関係ではいられなく
なるわけよ。二次的とは言えないと思うよ。翼と空気、大地が同時に力を及
ぼしあう。翼は空気という媒体を介して大地から力を受けるといっても良い
の。それだと、単純に空気が得た運動量=翼が得た力積(重力の為に相殺
されちゃうけどね)とは言えなくなって、セールスポイントの魅力が半減
しちゃうの。だから悪い事言わないから、空気の運動量で説明するんだっ
たら粘性を無視したモデルを使いなさいって。
地面効果(地表効果)は、別の現象。これは翼の下の空気の流れが地表の
影響でスムースに流れないことが関係してる。
307ご冗談でしょう?名無しさん:02/05/12 16:57 ID:???
なおどうしてダウンウォッシュを地面に居る人が感じないのかは、簡単。
一つは遠いことによって拡散しており、極めて小さいこと。更にはダウン
ウォッシュは一部音波として(密度の疎密波)としても伝わる。空気は
圧縮性だからね。更に複雑なのはダウンウォッシュは翼後方で渦に変わる
という点。つまり、空気はまっすぐに落ちず、ある高さで下の空気から
力を受けて水平方向の運動に変わる。その一部が音波(低周波)に変わって
大地に圧力という形で影響を及ぼす。
308ご冗談でしょう?名無しさん:02/05/12 17:50 ID:???
>>306-307
Kittyちゃんの妄想に付き合うのも厭きてきたなぁ。

「その後」のことは気にしなくて良いんだよ。ダウンウォッシュを地表で
感じないのはわかってるじゃない。(>>307の後半は妄想だけどな)
そーしたら空気は普通の大気として大気圧に寄与してるね。で、翼は
大気圧を受けるから、そーゆー意味では「その後」の影響はあるけど、
揚力に関する寄与は初めの運動量が変化したところでお終い。

もし「運動量の変化=力積」を認めないとゆーのなら、別板でやってくれ。
309ご冗談でしょう?名無しさん:02/05/12 18:20 ID:???
空気全体(恐らく、球の中心から球をくりぬいた形状をしてる)と地球、
そして飛行機この分解で見ないと運動量の議論は出来ないよ。その場合
その後とかその前とか関係なく、飛んでいる間ず〜っと、空気の運動量
は変化することになるね。翼で空気が切られて翼上に「穴」が出来ると
話は変わってくるけど。確かVan-Der-Waarsの未公開論文でそんなこと
が言及されてて、揚力の原因はその穴の部分に上から膨張して入ってく
る低圧な空気と、翼の下の部分の高圧な空気との差で揚力が表現できる
とか。確かに、その意味じゃ揚力の原因はVan-Der-Waaars力だな(W
粘性無視すると、翼上の空気の膨張を伴う自由落下で獲得する運動量
の増分で揚力説明できなくも無いし。Van-Der-Waarsで揚力説明する
立場なら粘性の影響は無視しなきゃね。(W
310ご冗談でしょう?名無しさん:02/05/12 21:45 ID:???
>>308
で最終的に揚力の原因は翼によってその上の空気を下に投げつける反動って
ことなの?>>307ではどうも他にも原因があることをほのめかしている
ような...
311ご冗談でしょう?名無しさん:02/05/12 23:28 ID:???
>>310
そう。運動量で考えることの利点は翼と空気分子の相互作用の詳細を知らなく
ても良いということ。どのように複雑な相互作用が働いているとしても、翼の
運動が原因で空気の運動量が変化したなら、それに相当する力を翼が受ける。
ニュートン力学の偉大なところだな。

それを理解できずに、下降気流が大気と相互作用することにより運動量を変化
させた分まで翼の影響だとする、変な妄想を持ったKittyが今日は暴れてるな。
因果関係が無茶苦茶。さらに定常状態とか瞬間の変化とかは想像すらできず、
ひたすら揚力と無関係な下降気流の行き先を心配しているんだな。地表効果
を考えるなら良いんだけどね。

で、それに便乗して>>309みたいなシッタカが登場する。曰く
 「Van-Der-Waarsで揚力説明する立場なら粘性の影響は無視しなきゃね」
だと。
粘性の原因もVan der Waals力が何なのかも知らないヤツが何言ってんだか(w

このスレが、物理をちゃんと理解してないヤツの妄想を披露する場と化して
しまうのは謎だな(w
312ご冗談でしょう?名無しさん:02/05/12 23:52 ID:???
>>310
それは>>311流の揚力の定義で、飛行機は揚力だけでは重力と釣り合って水平飛行
出来ないかも知れないとゆーのがネタじゃないの?
>>309
Van der Waalsの未公開論文ってどこでGetしたの?>>311にその論文見せて
Van der Waals力の知らざる側面(分子間力だけじゃないってことでしょ)
を教えてあげてよ。
313309:02/05/13 00:38 ID:???
>>312
未公開だから私も知りません。風の便りで聞いた話です。(W
>>310の隠しネタ:「空気と大気は違う!」
314309:02/05/13 00:40 ID:???
>>310の隠しネタ→>>311の隠しネタ
に訂正(鬱
>>310さん済みませんでした。
315ご冗談でしょう?名無しさん:02/05/13 01:35 ID:???
>>312
>それは>>311流の揚力の定義

つーか、物理屋にとってはごく当たり前の定義だと思うが。
何故か航空工学関係者は受け入れ難いみたいだけど。

>>313
何だ、未公開論文は知らないのかよ。ちょっと期待したんだけどな。
「風の便り」って、知ったかぶりのカミングアウトだな(w
Van der Waals力って何だかわかったか?
で、「空気と大気が違う!」と何か面白いことがあるのか。
316312:02/05/13 02:03 ID:???
>>翼の運動が原因で空気の運動量が変化したなら、それに相当する力を翼が受ける。
これ正しいようで飛躍あり。

翼の運動がきっかけでも、翼が実際に力を空気に与えなければ
その反作用を翼が受けようが無い。暗黙のうちに翼が空気に力を与えてること仮定
しちゃってる。(幾分かは知らないが、空気が翼から力を受けている可能性は否定
しないがね)

壁に手をそーっと当てて、それをゆっくりと離す。この時、手は
壁から力を受けるか?当てている時は、ごく僅かながらも壁に力を加えてしま
ってるかも知れないけどこれは可能な限り無視できる。離すときも、手を腕の
筋肉が動かすのであって、別に壁から力を与えてもらっているわけじゃない。

翼の上の空気の運動量変化はすべて翼から得たものであることを勝手に信じ
ている以上堂堂巡り。
317ご冗談でしょう?名無しさん:02/05/13 04:30 ID:???
>>316
因果関係の把握がイマイチだな。

>翼の上の空気の運動量変化はすべて翼から得たものであることを勝手に信じている

では、何が空気のダウンウォッシュを作るんだ? 地表からの音波か?(w
運動している翼の作用だろ。静止している翼には揚力は働かないよな。
であれば因果関係から
 「空気の運動量変化=翼が空気に与えた力積」
になるわけで、翼はその力積の反作用として揚力を得る。この場合、ダウン
ウォッシュがどのようにできたのかは問題ではないよ。翼(の運動)がダウン
ウォッシュの原因になっていればそれで良い。

壁と手の例はちょっと不適切、とゆーか意味不明。
団扇であおいで風をおこすことを考えてみると、気流が生じるのは
空気が団扇から力を受けたからだろ。
>暗黙のうちに翼が空気に力を与えてること仮定
が正しくないのなら、団扇からの力を否定するのと同じことだが、
そのためには他にどんな力が作用して風がおきるのかを示すことが
必要だよ。
318312:02/05/13 11:17 ID:???
>>316
オヒオヒ(W もし貴方が検察官だったら、間違いなく冤罪で多数の無実の人間を
13階段に送るタイプだろうな。因果関係と状況証拠を混同しておる。
街中で通りすがりの老人が突然、心臓発作で倒れて死んだ。それだけでタイーホ。
有罪・極刑。理由:人間、突然死はあり得ない。突然死の原因はその時、貴方が
側を通ったこと以外あり得ない。
これは極端として...別に翼を擁護しなければいけないような恩を感じてるわけじゃ
無いが...翼と空気がすれ違って空気の運動状態が変わった=>空気の運動量変化
は翼がすべて引き起こしたと錯覚する点は、下の例から見ても短絡的だぜ。

地面に突然ゴルフホールみたく、竪穴がぽっかり開いたとしよう。当然そこには
空気が入ってく。このことは例えば、線香の煙みたいなものを漂わせれば視覚的に
納得できる実験など簡単に行うことが出来る。(地中のツッカエ棒を外すとか)
この時、空気は自然に落下したのか、それとも地面が下がった際に地面が空気を
引っ張ったのか?実は、Newtonの万有引力の理論を信じようが信じまいが、いずれ
にせよ、「地(地面)が空気を引っ張った」共通の文で表現できる。(ニュアンス
は全然違うけどね)
では、ビルの屋上にコンクリート製の人口地面を構築して、その上で似た実験を
行ったとする。この場合もやはり同じ結果。しかしこの時は、自然落下(地球が
空気を引っ張る)と地面(=ビル屋上のコンクリート)が空気を引っ張る
という概念は明確に異なる。Newton力学を信じる限り、ビル屋上コンクリート
表面が空気を引っ張って穴の中に引きずり込んだとは普通考えない。空気が
地球に引っ張られて自然落下したと考えるのがNewton力学を学んだ人が考える
こと。Newtonの作用反作用の法則を認めて、どうして万有引力を認めない?
ま、自由だけどね。(W 万有引力を認めすぎちゃってるのかな?それとも。
319ご冗談でしょう?名無しさん:02/05/13 11:49 ID:???
>>318
最後の一文に更正の望みを託して、マジレス。

そのビルを逆さまに吊るして、人工地面を中に引っ込める(上に持ち上げる)
実験をしてみろよ。空気はビルの内部に入ってくるだろ。つまり上がる。
これは万有引力とは逆向きだぜ。さぁ、どーする?
320312:02/05/14 03:14 ID:???
せっかくのマジレスに亀で申し訳無いって感じだけど、さすがに実験は出来ないンで、
想像の域で答えるけど、確かに万有引力とは逆向きの方向に空気が上がる。で何か?
別に蓋と空気の引力は否定されないね。万有引力でもいいし(あくまでもNewton理論
によると極僅かで観測不能な位小さな量だろうけど)、大好きな翼と空気の間に働く
引力でも良いし。どちらか判定するのは不可能だろけど。また同時に下の空気が
基本的には圧力に起因する内力によって上昇していく(内力は重力と強い相関あり)
という説も古くからある有力な説。空気に重力が作用していても、地面でバウンド
されていたら地面からの抗力が存在している以上、内部運動が重力の方向と無関係
な方向になるのは別に不思議でもなんでもない。

ちなみに>>318(=312)での地面の穴への空気の進入も、別に穴底と空気との間の
引力の存否については何も言ってないことに注意。

要するに翼と空気の間に力がかかってなくても、空気の内部運動としての
ダウンウォッシュの存在はあり得る(否定はされてない)というのが>>318
での主張。穴との関連は、要するに大気を連続体というか、連結的なモデル
として捉える必然性は物理的には必ずしも無いということ。翼の上に空気の
「穴」(仮想的)が出来て、静的釣り合いが破れ内部運動(落下運動に近い)
が発生し「穴」を埋める。「穴」の直下の翼は上下の応力の釣り合いが破れ
揚力となる...とゆーのが一つの仮説。
対して察するに貴方の仮説は、翼と空気の間に、相対速度差に依存する相
互作用(引力)が何故か発生し、翼と空気が接しながらも何故か、空気に仕事
を与えダウンウォッシュとする。
まあ、いいや。貴方がこの仮説を信じるのは勝手だけど、一回自分で飛行機の
翼でも買って実験してみたらいいと思う。飛行機一機無駄にしちゃうかも知れない
けど(W でも時期的にビンラディン氏一派とかオウム一派と間違われないように
買うのは難しいから、ジュラルミン買って来て自分で工作だ。
飛行機の翼、地面にぴったり置いて、地面に、大気圧×翼表面積+翼重量が
かかるようにする(要するに翼下、真空にするね)。同時に圧力センサもしっかり
仕掛けとく。そこに大きな扇風機で風当てる。(時速170ノット以上ね)
重量が大気圧X翼表面積ー飛行機総重量+翼重量に変わったら貴方の説はかなり
有力。ハーバードでもプリンストンでもカーネギーメロンでも、UCLAでもNasaでも
ウォール街でも論文書いて送って発表するべき。
何よりも>>2http://ime.nu/www.allstar.fiu.edu/aero/airflylvl3.htmに
メール出してあげると喜ばれると思うよ。
321ご冗談でしょう?名無しさん:02/05/14 10:51 ID:???
>>320
まぁ、努力を認めて30点の出来だな。
で、気になる点に少しコメントだ。

>内部運動が重力の方向と無関係な方向になるのは別に不思議でもなんでもない。

そう。でもこれは分子の「熱運動」を考えるから不思議じゃなくなるわけで、
地面にバウンドとかは関係ない。

>静的釣り合いが破れ内部運動(落下運動に近い)が発生し「穴」を埋める

まだ「ダウンウォッシュ=重力」の発想から抜け出ていないから大幅原点。

>翼と空気の間に、相対速度差に依存する相互作用(引力)が何故か発生し

相互作用は別に相対速度に依存してるわけじゃない。まぁ、Van der Waals力
とかは量子力学を知らないと理解できないから、物理シロートには不思議なのか。

翼の上下に生じている圧力差は、あくまでもダウンウォッシュの結果であって
揚力の原因ではない、ってところが受け入れがたいのかな。
浮力の場合には圧力差が直接の原因なので勘違いしやすいな。
322ご冗談でしょう?名無しさん:02/05/14 11:59 ID:???
>>321
内部運動ってここじゃ、分子レベルの運動のことなの?(W
例えば等角速で渦巻いている風の平均運動量は0.しかし内部運動として暴風だって
ことは全然矛盾しない。
重心の運動を敢えて分子全体の重心の運動とするなら、内部運動ってのは
ある瞬間のある分子集団の重心の運動って感じかな?
重心の運動ってのはここじゃその位置に対象となるすべての分子の質量が
かかったものとして議論すること。
>相互作用は別に相対速度に依存してるわけじゃない
?
静止してる時は揚力が働かないから飛行機って滑走が必要なんでしょ。
翼って相対速度0の時でもVan der Waals力で浮き上がっちゃうわけ?
そーゆー翼だったらBoeing社に売り込め。喜んで買ってもらえるぞ。
323ご冗談でしょう?名無しさん:02/05/14 16:02 ID:???
>>322
前半の主旨は「ボクだって少しは理科を知ってます」ってことかな。
後半は「やっぱりvan der Waals力ってよくわかりません」か。
あと、力が作用していても釣り合っていれば加速度には寄与しないん
だが。
レベル低い...
324ご冗談でしょう?名無しさん:02/05/15 04:35 ID:???
>Van der Waals力とかは量子力学を知らないと理解できない
大問題発言ハッケソ
325ご冗談でしょう?名無しさん:02/05/15 21:06 ID:???
ダイポーノレダケダトオモテルヴァカハッケソ
326ご冗談でしょう?名無しさん:02/05/18 01:50 ID:IH8GEEn5
age
327ご冗談でしょう?名無しさん:02/05/18 20:10 ID:IH8GEEn5
粘性が高い空気は翼を上げないね〜。どうも
328ご冗談でしょう?名無しさん:02/05/18 20:45 ID:???
圧力差があるからダウンウォッシュが起こるのかダウンウォッシュがあるから圧力差ができるのか。
329ご冗談でしょう?名無しさん:02/05/18 23:40 ID:???
そんなことは余り問題になってないみたいだよ。卵と鶏の関係でしょ
結局。私はどちらでもないと思ってるけど。つまり空気の慣性が圧力差
とダウンウォッシュを同時に作ると。
330ご冗談でしょう?名無しさん:02/05/19 00:50 ID:???
>>328
問題の核心だね。
正解は後者。もし圧力差が原因だったらアップウォッシュになってしまうよ。

>>329
鶏と卵な関係じゃないね。
「空気の慣性」でどーやって揚力を説明するんだ?
331ご冗談でしょう?名無しさん:02/05/19 01:21 ID:???
ダウンウォッシュがあるから圧力差が起こるんだというのが>>330の意見
とするとやはりアップウォッシュも圧力差を作ることになるよね、
それがどーして、圧力差発生は必ずアップウォッシュになるのか不思議で
しょうがない。
332ご冗談でしょう?名無しさん:02/05/19 04:07 ID:???
>>331
F1マシンのウィングとかはアップウォッシュで圧力差を作っているがそれが何か?
もっともこの圧力差は飛行機の翼と上下が逆でダウンフォースだがな。
ヨットのセイルは「レフト・ライト」ウォッシュだぞ。それも不思議か(w
333ご冗談でしょう?名無しさん:02/05/19 12:55 ID:???
>>332
圧力差が原因ならアップウォッシュになるんでしょ。
ダウンウォッシュは圧力差は作るけど
ダウンウォッシュは圧力差を作りそれが最終的にアップウォッシュに
なる。ってことは運動量なんて与えてないことになるな。
334ご冗談でしょう?名無しさん:02/05/19 18:20 ID:???
圧力差が原因でもトータルでダウンウォッシュになると思うが。
というかこれって表裏一体だし。
335ご冗談でしょう?名無しさん:02/05/19 19:36 ID:???
>>333
ダウンウォッシュがあるために翼上<翼下なる圧力差が生じるということだが。
もしダウンウォッシュを相殺するようなアップウォッシュが存在すれば、翼周り
の空気の上下動は無くなるから何も起こらず、圧力差も生じない。
想像力、氏んでませんか?(w

>>334
思うのは勝手だが、その考えは過去ログで全部否定されているぞ。
まぁ、それって低圧から高圧に向けて風が吹くってことなんだが、
ハリーポッターに頼めば可能になるかもしれないな(w
336ご冗談でしょう?名無しさん:02/05/19 20:20 ID:YVFS8aCT
どーでもいいけど、結局、翼型のモノは空気を引っ張ることもありその時は
空気に運動量与えその反動でそのモノは反対向きに力受けるんでしょ。どうして
そういう形のモノは気まぐれにそういうことするの?他の形じゃ駄目なの?
分子とかが空気引っ張るんでしょ。分子って自分が所属しているモノの形
判る訳?>>335さんあたり良くわかってそうだから教えて欲しいな。
337333:02/05/19 20:33 ID:???
>>335=332
だよね。
違うって。ダウンウォッシュ→圧力差→アップウォッシュという矛盾が
起こるのはどうしてと聞いてるの!。>>332で圧力差が原因だったら
アップウォッシュになるって逝ってたじゃないの!。どうして発生した
圧力差はアップウォッシュの原因にならないと断定するの?
338ご冗談でしょう?名無しさん:02/05/19 21:42 ID:???
翼端で低圧側への流れがあるのは事実だが、空気から見れば機体を持ち上げてるわけで
作用反作用で下向きに曲がらなければおかしいよ。
339ご冗談でしょう?名無しさん:02/05/19 22:28 ID:???
頭が混乱してきた。
とりあえず、何らかの原因で(たとえばダウンウオッシュで)圧力の差ができたとするよね。
すると低圧から高圧に向かって空気は流れていくから、空気は翼に上向きの力を加えた上に
自分自身も上向きの運動量をもらってしまうことに・・・
どこがおかしい?
340339:02/05/19 22:42 ID:???
連続カキコスマソ
たとえばの話だけど、翼の上側は低圧になって下側は大気圧のままだとするよね。
すると低圧部には全方向から空気が押し寄せてくるわけだけど、翼に邪魔される分だけ
下方向からの流れはさえぎられるから流れは下向きに曲げられる。
この場合だと圧力差が原因でアップウオッシュが発生するという事はないみたいだけど。
341ご冗談でしょう?名無しさん:02/05/19 23:43 ID:???
27氏はそもそも圧力差の揚力への寄与なんて微々たる物だと主張してたはずだが…

↓元スレ354より
>圧力差はあるに決まってんだろ。それが揚力にどれくらい寄与するんだ?

最近では揚力=ダウンウォッシュによる圧力差ってところまでトーンダウンしちゃったのか?
342ご冗談でしょう?名無しさん:02/05/19 23:51 ID:YVFS8aCT
age進行でやれ。こそこそするな。(W
343ご冗談でしょう?名無しさん:02/05/19 23:58 ID:???
このスレ上げると嫌がられなかったか?
久しぶりに過去ログに目を通してみるか。
344ご冗談でしょう?名無しさん:02/05/20 07:38 ID:tvcQgw5j
ボートから石を投げればボートは反対方向に押されるわけだが、その原因が作用反作用か
運動量保存かと揉めているようなもの。
345ご冗談でしょう?名無しさん:02/05/20 08:09 ID:WyZcIz8V
270 名前:参加するカモさん :02/05/20 07:43
西村って話し方変わったね
なんだか淡々としてクールでかっこよくなったね
でも本当の西村とはちょっと違うような気がするなあ
今は誰に憧れて誰の真似をしてるのかなあ
中学の頃は東京ラブストーリーの「完治」に憧れてたよねえ
放送日の翌日はガンガンに完治に成りきっちゃって、あのもどかしい喋り方を連発してたよねえ
今だから言うけど、そんな西村のことみんな「気持ち悪い」って言ってたよ

それから西村って中学に入って急に背が伸びたよね
でもどうして学生服は取り換えなかったの?
ズボンはくるぶしが丸見えだったし、学生服は腰までしかなかったじゃん
あんな恰好で完治に成りきられても、ちょっとねぇ、、って感じだったよ
今だから言うけど、そんな西村のことみんな「気持ち悪い」って言ってたよ

ついでに言っちゃうけど、西村って給食を飲み込む瞬間、どうしてあんなに目を見開くの?
癖なのかも知れないけど、なんか牛みたいでおかしかったよ
女子の間では給食の時間に「ひろゆきを見てはいけない」という黄金律があったんだよ
今だから言うけど、そんな西村のことみんな「気持ち悪い」って言ってたよ

346ご冗談でしょう?名無しさん:02/05/20 10:16 ID:???
>>336
翼のあの形状は気まぐれじゃないぞ。カナーリ工夫してある。翼と分子は力を及ぼし
合っているから、分子から見ればどの辺が翼の表面なのかは「認識」してるな。

>>340
翼の外側にもアップウォッシュは生じないか?
つーか、掃除機の先端を上に向けてスイッチを入れたら、ダウンウォッシュが
生じて浮かぶのか(w どこかでガイシュツだな。
347ご冗談でしょう?名無しさん:02/05/20 11:21 ID:Va/huPAr
>>346
336だけど。
翼の表面は認識してるんだよね。大事なのは結局全体的な形を認識してる
ってことでしょ。
全体的な形でダウンウォッシュになるかならないか決まってくるんでしょ。
じゃなきゃどんな形でもいいことになるよね。
348ご冗談でしょう?名無しさん:02/05/20 19:38 ID:???
>>346
掃除機はいくら先端を上に向けたって吸い込んだ空気は横向きに排出せざるをえないような
形状になってるから例えが違う。
349339:02/05/20 20:40 ID:???
>>346、348
俺が引っかかってるのは上側低圧、下側高圧ってなるとアップウオッシュになるという部分ね。
なんか>>346読んだら今度は低圧部ができてダウンウオッシュができてるのにも関わらず
揚力は発生しないようだし…ドウナッテルンダヨヽ(`Д´)ノウワァァン!!
ってか掃除機は内部でダウンウオッシュが自分自身にあたって結局力は0になってるような。

350ご冗談でしょう?名無しさん:02/05/20 21:27 ID:???
圧力差とダウンウォッシュのどっちが作用でどっちが反作用かって議論だろこれは。
351ご冗談でしょう?名無しさん:02/05/20 22:40 ID:tvcQgw5j
>>330
>圧力差が原因だったらアップウォッシュになってしまうよ。
って主張が正しいとすると、揚力が発生した上に空気も持ち上げられることになってしまう。
明らかにおかしい。
352ご冗談でしょう?名無しさん:02/05/21 00:51 ID:V1DaJqdj
掃除機じゃないけど。
L字型に曲げた中空のパイプをy軸で高速回転させることで揚力を得る
空中浮揚装置の実験を昔見た記憶がある。
何でも遠心力でパイプ内部の気圧が下がるのがミソだとか。
子供心に素直にローター回せと思ったけど、誰か知ってる人いる?
353ご冗談でしょう?名無しさん:02/05/21 01:56 ID:???
>>348
そう、ダウンウォッシュが大事とゆーことだな。

>>349
>掃除機は内部でダウンウオッシュが自分自身にあたって結局力は0になってるような

わかってきたね。
もし翼上面の低圧部に上空から空気が入ってきたとすると、その後は
翼に衝突してお終い。変だろ。

>>351
つまり翼上面に発生した低圧部が揚力の原因じゃないってことになるんだな。
掃除機で空飛べるの?

>>352
??? パイプ内部の気圧が下がっても上向きの力にはならないと思うが…
354ご冗談でしょう?名無しさん:02/05/21 03:36 ID:???
メモ:
掃除機のパイプに吸いこまれる空気の動き
最初は加速される。パイプの中に入ってからはほぼ等速運動
ほぼ定圧で進行。
パイプの壁には圧力以外与えずパイプに沿って進む。(定常状態では)
パイプの先端の向きとパイプの終端の向きが異なるとき
先端の空気を加速したのは何?
答。パイプの中の空気。それはきちんと反作用を受けている。
それでいて、最終的には流れの線は曲がってもまったくおかしくない。
不思議だね。
単純に空気に作用・反作用の法則を当てはめるとパラドックスと思って
しまう。
355ご冗談でしょう?名無しさん:02/05/21 08:48 ID:???
>>353
>つまり翼上面に発生した低圧部が揚力の原因じゃないってことになるんだな。

なんで圧力が空気だけ(相対的に)引っ張って物体には影響を及ぼさない?

掃除機の場合は内部でせき止めたときの圧力上昇があるから結局低圧部の引張と
釣合ってしまうわけだ。圧力を考えてもきっちり釣合ってるよ。
揚力の原因、というか実際に働いている力は圧力差によるものに他ならない。
356ご冗談でしょう?名無しさん:02/05/21 09:31 ID:???
>>355
>掃除機の場合は内部でせき止めたときの圧力上昇があるから

翼の場合、引っ張った空気はせき止められないのかな(w

過去ログ読んでる? >>283とか見てね。
ギロンしているのはその圧力差が生じる原因だよ。
357ご冗談でしょう?名無しさん:02/05/21 17:38 ID:???
>>356
>ギロンしているのはその圧力差が生じる原因だよ。
ってのには同意するよ。あくまで
>もし圧力差が原因だったらアップウォッシュになってしまうよ。
って部分の誤りを正したかっただけ。
358ご冗談でしょう?名無しさん:02/05/21 18:19 ID:???
>>357
大気圧に比べて翼上の圧力小、翼下の圧力大ならアップウォッシュだが。
翼上のみが低圧とゆー条件ならアップウォッシュになるとは限らないな。
いずれにしろ、最初に「圧力差ありき」では説明は破綻する。掃除機が
浮かぶなら話は別だが(w
359ご冗談でしょう?名無しさん:02/05/21 18:59 ID:???
>>358
実際の翼では大気圧に比べて翼上の圧力小、翼下の圧力大でもダウンウォッシュができてますよ。
事実と反することを言ってどうしますか?

360ご冗談でしょう?名無しさん:02/05/21 20:20 ID:???
>>359
だからぁ、その圧力差がダウンウォッシュをつくってるのかよ
って話だ。違うだろ。
アゲてるとヘンなのが乱入しちまう…
361ご冗談でしょう?名無しさん:02/05/21 20:52 ID:???
>>360
>だからぁ、その圧力差がダウンウォッシュをつくってるのかよ
>って話だ。違うだろ。

それはあなたの思い込み。作用反作用をご存知ですか。
空気が飛行機を持ち上げた分押し下げられてるに過ぎない訳。もちろん逆の見方もできる。


362ご冗談でしょう?名無しさん:02/05/21 22:09 ID:???
>>356
先生、翼は掃除機みたいに無理やりパイプで流れを導いてるわけではないから
全部の流れをせき止めることは無理だと思います。
363ご冗談でしょう?名無しさん:02/05/21 22:38 ID:???
もうひとつ質問。>>330の”圧力差が原因だったらアップウォッシュになってしまうよ”とい
う言葉が正しいとすると、逆に上面側を高圧にして下面側を低圧にするとダウンウォッシュが
発生して揚力が発生することになるんですけど。どこか矛盾してませんか。
364ご冗談でしょう?名無しさん:02/05/21 23:01 ID:???
>>361
掃除機のパラドクス(wがわかってないねぇ。どんな場合でも圧力差が
あればダウンウォッシュが生じるわけではないだろ。

ダウンウォッシュを生じさせた力積の反作用として翼は上向きの力を受ける。
これが揚力の本質。

で、この状態の流体力学的な方程式を解いてみると、翼の上下で空気の
圧力に差ができているわけで、それを見て
「これこそが揚力の源、圧力差マンセー! ダウンウォッシュ? 
 あるかもしれないけど、圧力差があるから揚力には関係ないよ」
になってしまうところがヘンだってことなんだが… わかる?
365ご冗談でしょう?名無しさん:02/05/21 23:11 ID:???
>>362
せき止められない部分がダウンウォッシュか?
掃除機の先端をすばやく水平に移動させれば浮かぶのか(w

>>363
ダウンウォッシュが翼から離れていかないと揚力にはならないが。
掃除機と同じだぜ。
366ご冗談でしょう?名無しさん:02/05/21 23:20 ID:???
>>364
掃除機の場合は全体にわたって圧力を積分すると0になってるからダウンウォッシュも
無いと思われ。
翼の場合は…圧力差がある限り積分したって0にならないわな。

> で、この状態の流体力学的な方程式を解いてみると、翼の上下で空気の
> 圧力に差ができているわけで、それを見て
> 「これこそが揚力の源、圧力差マンセー! ダウンウォッシュ? 
>  あるかもしれないけど、圧力差があるから揚力には関係ないよ」
> になってしまうところがヘンだってことなんだが… わかる?

というかいまさら誰もダウンウォッシュが揚力と無関係とは言ってないといってみるテスト。
何かずれてるみたいなので落ち着いてもう一度レスを読み直してみることをお勧めする。
367362&363:02/05/21 23:59 ID:???
>>365
掃除機の先をすばやく動かすなんてことするよりも掃除機の底辺りに穴を開けてやれば
いくらかの揚力は発生するでしょうね。やりすぎるとヘリコプターですが。
翼の周りは穴どころか全部フリーですね。

で、後半部分ですけど>>330>>358で圧力差でアップウォッシュが発生するとご自分で
おっしゃってますけど…
これは撤回ということでよろしいのでしょうか?

368ご冗談でしょう?名無しさん:02/05/22 00:19 ID:???
>>366
揚力の原因は? と聞いてみるテスト。

>>367
高圧部→低圧部に空気は流れないと言え、ってことか(w
369366:02/05/22 01:49 ID:???
揚力の原因…つまりなぜ圧力差orダウンウォッシュが発生するのかということだが
俺には良くある答え方しかできないんで…
370ご冗談でしょう?名無しさん:02/05/22 05:01 ID:???
上向きノズル掃除機のパラドクス。何故浮き上がらない?
答え簡単。吸いこんだ空気は掃除機じゃなく、地球によって引っ張られてチューブの中に入ってチューブの壁から力を受けて曲がって掃除機の中に入った。
その後は掃除機内部のファンによって加速されてフィルターを通って再び外に出る。普通はチューブの中に落ちることなど出来ない。既に入っている空気の抵抗を受けるから。
既に入っている空気は掃除機によって外に出された。だからそのすぐ上の空気はチューブの中に落っこちることが出来た。
だから掃除機(チューブも含む)は上の空気を直接加速したわけじゃないから別に反作用受けなくてもいい。
むしろチューブの中に空気が落ちるときチューブが若干下向きに力を受ける。
371ご冗談でしょう?名無しさん:02/05/22 10:30 ID:hYRwIc8Z
>>370
過去ログ読め。
ノズルを下向きにした場合とかのこと考えた?
372ご冗談でしょう?名無しさん:02/05/22 10:48 ID:???
>>369
翼上面<下面の圧力差になるのはダウンウォッシュが生じた結果だってことが
まだわかってないな。

ダウンウォッシュになるのは圧力差が生じた結果だと主張する圧力差マンセーな
一派は、低圧部から高圧部に向けて気流が生じることは当然だと言っているのと
同じだが、それが変なことであるとは思わないらしい(w

>>370
>吸いこんだ空気は掃除機じゃなく、地球によって引っ張られて

でたな、万有引力オトコ。床を掃除するときも地球が空気を引っ張るのかよ(w
373ご冗談でしょう?名無しさん:02/05/22 10:57 ID:???
>>372
じゃあ、床を掃除する時は地球は重力一時止めてくれるのか?
374ウッソ ◆mCVsUsSo :02/05/22 10:57 ID:???
物理的に絶対に飛ぶことが出来ない蜂がいるそうです。
どう考えても飛行出来るはずが無いのに、
その蜂は普通に飛んでいます。

これってどういうことなんだろう・・・
まだまだ科学には未知な部分が多いのかも。
375ご冗談でしょう?名無しさん:02/05/22 11:25 ID:???
>>373
あのね、空気がね、上がって行くときね、
「重力が下に引っ張るから上がる」
って言われても、困っちゃうなぁ(w

>>374
物理的に絶対に飛べない蜂は、飛べないが…
流体力学的に飛行が説明できない
とゆーことならあり得るがな。
376ウッソ ◆mCVsUsSo :02/05/22 11:32 ID:???
>>375
あ、そっちです。
スマソ。

羽の総面積?と蜂の重量の比率、羽ばたく回数?から考えると
絶対不可能だそうです。うろ覚え&素人でスマソ。
377ご冗談でしょう?名無しさん:02/05/22 11:53 ID:???
>>376
エスティメートするときに粘性をちゃんと考えていない、に100ベルヌーイ。
378ご冗談でしょう?名無しさん:02/05/22 16:24 ID:???
>>375
空気が上に上がると、下は真空になるんですかな?
379ご冗談でしょう?名無しさん:02/05/22 20:08 ID:???
>>372
質問。
翼上面の圧力<下面の圧力で翼にはどのような力が働くとお考えで?
380ご冗談でしょう?名無しさん:02/05/22 21:43 ID:???
>>379
上向きの力。

で、質問
そのような圧力差があると空気はどちらに流れるとお考えで?
381ご冗談でしょう?名無しさん:02/05/22 22:10 ID:???
>>380
翼に上向きの力を作用させるなら反作用で下向きの流れ。
382ご冗談でしょう?名無しさん:02/05/22 22:26 ID:???
>>381
あれぇ、圧力差にはメンションしないのかなぁ。
低圧→高圧に向かって気流が生じたら変だよなぁ。
そろそろ圧力差マンセーやめたら(w
少なくとも「ダウンウォッシュが生じている」と「圧力差が生じている」が
同等ではないって、わかってきたんじゃないか?
383ご冗談でしょう?名無しさん:02/05/22 22:51 ID:???
>>382
運動量保存がなにより優先するはずだけど。かたや
> 低圧→高圧に向かって気流が生じたら変だよなぁ。
という考え方は直感的にはそうだけれど確たる保証は無いもんなあ。
何処が間違ってるかといえばこの部分ではなかろうか。



384ご冗談でしょう?名無しさん:02/05/23 00:13 ID:???
>>383
運動量「保存」ってのがカナーリ引っかかるが、そこは大目に見ると
 「運動量が保存しているので圧力差が生じる」は正しいが
 「圧力差が生じているので運動量が保存する」は明らかに誤りだろ。
運動量保存のほうが全然偉いのに(w 圧力差マンセーしてるとそんなことも
わからなくなる。
で、圧力差マンセーからなかなか脱却できないでいるのは、浮力と混同している
からなんだな。浮力の原因は大気圧の圧力差だからな。

とゆーことで、もう一度>>283を読み返してこのラウンドは終了かな。
385ご冗談でしょう?名無しさん:02/05/23 00:54 ID:???
>>384
なんで分からないのかな。大気が翼に力をおよぼす以上その反作用で
空気も翼から逆向きの力を受ける、ということを否定するのは作用反作用の
法則を否定するということだよ。これを否定するというのであれば議論は成立しそ
うにないが。
386ご冗談でしょう?名無しさん:02/05/23 01:03 ID:69Jsqcli
>>368などの一連の、まず高圧部から低圧部への流れありき的な発言を見ると、むしろ圧力差マンセーなのは
どっちだろうという気にさせられました。
387ご冗談でしょう?名無しさん:02/05/23 02:37 ID:???
空気のように圧縮性がある場合は少し難しくなるので、水中の中翼にかかる揚力で考えます。
要するにこのスレで対立しているのは、揚力は重力乗数gに無関係なのか、それとも関係する
のかということです。ベルヌーイの定理を利用した説明や、水(空気)の慣性によって、翼上面に
翼と非接触面が出来て、その部分には応力(圧力)が作用せず、浮力が増大して揚力となるという
説明では、水のように非圧縮性で流速が十分早い場合、gにほぼ比例して揚力が増大する筈です。
一方、運動量(ダウンウォッシュ)の説明の場合、gが変わっても揚力は流速との相対関係で決ま
る筈ですから、揚力は増加せず、重量が相対的に増加し、墜落します。いったいどちらなんでしょうか。
388ご冗談でしょう?名無しさん:02/05/23 07:51 ID:???
>>385
あのなぁ、ウソを書くんじゃねぇーよ。
>>283を読んだのか? どこで作用反作用を否定しているんだ?
ダウンウォッシュを発生させた力積の反作用が揚力だぜ。
その力は具体的には圧力差として現れているってことだ。

>>386
低圧→高圧に気流が生じるのはどんな場合かな(w

>>387
また、こんなのが出てきちまうじゃねぇーか。
揚力と浮力がゴチャゴチャの典型だな。
誰かサルベージしてやれよ(w
389ご冗談でしょう?名無しさん:02/05/23 10:18 ID:???
>>387
揚力の式知ってる?gなんて入ってこんぞ。

>>388
圧力差が翼に力を及ぼした時の反作用の力積の方向はどっち向き?
390ご冗談でしょう?名無しさん:02/05/23 10:58 ID:???
>圧力差が翼に力を及ぼした時の反作用の力積の方向

先生!
なんだかえらそうに書いてますが、質問に誤りがあります。
知ったかぶりの恥晒しなので、放置してもいいですか?
391ご冗談でしょう?名無しさん:02/05/23 11:20 ID:???
>>390
ちなみにどこが誤ってるの?
392ご冗談でしょう?名無しさん:02/05/23 11:23 ID:???
揚力の式って?そんなのあるの?
393ご冗談でしょう?名無しさん:02/05/23 12:59 ID:???
物質Aの運動量変化とは、その物質Aに作用している力すべての力積に等しい。
一部の力が与えた力積は、Aの運動量変化と一致する必要は無い。
硬い地面の上にある石を上から押しても、石の運動量は変化せず、押したものは与えた
力に相当する反作用を石から受ける。石は地面から上から押した力と同じ力を抗力として
得ており、それが与える力積の影響で、石の運動量変化は0になっている。
394ご冗談でしょう?名無しさん:02/05/23 13:13 ID:???
>>391
一番いけないのは動圧と静圧の区別ができてないところ。

>>392
あるよ。クッタ‐ジューコフスキー。

>>393
ん?なんだ?
力の釣り合いのことを力説してるのか(w
395ご冗談でしょう?名無しさん:02/05/23 15:35 ID:???
>>394
この場合動圧と静圧の区別がどう関わってくるのでしょうか(w
396ご冗談でしょう?名無しさん:02/05/23 15:53 ID:69Jsqcli
というか翼にとっては静圧がすべてではなかろうか。
397ご冗談でしょう?名無しさん :02/05/23 17:04 ID:RbYOqt0u
渦で飛んでる
398名無し:02/05/23 19:57 ID:/Tp4zecA
>>392

たしか
W=1/2*ρ*V**2*S*Cl
で表せたはず

ρは流体の密度
Vは速度
Sは翼面積
Clは揚力係数

だったと思う。
399ご冗談でしょう?名無しさん:02/05/24 02:04 ID:OXS5PGt9
過去ログみた。ジューコフスキーの定理って奴、2次元の極端に理想化された
モデル。なにせ重力も働かず、無限の大きさを持ち、しかも地面も無い。
流体にも粘性も圧縮性も無い。それでも非常に苦労して導出されたみたい。
流体力学の珠玉みたいな定理。
だけど、飛行機の揚力まで巧く説明出来るかな?
でも、ここでの揚力は「飛行機が重力に打ち勝つ力」という意味でしょ。
ま、揚力として、無重力で粘性が十分低く、地面から十分遠い一様な流れ
で物体に働く力の特定の成分と定義すればそれまでだけど。
圧力はポテンシャルみたいなもので、勾配しか意味の無い量で、定数の差
は無意味だけど、空気を扱う場合で物体と相互作用する時は、地面の圧力
を大気圧と一致させるように正規化するけど、これは当然のように見えるけど
実は恣意的な行為。空気力学にこれを適用したら実験結果に合うのか合わないのか?
合ってないんじゃないか?と主張する人も居るみたいだね。データの公表が断片的
だから合ってると思いたい人も、今一つ自信を持って発表出来ないのかな?
この点がスレで盛んに圧力差による説明に反論する人の最終的な論拠じゃないかなって思ってたりする。
400ご冗談でしょう?名無しさん:02/05/24 02:27 ID:???
1.圧力差以外の謎の力で流れが曲げられているとする。
2.そしてこのときの力積の反作用が揚力となる。
3.つまりこの謎の力の大きさは揚力の大きさと等しい。
4.ところで圧力を翼全体にわたって積分したもの=揚力の大きさである。
5.よって圧力差がこの謎の力である。

どこが間違っているでしょうか?
401ご冗談でしょう?名無しさん:02/05/24 02:38 ID:OXS5PGt9
>>400
謎の力=圧力差になるように、圧力の地面での値や、翼周りの値を正規化
すればいい。つまり辻褄が合うように調整すること。
4は正確には圧力の不定部分を揚力の大きさ=翼全体での圧力の積分
にするように決めること。
だから5は正確には、謎の力の大きさ=空気からの応力
であり、圧力差がそれに一致するように、圧力の不定部分を調整出来る
という意味。圧力と応力を区別するだけの話。
別にこの話からじゃ矛盾はでないんじゃないの?
402ご冗談でしょう?名無しさん:02/05/24 07:12 ID:???
う〜ん、良いカンジみたいなんだが…

>>399
クッタ‐ジューコフスキーの定理は、流れの中に置かれた物体の周りに循環が
あれば流れと垂直な方向に力を受ける、といったモノ。マトモな物理屋なら、
その力は流れを曲げた(=運動量を変化させた)力積の反作用だとすぐにわかる。
世の中には循環の字ヅラに惑わされて、その力は「渦」で生じると説明する人
もいるようだがな(w

>>400
悪くないよ。1.の「圧力差以外の」を削れば良い。
翼とは違うけど凧の揚力を考えるとわかりやすいかな。
凧が気流を下に曲げた=運動量を「謎の力」で下向きに変化させたので、その
反作用が揚力になると単純に理解できる。で、この時の圧力は気流が衝突する
面の方が裏側より高くなっている。
つまり、「謎の力」の作用の結果「圧力差」が発生するとゆーことだ。
403ご冗談でしょう?名無しさん:02/05/24 10:14 ID:???
すると結局謎の力=圧力差で流れ曲げてんのか?
404ご冗談でしょう?名無しさん:02/05/24 11:12 ID:???
>>403
圧力差は流れを曲げないよ。
謎の力で曲げた反作用を凧は圧力の増分として感じるだけ。
405ご冗談でしょう?名無しさん:02/05/24 16:28 ID:???
大事なのはこの謎の力が重力に関係するかどうかだね。
406ご冗談でしょう?名無しさん:02/05/24 18:48 ID:???
>>405
また、そのネタ?
この謎の力の正体は、ほぼクーロン力だ。
407ご冗談でしょう?名無しさん:02/05/25 01:35 ID:???
>>405
じゃこうなるよね。翼を構成する十分小さな要素(分子と言ってもいいのかな?)
と同じ程度の大きさの空気の要素(これも分子と言えるかな?)同士は、点とみ
なせる位小さいのに、相互作用する力は平行で反対向きだけど、必ずしも両方
の点を取る直線とは平行でない力が働くことになる。(非中心力)
静止している空気中ではこの力は平均化されて感じられないけど、一斉に動いて
いる空気中では顕在化してくるということだよね。そすると、奇妙なんだよね
対称性の破れが発生してしまう。宇宙で特定の方向が絶対的な意味を持ってしまい
地球が自転してるとか、公転してるとか怪しくなってしまう。
408407:02/05/25 01:36 ID:???
>>405
失礼。>>406へのレスでした。
409ご冗談でしょう?名無しさん:02/05/25 01:37 ID:???
>1.謎の力で流れが曲げられているとする。
>(中略)
>5.よって圧力差がこの謎の力である。
ってのは既に>>402で認めたらしいぞ。
つまり謎の力が流れを曲げるってことは圧力差が流れを曲げるということですね。
410ご冗談でしょう?名無しさん:02/05/25 03:58 ID:???
>>407
空気の分子は点とみなせるが、翼のほうは点ではないな。
対象性の破れってなんだ?

>>408
圧力差マンセーの粘着Kittyだな。

5.の解釈は>>402
>「謎の力」の作用の結果「圧力差」が発生する
となっているが。あと>>404でも
> 圧力差は流れを曲げないよ。
> 謎の力で曲げた反作用を凧は圧力の増分として感じるだけ。
とあるが、理解不能か?
空気を連続体近似でしか認識できないマヌケが圧力差マンセーに陥り易い
のは相変わらずだな。

空気中に置かれた物体には空気からの圧力が作用するが、物体が無い場合、
空気自体には圧力が作用しているのか?

Yesと答えたキミにはマンセー警報だ。(w
411ご冗談でしょう?名無しさん:02/05/25 05:06 ID:???
>>410
翼の分子は面だってこと?まさか飛行機の翼が超高分子単結晶だってこと?
412ご冗談でしょう?名無しさん:02/05/25 05:37 ID:???
>>411
翼の方は原子がイパーイ並んでいるから、相互作用を考えるときには「点」と
考えることはできないだろ。
低レベルなツッコミは止めてね。って、高レベルなツッコミは皆無に等しい(w
413ご冗談でしょう?名無しさん:02/05/25 06:56 ID:???
>>412
そーゆー問題じゃなくて、何で空気の分子の翼に平行な一方向の動きを感知して下に
引っ張るんだってことよ。静止してるとき揚力働かないのは何故かって
ゆーと、結局、空気分子が翼に平行な今度は逆方向に動いている時は空気を反発させ
るので、静止してるときは、引っ張る分子と反発させる分子の数が単位時間に大体
同じなので、貴方のゆー揚力が働かなくなると解釈せざるを得ないよね。
流れの中で翼が揚力を得るのは、空気分子がある平行な方向の速度成分が統計的に
増えるからでしょ。引っ張る分子のほうが多くなって揚力が増す。
これはどう説明するつもりかな?結局翼は異方性単結晶で構成されてると言うの
だろうがね。
414ご冗談でしょう?名無しさん:02/05/25 09:57 ID:???
>>413
真性? 誰か質問の意味を解釈してくれ。

流れを下に曲げるのに適しているのが翼のあの形状だが。
415ご冗談でしょう?名無しさん:02/05/25 13:47 ID:KAHJm6hD
>>410
>>402でわざわざ「圧力差以外の」という言葉を削っているけど
謎の力と圧力差が別物なら削る必要もないと思った。
416ご冗談でしょう?名無しさん:02/05/25 15:25 ID:???
つまるところこういうことを主張しているのでは?

翼は謎の力で空気を引っ張る。その反作用として翼も空気から引っ張られる。
なお、この結果として流れはダウンウォッシュとなるし、圧力差も生まれる。
417ご冗談でしょう?名無しさん:02/05/25 17:28 ID:???
>>417
違うな。
翼は謎の力で流れをダウンウォッシュにする。その反作用として翼は
空気から上向きの力、揚力を受ける。この結果として圧力差が生まれる。

翼周りの圧力を測定すれば、揚力の大きさを知ることができるんだな。
418417:02/05/25 17:30 ID:???
>>416へのレスでした。
419ご冗談でしょう?名無しさん:02/05/25 17:57 ID:???
謎の力が分子間の力に由来するものだとしたら、何故自然界には、
ちょうど逆の反揚力(飛行機と全く同じ形をしながらも、進んでいると
見かけ上の重力が増す)が生ずるような物質が無いのであろうか?
あってもおかしくないし、むしろ何故殆どの物質に正揚力が生ずることを
>>414等は説明しなければならない。
420Nanashi_et_al.:02/05/25 19:26 ID:???
でもさっき落ちました
421ご冗談でしょう?名無しさん:02/05/25 23:27 ID:???
>>419
久々にみるなぁ、真性…
“反”分子間力ってのがあるのかよ(w
422ご冗談でしょう?名無しさん:02/05/26 00:48 ID:???
>>416>>417の違いよくわかりませーん。
結果としてダウンウォッシュになることと、空気が引っ張られてダウンウォッシュになることの違いってあるの?
423ご冗談でしょう?名無しさん:02/05/26 01:22 ID:???
>>422
違うのは圧力の方だろ。
注射器のポンプを引いたときに内部が低圧になるのとは違う
ってことだが、>>410の答えがyesならわからねぇだろーな(w
424ご冗談でしょう?名無しさん:02/05/26 01:36 ID:???
>>410の言明は少し変
正確に言えば、空気中に置かれた物体には空気から応力(良く圧力と呼ばれて
いるね)が作用し、通常は浮力という形になるが、大抵の物体は重いので
浮かばない。
空気自体にはその任意の部分に圧力場というものを定義することが出来てその
勾配によって加速する。圧力場の勾配があって初めて空気は運動できる。
無風の時、圧力場の地面における値を大気圧(無風の空気によって地面が受ける
応力)と定めておく。これは圧力場から風速の計算をするためには余り意味を持つ
ことではないが、物体が受ける応力をこの圧力場から計算しても実験値とそうずれ
ないことからそうされてるんでしょう。多分。
425ご冗談でしょう?名無しさん:02/05/26 01:44 ID:???
なんでそこまでダウンウォッシュにこだわるんだろうね。
翼が空気を引っ張ればその時点で反作用として揚力は発生する筈だが。

もしかして作用反作用を否定してるのか!?
426ご冗談でしょう?名無しさん:02/05/26 02:17 ID:???
>>425
引っ張る理由は?
427ご冗談でしょう?名無しさん:02/05/26 02:32 ID:???
>>426
>>417に聞いてくださいです…
428ご冗談でしょう?名無しさん:02/05/26 02:38 ID:???
よし>>417に聞いてみよう
その答えは
>>427に聞いてくれ(W
429ご冗談でしょう?名無しさん:02/05/26 03:50 ID:???
AがBを引っ張る理由は?
BがAを引っ張ったから。
じゃあBがAを引っ張る理由は?
AがBを引っ張ったから。
あれ?
430ご冗談でしょう?名無しさん:02/05/26 03:55 ID:???
>>424
圧力勾配があれば必ず風が吹くのかな?
地面での気圧>上空での気圧だが、常に上昇気流があるって主張か?

>>425
翼の揚力が重力と釣り合っている状態と、翼が吸盤で天井にぶら下っている
状態が同じだと思っているのかよ(w
431ご冗談でしょう?名無しさん:02/05/26 04:53 ID:???
>>430
空気の運動方程式に重力項が入ってたら、圧力勾配はあるが重力で相殺され
て風は吹かないな。
432ご冗談でしょう?名無しさん:02/05/26 08:54 ID:???
>>431
空気に対して重力を相殺するように作用する力は何かな?
433ご冗談でしょう?名無しさん:02/05/26 09:41 ID:???
>>432
空気内部(内力)として働く圧力勾配
434ご冗談でしょう?名無しさん:02/05/26 16:37 ID:???
>>433
ある領域の空気には領域外から圧力が作用しているのではないのかな?
その領域の表面で圧力を積分して求めた力と、領域内の空気に作用する
重力が釣り合うと考えないと、辻褄が合わなくなるが。
435ご冗談でしょう?名無しさん:02/05/26 17:03 ID:???
>>430
>>>425
>翼の揚力が重力と釣り合っている状態と、翼が吸盤で天井にぶら下っている
>状態が同じだと思っているのかよ(w

ってそういうこと言ってるわけじゃないんだけどな。
436ご冗談でしょう?名無しさん:02/05/26 18:55 ID:???
上面と下面の速度差に起因する圧力差で飛びます。
437ご冗談でしょう?名無しさん:02/05/26 19:13 ID:???
なんでその速度差が起こるかってことなんだが…
438ご冗談でしょう?名無しさん:02/05/26 22:36 ID:???
>>435
揚力と吸盤が違うのならダウンウォッシュに拘る必要があるだろ、木瓜。
オマエが変なことを書くから>>436みたいなコドモが出てきちまう(w

で、今は連続体近似の信者がどこまで頑張れるかを試してるんだがな。
早く「空気にも応力(=圧力)は作用します」って書いてくれよ(w
気違い「元スレ27」よ、もうやめれ。見苦しい。
440ご冗談でしょう?名無しさん:02/05/26 23:57 ID:???
儲、必死だな。苦しかったら見るなよ(w
連続体近似を捨ててみたらどーだ? ラクになるぞ。
441433:02/05/27 03:26 ID:???
>>434
ん?俺?>>432へのレスかと思ったよ。正にその通り。
>>432にきちんと教えてやって欲しいな。
442ご冗談でしょう?名無しさん:02/05/27 05:49 ID:???
>>438
理解できないようだから繰り返すけど、翼が空気を引張るならその反作用で
空気から引張られるのが作用・反作用の法則ってやつだよ。
443ご冗談でしょう?名無しさん:02/05/27 07:31 ID:g6ljSd/b
これ↓と、
>翼は謎の力で空気を引っ張る。その反作用として翼も空気から引っ張られる。
>なお、この結果として流れはダウンウォッシュとなるし、圧力差も生まれる。

これ↓
>翼は謎の力で流れをダウンウォッシュにする。その反作用として翼は
>空気から上向きの力、揚力を受ける。この結果として圧力差が生まれる。

って同じことだろ。
444ご冗談でしょう?名無しさん:02/05/27 09:21 ID:???
>>441
「空気に圧力が作用する」って、どーゆー状態なのか考えたことあるか?

空気中に物体を置くとその表面に圧力が作用するのはわかるよな。
空気分子は温度に応じてランダムな速度で運動しているから、物体の
表面にも衝突する。このときに空気分子の運動量を変化させた力積の
反作用が、物体にかかる圧力だ。

物体が空気から受けるのと同じ圧力が、実際に空気自身にも作用すると
思い込んでいるからダメなんだな。ちゃんと分子運動の平衡状態とかで
考えないといけない。

ちなみに>>434=432だよ。
で、連続体近似が悪いと主張するわけではないぞ。でも揚力の理解の
妨げになるってのは確かだろ。スレが伸びてるからな(w
445ご冗談でしょう?名無しさん:02/05/27 09:51 ID:???
>>443
先端を閉じた注射器に空気を入れてピストンを引くとどーなる。
圧力差はできるがダウンウォッシュは発生するか?

揚力のキモはダウンウォッシュの発生だろ、ってことなのに、
当たり前のことを繰り返してイイ気になってるマヌケもいるし…
446ご冗談でしょう?名無しさん:02/05/27 10:38 ID:wK9NFW9Y
下から空気が押している。(気体の分子運動論)
447ご冗談でしょう?名無しさん:02/05/27 11:19 ID:???
>物体が空気から受けるのと同じ圧力が、実際に空気自身にも作用すると
>思い込んでいるからダメなんだな。ちゃんと分子運動の平衡状態とかで
>考えないといけない。
もっと丁寧に書いてみそ。非常にクリティカルなこと書いてあるみたいな
んで。
448ご冗談でしょう?名無しさん:02/05/27 11:56 ID:???
>>447
空気には物体の表面に相当するモノが無く、領域の境界を設定するだけ
なのに、その表面に実際に力(圧力)が作用すると考えるのがダメ。
表面で内部と外部の空気分子が衝突していると考えるのは、もっとダメ(w

流体力学では空気を金属(液体金属か)のような連続体として近似し
仮想的な内部応力(圧力)を導入して、その運動を論じているが、
実際には空気の各部に力は作用していない。

にもかかわらず、圧力差マンセーな連中は現実に空気に圧力が作用している
と思い込んでるから、いつまで経っても揚力を理解することができない。

これでわかるか?
449ご冗談でしょう?名無しさん:02/05/27 12:41 ID:???
晒し上げ
450ご冗談でしょう?名無しさん:02/05/27 13:05 ID:???
逝き詰ったか、儲(w
451#:02/05/27 13:46 ID:???
 
452ご冗談でしょう?名無しさん:02/05/27 14:12 ID:???
>>448
空気を連続体近似するのは、それなりのスケールでやるから出来るんであって
確かにミクロスケールでは統計的な揺らぎが当然ある。それが
>境界で内部と外部の空気分子が衝突するのはもっとダメ(W
ということなんだろな。それは確かにその通り
しかし、マクロスケールで見れば、仮想的な内部応力として圧力という概念を
持ち出す(流体力学的な見方)のは別に悪いことじゃないし、普通そうする。

分子同士の衝突が応力発生の源であるというのは、また別の視点でしょ。
衝突ってのはこの場合、分子同士がある一定の距離を置いて反発力を及ぼし会う
ということも含んでいる。

仮に空気分子が互いに衝突しないので、連続体近似が出来ないという主張でも
分子にはそれこそ慣性があるのだから、翼の上をよぎる際に翼の上の圧力(応力
の総和)が減少することも説明できる。
この場合、分子が地面に鞠のように撥ねて運動してるってイメージでしょ。
分子は熱運動で凄いスピードだけど、基本的に自由落下運動をしてる。相互作用は
地面や翼との間だけ。
翼の後ろが下がってる分、翼に当たって撥ねかえる時の衝撃が緩む。
翼の下は通常どおり。だから、貴方が言う分子衝突衝撃力=圧力の上下の差が生
まれてくるんでしょうね。結局。

でもこれって分子運動論を持ち出さなければならない話か?
453ご冗談でしょう?名無しさん:02/05/27 15:01 ID:???
>>452
う〜ん、全体的には悪くない方向なんだが、誤りが散見してるなぁ。
もう少し分子運動論に対する理解が深まれば、持ち出したくなるよ。
特に気になるのは

>翼の後ろが下がってる分、翼に当たって撥ねかえる時の衝撃が緩む。

これは何? それと、翼の前が上がってる分は、どー解釈するのかな?
粘性が生じる理由を考えてみると良いよ。

流体力学で翼の揚力をちゃんと説明できれば、それで良いんだがな。
ナビエ‐ストークスを解けば求まりますってヤツはダメね(w
454ご冗談でしょう?名無しさん:02/05/28 04:34 ID:???
>>452の意見は過激。でも新鮮(分子論じゃなく、後半の翼が下がって衝撃云々の
所のみだけど)
>>453が良く判ってないみたいだけど、翼の前面部分上がってるけど、迎え角付いている
から実際それほど大きな角度付けて空気に当たってない。(機種・積載荷重
対気速度にもよるけど、迎え角は10°は付いてるのが普通)
それよりも、下がっている部分の角度のほうがずっときついし面積も大きい。
だから上がっているから全体では衝撃が緩和されると決め付けるのは即断的

>>452の分子ボール地面撥ね返り運動モデルは余りにも即物的過ぎて分子運動論の
奥深さを歪めているとしか思えないモデルだけど、>>453に説明して「あげる」
為には仕方が無いので敢えて使うとして...(擬人法さえ使う)
今、簡単の為に空気分子の地面に対する水平速度が0だとすると、垂直速度
だけで地面でバウンドを繰り返している。
ある空気分子Aが翼に当たって撥ね返されたとする。Aは翼に当たるまで翼の
存在を「知らなかった。」翼の後ろの方は、Aからは地面と区別できない。
しかし後ろの方が水平方向に対して下がっているのでAは翼が高速に移動する
際、「地面が一定の速度で下がっている」と感じるだろう。
一方、Aの衝突を撥ね返す翼の分子Bは、Aが静止しているときよりも遅く落ちてくる
と感じるだろう。落ちてきた分子を撥ね返す時、いつもと同じ力を与えては
Aに余計なエネルギーを与えてしまうことになる。(というよりも、Aの速度が
Bから離れるようになるまでの時間が静止している時よりも短くすぐ離れていって
しまうので、エネルギーを与えたくとも与えようが無い。)
勿論、この際にAがBに与える力積は静止してるときに比べて小さいことになる。

釘を指しておきたいがこの説明はあくまでも、プリミティブな説明で、空気力学の
奥の深さを知る為には流体力学の方法は欠かせない。分子間力とか考えるんだったら
なおさらのこと。分子論と流体力学は本来は敵対するものではないから。
その証拠に量子力学は統計力学の拡張だけど連続体モデルを使ってるね。(シュレジンガー方程式が良い例)
上の説明じゃ失速は説明しにくい。分子間相互作用による圧力の音速による伝播を
考えていないから。(相互作用に特別な仮定をおくと出てくるのが粘性効果)
非圧縮性ならば、一つの分子の影響が全部に瞬時に拡散する。失速等の興味深い
現象(飛行機乗りにはたまったものじゃないかも知れんが)は流体力学的考察抜きでは
語れないだろうね。
455ご冗談でしょう?名無しさん:02/05/28 04:39 ID:???
>だから上がっているから全体では衝撃が緩和されると決め付けるのは即断的
だから上がってるから全体では衝撃が緩和*されない*と決め付けるのは....

>翼の後ろの方は、Aからは地面と区別できない
翼はAからは....

>Aが静止しているときよりも遅く落ちてくる
翼が静止しているときよりも、Aが遅くおちてくる...

に訂正します。
456454:02/05/28 04:40 ID:???
454-456は454によるものです。
457ご冗談でしょう?名無しさん:02/05/28 09:36 ID:???
>>454
「ボクは物理ができましぇ〜ん」って宣言してるのかよ(w

>簡単の為に空気分子の地面に対する水平速度が0だとすると

水平方向に飛んでいる翼から見れば、空気分子は後ろに飛んでる罠。

>この際にAがBに与える力積は静止してるときに比べて小さいことになる

ならねぇーよ。鉛直方向の運動量変化は翼の水平速度に依存しない。
ベクトルを知らないようだから、もしかしてマジ厨?
458ご冗談でしょう?名無しさん:02/05/28 12:29 ID:???
>>457
スロープの上に鉛直方向からボール落としたら、鉛直にはバウンド
しないね。バウンドする前と後では斜面に対する垂直速度成分のみ
が変わるだけ。(簡単の為重力加速度無視)完全弾性衝突知ってる?
ボールの質量1鉛直落下速度をvで等速、斜面の水平に対する角度αとでも
置くと、斜面に垂直な速度成分vcosα、これが力積
実際には、斜面にかかる鉛直成分のvcos^2αが重要(衝突角度β=90°-α)
だからvsin^2βでもOK
α=30°で0.75vつまりα=0の時の75%にしかならない。
スロープがボールの入射角がずっと浅くなるように動いているとこの値は
もっと小さくなる。少なくとも鉛直斜面静止の時の力積vよりはずっと
小さくなる。vに対して斜面の動く速度がずっと早いとボールの斜面から
見た衝突速度が速くなって必ずしもそうとは言いきれなくなるが。
しかし衝突角度βはこの場合極端に小さくなる。
459ご冗談でしょう?名無しさん:02/05/28 16:52 ID:???
>>458
おぉ、ちょっとマトモだ。

>鉛直成分のvcos^2α

オレは完全弾性衝突を知ってるから 2v cos^2α にするけどな(w

> スロープがボールの入射角がずっと浅くなるように動いているとこの値は
>もっと小さくなる。少なくとも鉛直斜面静止の時の力積vよりはずっと

これは、翼が動いていると入射角も浅くなるが、見かけの入射速度も上がる。
その辺をちゃんとエスティメートしてみたらどうかな。
460ご冗談でしょう?名無しさん:02/05/28 22:42 ID:???
>>457
> ならねぇーよ。鉛直方向の運動量変化は翼の水平速度に依存しない。

依存してるぞ。
461ご冗談でしょう?名無しさん:02/05/29 02:09 ID:???
>>459
そこまで言うんだったらYouがやるべきだな。恐らく>>458はもうとっくの
昔にそれを終わっているんだろうが、敢えてカキコしないのは、Youにそれを
敢えてしてもらいたいと思ってるからじゃないか?(間違ってたらスマソ
>>454が言ってるように、分子をボールのような比喩で近似するのは
例え判りやすく一見はっきりした結果を出すように見えても、流体の挙動の
一部しかシミュレート出来ない。現実の空気の性質から乖離(粘性という点では
無いよ)してその理解を歪曲するだけでなく、見逃すことの出来ない物理的本質
を捨ててしまう。出来うる限りそういう説明はしないほうが良いという考
え方で俺と意見が一致するんだが。
例えYouが>>458の結果をModifyしてそれを使うにしろ、それはここでは一応
>>454のアイデアが発端になってるんだから、>>454が明確に
注意している歪曲の危険性はきちっと断らないといけないと思うよ。
その点は>>458も同じだが。
462461:02/05/29 02:28 ID:???
良く読んでみたら発端は>>452みたいね。もっと良く読んでみたら、かなり
前から似たようなこと言ってる人居たみたいだけど
463ご冗談でしょう?名無しさん:02/05/29 09:38 ID:???
>>460
そりゃ、凧の揚力は風速に依存する罠。
それとも、流れに対して斜めに置いた板の揚力=翼の揚力ってことか?
>>454はそう主張してるがな。巡航中のダウンウォッシュは翼の下面で
発生してるかよ(w

>>461
分子運動論だと
>理解を歪曲するだけでなく、見逃すことの出来ない物理的本質を捨ててしまう
ってことか(w

翼の揚力とは違うが、>>458は平板の揚力の説明としては良いカンジだぞ。
464454ではないのだが:02/05/29 10:36 ID:???
>463
>流れに対して斜めに置いた板の揚力=翼の揚力ってことか?
>>454はそう主張してるがな。

まさか、違うって思っているんじゃないよね?
紙飛行機の翼の揚力が、実際の航空機の翼の揚力と
本質的に違うとでも?

Kittyな「元スレ27」らしくもない間違いだ。
465ご冗談でしょう?名無しさん:02/05/29 11:06 ID:???
>>464
文脈からは、
 流れに対して斜めに置いた板が(分子の弾性衝突で得た)揚力
ってことだろ。

その「元スレ27」に可愛がられた体験がトラウマになって、
アラ探しをしてないと心の平静が保てないのかよ(ww
466ご冗談でしょう?名無しさん:02/05/29 11:40 ID:???
>>465=459だろ?
そういう余計なツッコミはしなくていいから、>>458がやりかけた、「モルキュラボールバウンド」
理論をもっと緻密に書いてギャラリーを納得させなさいよ。もっともダウンウォッシュ発生云々は
それこそ>>458系の「単純化による見逃すことが出来ない物理的本質の損失」そのもので
多分出てこないだろうがね。「モルキュラボールバウンド」でダウンウォッシュまで出せたら
相当の通人として殿堂入りだけどね。誉めてあげるよ(W
で、そろそろキャップでも被れば?
467ご冗談でしょう?名無しさん:02/05/29 13:39 ID:???
>>466
「ギャラリー」って、納得したいのはオマエだろ(w
三角関数を知ってるなら、ちょっとやってみな。
添削してやるよ。

キャップってなんだ?
466じゃないけど、ギャラリーその1です。
このスレの最初から読みましたが、私も分子モデル(?)に
よる揚力発生の説明を伺いたいものです・・・できるものなら。
(まぁ、どうせまた難癖つけて逃げるんだろうけど・・・)
469ご冗談でしょう?名無しさん:02/05/30 00:54 ID:NBVQruaO
>>463
> そりゃ、凧の揚力は風速に依存する罠。
って飛行機の揚力だって風速に依存してるが。
470ご冗談でしょう?名無しさん:02/05/30 01:28 ID:???
ちょっと気になったんで。
>>445
飛行機と違って注射器内部の空気はピストンと同様に反対側の壁も引張ってるから
ダウンウォッシュは発生しませんね。

471ご冗談でしょう?名無しさん:02/05/30 01:40 ID:???
>>469
翼の揚力の主な原因が空気の弾性衝突によるものであるってことか?

斜辺を下にした三角定規みたいな断面の翼を迎角ゼロで動かすときに
揚力は発生するかしないか、を考えてみればわかるかな。
472ご冗談でしょう?名無しさん:02/05/30 01:49 ID:???
>>470
飛行機では何故ダウンウォッシュになるのかな?
473ご冗談でしょう?名無しさん:02/05/30 02:22 ID:NBVQruaO
>>471
> 翼の揚力の主な原因が空気の弾性衝突によるものであるってことか?
んなこといってねー。飛行機の揚力も風速に依存してる事実を指摘しただけ。
既出だと思ったけど。

> 斜辺を下にした三角定規みたいな断面の翼を迎角ゼロで動かすときに
> 揚力は発生するかしないか、を考えてみればわかるかな。
ちなみにその場合には揚力は発生するぞ。
474ご冗談でしょう?名無しさん:02/05/30 02:34 ID:???
>>472
それをスマートに説明できればこんなスレッドとっくに終了している、と言ってみるテスト
475ご冗談でしょう?名無しさん:02/05/30 03:02 ID:???
>>473
>ちなみにその場合には揚力は発生するぞ。

理由は?

>>474
「圧力差で引っ張る」と思わなくなったのは進歩だな(w
粘性って何で生じるのか考えたことはあるか?
476ご冗談でしょう?名無しさん:02/05/30 03:09 ID:???
>>467
キャップは文脈から察するに、黄色い帽子のことだと思われますが、
一応ここは2chなので、キャップ機能のことだとも考えられます。
fushianasanと名前の欄に記入してカキコされると名前欄にカキコ
したマシンに保存されているクッキーから生成された乱数固定文字
が表われ、以降同様の方法で同一の文字列をカキコすることが出来ます。
例として、"元スレ27◆kiTtyGuY"って感じで出てきます。見たことも
あると思います。ぜひやってみてください。貴方のレスを軸にこのスレ
は回っています。
477ご冗談でしょう?名無しさん:02/05/30 09:38 ID:???
>>475
理由はキャンバがついてるから。
>>477, >>475
というより、三角定規の例では ”有効”迎角が0になってないからと
言った方が良い鴨。三角定規で(有効)迎角0とは斜辺が水平の場合。
(三角定規の話しは本筋と関係無いからここまでにしようよ)
479ご冗談でしょう?名無しさん:02/05/30 15:29 ID:???
斜辺が水平の場合
>>477は揚力が有る
>>478は揚力が無い(=有効迎角0)
どちらが正しいの?
478だけど、スマソ、勘違いしてた。
>>471の設定が2番目に長い辺が水平(斜辺が水平に対し角度を
もっているだと思っていた。この場合は、もちろん揚力発生アリ。
>>471のオリジナルの設定だと、揚力発生はない。(むしろ、
若干の下向きの揚力が発生するかもしれない)。
キャンバの有り無しは揚力と直接は関係無いよ。あくまで流体
の流れの方向を翼によって変えるか変えないかです。

さて、私を含め、ギャラリーはこの辺で静かにしてましょう。
481ご冗談でしょう?名無しさん:02/05/30 22:22 ID:???
斜面が水平(翼の下面が水平)の場合
>>477は揚力が発生する (キャンバがついてるから)
>>480は揚力発生は無い (キャンバには関係無い)
どちらが正しいの?
482ご冗談でしょう?名無しさん:02/05/31 00:58 ID:???
そもそも三角定規みたいな剥離が起きまくりそうな形を例にするのはどうかと。
半円とかの方がふさわしいと思われ。
ちなみに半円の直線部を流れに水平にして置くと迎角は0になるけどキャンバがついてる
から揚力は発生する。
逆にキャンバのついてない対称翼だと迎角0なら揚力も0。
483ご冗談でしょう?名無しさん:02/05/31 07:35 ID:???
>>482
迎角0で揚力が発生するとき、ダウンウォッシュはできますか?
484ご冗談でしょう?名無しさん:02/05/31 10:23 ID:???
>>483
当然。
485ご冗談でしょう?名無しさん:02/05/31 11:06 ID:???
>>484
もっとキャンバの小さな円弧のような翼のほうが
考えやすいですが、後ろ側の下りに沿った流れが、
そのまま翼から離れたのがダウンウォッシュですか?

>>466-467あたりの「モルキュラボールバウンド」理論の続きをキボンヌ。
487ご冗談でしょう?名無しさん:02/05/31 13:18 ID:???
飛行機のこと言ってるんだから、重力無視はイカンね。
単純なモデル作るときでも、重力の存在はきちんと組み込んでやろうよ。
円弧の流れも同じ。重力きちんと考えているのか?
488ご冗談でしょう?名無しさん:02/06/01 15:25 ID:???
( ゚д゚)ポカーン
489ご冗談でしょう?名無しさん:02/06/01 21:37 ID:???
( ゚д゚)ポカーン (゚д゚)ポカーン
490大鳥居つばめさん ◆/stTH5oI :02/06/01 22:33 ID:???
( ゚д゚)ポカーン (゚д゚)ポカーン (゚д゚)ポカーン (゚д゚)ポカーン
491ご冗談でしょう?名無しさん:02/06/02 03:38 ID:???
( ゚д゚)ポカーン (゚д゚)ポカーン (゚д゚)ポカーン (゚д゚)ポカーン( ゚д゚)ポカーン (゚д゚)ポカーン (゚д゚)ポカーン (゚д゚)ポカーン



結局、元スレ27はまた逃げ出した訳か。まったく・・・。
493ご冗談でしょう?名無しさん:02/06/02 23:38 ID:???
誰でもいいですから>>485に答えてください。
494ご冗談でしょう?名無しさん:02/06/03 01:25 ID:DI2r4gS3
結局、元スレ27はまた逃げ出した訳か。まったく・・・。
495ご冗談でしょう?名無しさん:02/06/03 08:19 ID:???
>>485
そのまま離れることはないんじゃない?
496ご冗談でしょう?名無しさん:02/06/05 06:53 ID:9K75iGT0
さがっちゃったね。
497ご冗談でしょう?名無しさん:02/06/05 09:02 ID:???
マヌケなコドモが出てきたんで放置されてるんじゃねーの?(w
円弧翼の話は良いカンジなんだがな。
498ご冗談でしょう?名無しさん:02/06/05 09:56 ID:???
カンジって、何を感じるの?朝っぱらからさ。
それはさておき、このスレでは、飛行機の翼に発生する揚力(重力原因の
一方向への引力を相殺する力)の原因を説明するのに、空気の運動量、
とくに独立運動する離散モデル(分子モデル)を使って決定論的に説明
しつつも、結局運動量保存則という原理的法則から当然充たすべき結果
(言わば当たり前で何ら情報量を持たない)を言うだけという「説明」
に止めるのが良いのか、
それとも流体固有の予測不能性に充分に言及した上で原因
を分子の慣性に求める説明が分かりやすいかで意見が別れている
気がするけど。どちらが物理的な説明として潔いと思う?
499ご冗談でしょう?名無しさん:02/06/07 08:47 ID:9OutrZe5
初めまして、自分が思うには飛行機が飛ぶのは
翼の上面の気圧が低くなることが一番の理由だと思います。
空気を下向きに流れを変えてその反動で陽力が発生するのも
あると思いますがあまり大きくないと私はおもいます。
 飛行機の失速は翼の上面の空気が剥がれていっきに陽力を失うものです。
 複葉機の陽力の効率は単葉着よりも低い
 尾翼のないコンコルドなどは離着陸のとき翼の後縁をあげる
 翼の後縁が上に反りあがっているSキャンバーの翼型もあります。
わたしもなぜ飛行機がとぶのかよくわかりませんが、
翼の上面の空気の流れが速くなるのは翼の上面の気圧が低くなっている証拠だと思います
空気の流れの進行方向に対して翼の上面が影に隠れていることが
大きく影響していると思います。
500ご冗談でしょう?名無しさん:02/06/08 17:46 ID:XlcBtW+6
晒し上げ
501ご冗談でしょう?名無しさん:02/06/16 02:21 ID:kkMBq6L4
age
502ご冗談でしょう?名無しさん:02/06/16 08:27 ID:???
とんでとんでとんでとんでとんで
とんでとんでとんでとんで
まわってまわってまわってまわる〜
503大鳥居つばめさん ◆/stTH5oI :02/06/16 12:08 ID:???
飛行機は、飛行に関してまだ改良の余地があるんですか?
乱流のところに謎があるんですか?
504ご冗談でしょう?名無しさん:02/06/16 12:17 ID:YukdzHK3
あげ
ページを完全にロードしてないうちに書き込んだら名無しになった。
506ご冗談でしょう?名無しさん:02/06/18 03:32 ID:???
物理屋は、因果関係を追及しながらも、論理関係をその中に見出そうとする。
数学屋は、論理関係を追及しながらも、因果関係をその中に見出そうとする。

飛行機の揚力も、数学屋と物理屋では微妙に説明が違うのは当然か。

物理屋は、運動量保存乃至はエネルギー保存則という論理で説明したがるが、
思考方法は、分子運動レベルの統計的偏りによる因果関係で理解しておる。
数学屋(ベルヌーイも含む)は、エネルギー保存や運動量保存を基礎において
考え(論理的に揚力を考える)ながらも、統計的な安定性は余り信じておらず、
揚力発生という現象に因果関係的な意外性を求めたがるみたいだ。
507ご冗談でしょう?名無しさん:02/06/18 13:05 ID:Nc9fc3Hy
age
508ご冗談でしょう?名無しさん:02/06/18 19:22 ID:Yoz6EuOX
風洞実験して見れ
509ご冗談でしょう?名無しさん:02/06/18 19:35 ID:b3B+/Dv/
飛行機は鉄の塊やから、空飛ぶねん。
硬くて重くて丈夫やから空飛ぶねん。
もし、ぺらぺらな紙で作ってみ?
ぜったい飛ばへんから。まぁ、風に
吹っ飛ばされて、木っ端微塵になって
飛び散るかもしれんがな・・・。
510ご冗談でしょう?名無しさん:02/06/19 18:40 ID:yMyN0HPV
フォースの力だ>>1
511ご冗談でしょう?名無しさん:02/06/21 03:01 ID:???
>>506
数学屋ってゆーか流体力学屋の言ってる因果関係なんていい加減だぞ。
空気のカタマリ同士が圧力を及ぼし合っていると信じ込んでるからな。
それを吹聴して、圧力や粘性を考えるのが苦手な厨房を沢山作ってし
まっているのは、ほとんど罪。と、言ってもわからんだろーな。(フッ
512ご冗談でしょう?名無しさん:02/08/21 03:08 ID:UWCx0kfx
ageてみる
>>505
514ご冗談でしょう?名無しさん:02/09/24 01:39 ID:???
>>499
建前はその通りといいたいところだが、実際の航空機では色々な理由があって、羽で空気の流れを
変えてその反動で飛んでる要素も大きい。それでも、航空機の荷重の99%までは、翼の上下の圧力差
で支えられている。

第一の理由は、圧力差だけに頼ると上昇(下降)する時に、気流の向きが変わるので、圧力差が小さくな
り、上昇速度を維持出来ない。水平飛行に移っても、上下振動を繰り返すことになる。
迎え角の操作で、気流を下に押し下げてその反動を利用して飛んだほうが安定性が良い。(当然燃費は
悪くなるが)。通常の航空機は、低空の場合、相当高速で飛ばなければ、水平飛行する場合圧力差だけ
で高度を維持するのは難しい。そのような速度では、低空大気の密度の不安定さから、機体が速度に
耐えられるかどうかは別問題であるが。

第二の理由は、やはり下降の確実さを追い求めた結果である。上空数千メートルから下降すると航空機は
激しく加速される。上空で十分速度を落とし、スピードを殺しながら降りるのは、気流押し下げ反動を
高度維持に利用するほうが得策である。低空で低速かつ確実に下降するのにも有利である。着陸の際は
なおさら。

第三の理由は、遷音速時には、圧力差による高度維持は難しい。上面の圧力が衝撃波によって高くなって
しまうからである。気流を押し下げる方法ならば、揚力を維持出来るばかりか、増大するとも言える。
(燃料消費は音速以下の場合に比べて甚だしく多い)超音速領域では、気流押し下げのみを揚力としている
といって良い。「波に乗る」という感覚になる。
515ご冗談でしょう?名無しさん:02/09/24 21:55 ID:???
圧力差が生じる理由について言及しない(できない)のが
マヌケな工学屋の特徴か。
516ご冗談でしょう?名無しさん:02/09/24 23:14 ID:???
終わったスレだが…

>>514
>第一の理由
気流の向きにあわせて姿勢を変えれば(迎え角を一定に保てば)圧力差は保てるが?

>第二の理由
>上空数千メートルから下降すると航空機は激しく加速される。
適切な角度で降りてくればそんなことは無い。

517514じゃないが:02/09/25 02:51 ID:???
>>516
迎え角を変えれば、圧力差は保てるが、揚力の向きも変わるので、重力に対抗できない。垂直上昇(したいと
思っても大抵無理で、後ろ向きに墜落してしまうが)したいというなら話は別だが
適切な角度で降りることは、速度を落として、迎え角による気流下向き加速による反動の揚力の助けを借りて
降りること。気圧差だけで揚力を保つ場合、一般に揚力は進行方向成分が発生し、加速されてしまうか、
下降速度が付くと揚力が増し下降速度が落ちてしまう。失速角度で水平飛行しながら減速するとこういうことに
なる。つまり、滑空状態となり、時間もかかり、どこに降りられるのかすらわからくなる。水平飛行のまま
失速してしまうと、制御不能で墜落するタイプの航空機も多い。
518ご冗談でしょう?名無しさん:02/09/25 10:15 ID:???
>>517
> 迎え角を変えれば、圧力差は保てるが、揚力の向きも変わるので、重力に対抗できない。
>気圧差だけで揚力を保つ場合、一般に揚力は進行方向成分が発生し、加速されてしまうか、
> 下降速度が付くと揚力が増し下降速度が落ちてしまう。

多少推力調節してやればいいだけの話とオモワレ。

519ご冗談でしょう?名無しさん:02/09/27 00:51 ID:???
結局このスレも3まで伸びたのに誰も納得のいく説明ができなかったな。
説明が困難であることが分かったことが唯一の成果か。

>>515
言及してみれば。
520ご冗談でしょう?名無しさん:02/09/27 20:23 ID:???
言及してみても、それを理解できないヤシが電波飛ばして終わる罠。
521ご冗談でしょう?名無しさん:02/09/28 18:41 ID:???
もう一度小出しにせず、体系的に一気に全部説明すれば、納得する人が
増えるかもな。
522ご冗談でしょう?名無しさん:02/09/29 02:28 ID:???
解らないところを素直に訊いてみたら。
523 :02/10/12 06:34 ID:R4Ts71cV
今夜の実況は こちらで

乱気流 ◆ TURBULENCE
http://live.2ch.net/test/read.cgi/endless/1034366660/
524物理赤点喰らった運転手:02/11/04 04:57 ID:u+yEdCvK
もうこのスレは死亡かな?1年ぶりにざっと読んだけど、相変わらず難しいっすね。
昔物理をとっていたがサッパリだった、某社ボーイング運転手です。

職場の周りを見ても、「上面がカーブしてて経路が長いから・・・」の迷信ばっかりです。
「出発前のエクステリアチェックで、テメ〜の機種の翼型を下から良く見やがれ」と言いたいところです。
最近は文系出のパイロットが多いので、気にもしないんでしょうね。

もし既出の話題だったらごめんなさい。
自分の昔からの疑問ですが、「フラップダウン時に、一時的に揚力が急増するのはなぜ?」ってのに解答を下さい。

単純化して言うと、200KT、FLAP−UPでPITCH3度で飛んでいたとしますね。
FLAPをひとつ下げると、本来は200KT、PITCH1度で釣り合うとしますね。

ところが下ろした直後には、PITCHをマイナス1度にしないと、上昇してしまうんです。
ある程度の秒数が経った後は、適正なプラス1度PITCHで釣り合うようになります。
この現象は小型機でも大型機でも体験してきました。
もちろんこのPITCH度数は模式的に書いただけで、どんなPITCH変化になるかは機種やFLAP角度に依存しますが。
少なくとも、FLAP−DOWN直後には、その本来のFLAP&速度の組み合わせから見ても、その釣り合い角度よりももっとPITCHを下げなければ、飛行機の高度は一気に上昇してしまいます。
小型機などではFLAPの影響でスタティックポートでの気圧が変化して高度計に悪さをすることもありますが、電波高度計やILS−GSで見ても明らかに上昇します。

FLAPのエクステンドによって、キャンバーの変化と翼面積の増大を作りますね。
結局、定常流の中で翼型を変化させる訳で、この時の過渡的な現象としては、一体どのような事になっているんでしょう?
自分の妄想では、「翼周りの循環の強さが変化するのは、かなりゆっくりなのでは?」と思っています。
循環の「慣性」ってのは意外に大きくて、FLAP操作の前後で、翼型の変化に追い付かずにこういう妙な事を仕出かすんでしょうか?
525ご冗談でしょう?名無しさん:02/11/06 07:09 ID:???
>>524
定常流というのが幻想。フラップダウンすると、流れの慣性によって
翼に作用する圧力分布が変わって強い揚力が発生。しかし時間とともに減少。
揚力は(恐らく)0に近づいていくでしょう。ただし揚力半減期は大気と
航空機の関係から見て、宇宙の年齢より長い筈。理論的には揚力が発生する
定常流は存在しても不安定で安定な揚力0の定常流しか実現されないのでは?
と想像されてます。無論、これは過渡減少としての揚力が発生することを否定
するものではありません。揚力は放射能と同じくやがて0に近づく存在なの
かも知れません。飛行機は空気の流れの過渡現象で飛んでいると推測されて
ます。
蛇足ですがこのことは別にフラップダウンしなくとも、マイナスピッチでも起
こる話です。あるピッチで揚力と重力が釣り合っても、同じ対気速度ならば、
やがて重力が勝って降下が始まる筈です。
氾濫を繰り返す暴れ川の流れが、堆積物によって自然に安定する理由と似て
います。
ところで、循環という概念を使われているようですが、これは理論的過ぎる
話なので、運転中はくれぐれも忘れて頂きたいと思います。
「翼周りに渦が出来るといいんだ〜。あれ、見えるぞ。ラッキー」と誤解され、
失速墜落されることを非常に恐れます。運転中のアルコールは是非お避けくだ
さい。
循環は目で見たり、対気速度のように測定出来るようなものではありません。
ところで、フラップ使わずにマイナスピッチで降下出来る大型航空機の運転
したいと思いませんか?
操縦席から滑走路が見やすくて良いと思いますが。
526ご冗談でしょう?名無しさん:02/11/07 02:54 ID:???
>>524
>単純化して言うと、200KT、FLAP−UPでPITCH3度で飛んでいたとしますね。
>FLAPをひとつ下げると、本来は200KT、PITCH1度で釣り合うとしますね。

ピッチが3度→1度にかかる時間>フラップが下がる時間
だったら上昇しますが、これが原因ではないですよね。

>>525は何気に強力な電波だな...
527ご冗談でしょう?名無しさん:02/11/07 03:04 ID:???
ageてみよう
528ご冗談でしょう?名無しさん:02/11/07 03:33 ID:???
>>525
下降する時、フラップ下げて、そのタイミングにあわせてピッチを下げても同じ現象が起こるのですか?
起こらないとすれば、>>526氏の回答通り、操縦技術が未熟だからでしょう。
普通は、フラップを下げたら、それに応じてピッチを下げて揚力が重力を超えないようにします。それが操縦というものです。
ピッチの下げ遅れで揚力>重力が発生したならば、下降速度が現象し、下手すれば上昇してしまいます。
もしそうだとすると、私は何気に日本のボーイング運転手に強い不安を感じるのですが。ANAだかJASだがJALだか存じませんが、この前酒気帯び
ボーイング運転未遂で捕まったそうではないですか。
よく飲み会の帰りの自動車運転で、スピード付いた状態でカーブでハンドル切って、曲がり出しても、ハンドルを戻さず(時には逆ハン
切る必要ある)、スピンしてあぽーんしてしまうのと同じことだと思いますが。
最近の航空機はラダーも使わないそうですが、もしかして、垂直安定板だけで、逆ハン切らなくても良いと思ってらっしゃいませんか?

失礼なことを書いてしまったかも知れません。
もし、フラップ下げるためにあらかじめピッチを下げ、下降速度が上昇している際に、フラップ出すと何故か、異様に下降速度が小さくなり、
しばらくその状態で待つと下降速度が付いてきて、適正な速度になり、その際おもむろにピッチ上げでその下降速度を維持するというので
あれば、おっしゃる通り不思議な流体的現象だと思います。
529528:02/11/07 03:35 ID:???
528は>>524さんへのレスです。
530524の運転手:02/11/07 20:10 ID:EfhNWBST
みなさん、>>524を良く読んで、もう一度お願いします。

ピッチ3度から1度ではなくて、ある一定時間、「マイナス1度」まで下げないと釣り合わないのです。
フラップセッティングの釣り合いから言ったら、本来は3度から1度まで2度だけタイミングに合わせて下げれば良い筈。
ところが、フラップを動かした後しばらくは、異様に強いリフト増加を体験しています。
ですので過渡領域とでも言いますか、この間は「本来の釣り合いピッチ」よりも意図的にピッチを下げています。
非常に細かい事ですので、はっきり言って安全性には何の関係もありませんが、小型訓練機の頃からの疑問でして。

>>525
マイナスピッチアプローチの飛行機は珍しくありませんよ。
また、普通の大型機のグレアシールドからは、十分に前下方視界が良いですよ。
セスナのような小さい飛行機では前が見にくいですが。

>>528
YAW−DAMPERのRUDDER−COORDINATION機能をご存知でしょうか?
高ターニングレイト時には100%のアシストではありませんが。
逆ハンって何の事でしょうか?
531ずぶ素人:02/11/07 21:03 ID:???

>職場の周りを見ても、「上面がカーブしてて経路が長いから・・・」の迷信ばっかりです

これ、ほんとに迷信なんすか?
532ご冗談でしょう?名無しさん:02/11/08 05:53 ID:???
>>530
多分>>528の「逆ハン」は逆ハンドルのことで、船なら取り舵(だっけ?)
に相当する意味のことだと思われます。旋回修了時に、旋回慣性の回転
を止める為に、逆の舵を取ることですね。自動車でも、高速でカーブを
曲がる時はハンドルを戻すだけでなく、逆ハンドルを切らなければ
曲がれない場合があります。(確実に道交法違反運転でしょうが)

飛行機は慣性モーメントが小さいから、垂直安定板だけで何とかな
るのでしょうけど、船の場合は重いので、必ず取り舵を取らなけれ
ば、横滑りする筈です。飛行機のようにダッチロールすることはな
いでしょうが。飛行機の旋回後に発生するダッチロールをラダート
リムを利用して減衰させるのは、振動周期が短い分、パイロットに
負担をかけるので自動化されているのだと思います。
533532:02/11/08 06:02 ID:???
調べてみたら、取り舵じゃなくて、「当て舵」でした。
素人なもんで、言葉間違えてました。

鬱氏
534ご冗談でしょう?名無しさん:02/11/09 23:47 ID:???
>>524の運転手さん。
フラップ完全に上げた状態で、マイナスピッチで、着陸するって大型機の
場合、大丈夫なんでしょうかね?スピード付き過ぎてしまうんじゃない
でしょうか?
535孫悟空 ◆yGAhoNiShI :02/11/11 17:29 ID:???
ドラゴンボールZ
フジ(関東)で毎週月曜16:30〜放送中!!

  ::., :.;;;:: ;;;;;;;;;;;;;;;;;;;::::.::;;;;;;;;;;;;;;:;;;:;;;;;;;;;;;;: ...: :: ..: :::::::;;;;;;;;;;;;;;;;;;;:..:: :;;;;;;;;;;;;;;
  ;;;;::.::;;:::::::::::.::::..::::::::..,:;;;;;;;;;;;;;;;;;;;;;;;;;;;;;;:;;;;;;;;;::::::;;;;;;;;;;;;;;;;;;::::::::::::::.:;;;;;;;;;;;;;;;;;
  ;;;;;;;;;;;;;:::.:::.;;;;;;;;;;;;;;.:::;;;;;;;;;;;;;;;;;;;;;;;;;;;::...::...,;;;:..:.:::::::: . ;;;;;;;;;;;;;;;;;;;;; :;...::.:;;;;;;;
  ::.::..:.:::;;:::;;;;;;;;;;;;::::.;;;;;;;::::::;::.;;;;;;:::..     .::::.,::;;;;;;:::;;;;;;;;;;;;;;;;;;:::: ::;;;;;;;;;;;;;;
  ;;;;;;;;;;;::: ::::;;;;;;:: ::: ::::...  .: . . _.∩_  ..:;;;:;;;;:;;;;;;;;;;;;;;;;;;;;;;:::: :;;;;;;;;;
  ;;;;;;;;;;;;;;;;;;;;;;::::::...         ヽヘ;;. 人丿ス  :: ::::::;;;;;;;;;;;;;;;;;;;:::;;;; .:;;;;;;;
  : :. :;;;;;;;;;;;;;;;;;;..    从    θ斤:エh u    .:::::;;;;;;;;;;;;;;;;;;;;;;;;;;;;;;;::;;;;
  ;;;;:;,:.:;;;;;;;;;;;;:::.  __ 《Y》_   ∪レ..... 弋|      :::::;;;;;;;;;;;;;;;;;;;;;;;;;;;;;;;::;
  ;;;;;;;;;;;;;;;;;;;;::...  .uヘ人iイ  .  (. .」_ ノ    ...::::;;;;;;;;;;;;;;;;;;;;;;;;;;;;;;;;;;;;;:
  ;;;;;;;;;;;;;;;;;;;;,:..     (∨ヘ      |....|: .)    .:::;;,,;;;;;;;;;;;;;;;;;;;;;;;;;;;;;;;;;;;;;;
  ;;;;;;;;;;;;;;;;;;::::....    .|;|レ'      .(_;);;.| -〜、 ..:..:..:,,;,;;;;;;;;;;;;:::::: :::: ::::
  、 ._  _.:;〜⌒^^⌒⌒´⌒` ̄ ̄ ....::,...⌒~^⌒ ̄ ̄`〜._:::;:..:::
   ⌒ ⌒    ....::::::::::.:::.::;: :::... .:::: :::.  ..::  :::::::: ;::::::;:;.;:;,;,.,,; ...::⌒
  :;;;:::;::;: :::;:;;:::::::..::::::::::::::::;::::::::::::::::::::::::::::::::::::.  :; ..;::::::;::;: :::;:;;:;:::::;: :::;:;::
と〜けたこおりのな〜かに〜♪恐竜がい〜たら〜たまのりし〜こ〜みたいね〜♪
536ご冗談でしょう?名無しさん:02/12/05 16:46 ID:6WkINEYl
age
537孫悟空 ◆yGAhoNiShI :02/12/09 16:05 ID:sm7i5Nj0
ドラゴンボールZ
フジ(関東)で毎週月曜16:30〜放送中!!

  ::., :.;;;:: ;;;;;;;;;;;;;;;;;;;::::.::;;;;;;;;;;;;;;:;;;:;;;;;;;;;;;;: ...: :: ..: :::::::;;;;;;;;;;;;;;;;;;;:..:: :;;;;;;;;;;;;;;
  ;;;;::.::;;:::::::::::.::::..::::::::..,:;;;;;;;;;;;;;;;;;;;;;;;;;;;;;;:;;;;;;;;;::::::;;;;;;;;;;;;;;;;;;::::::::::::::.:;;;;;;;;;;;;;;;;;
  ;;;;;;;;;;;;;:::.:::.;;;;;;;;;;;;;;.:::;;;;;;;;;;;;;;;;;;;;;;;;;;;::...::...,;;;:..:.:::::::: . ;;;;;;;;;;;;;;;;;;;;; :;...::.:;;;;;;;
  ::.::..:.:::;;:::;;;;;;;;;;;;::::.;;;;;;;::::::;::.;;;;;;:::..     .::::.,::;;;;;;:::;;;;;;;;;;;;;;;;;;:::: ::;;;;;;;;;;;;;;
  ;;;;;;;;;;;::: ::::;;;;;;:: ::: ::::...  .: . . _.∩_  ..:;;;:;;;;:;;;;;;;;;;;;;;;;;;;;;;:::: :;;;;;;;;;
  ;;;;;;;;;;;;;;;;;;;;;;::::::...         ヽヘ;;. 人丿ス  :: ::::::;;;;;;;;;;;;;;;;;;;:::;;;; .:;;;;;;;
  : :. :;;;;;;;;;;;;;;;;;;..    从    θ斤:エh u    .:::::;;;;;;;;;;;;;;;;;;;;;;;;;;;;;;;::;;;;
  ;;;;:;,:.:;;;;;;;;;;;;:::.  __ 《Y》_   ∪レ..... 弋|      :::::;;;;;;;;;;;;;;;;;;;;;;;;;;;;;;;::;
  ;;;;;;;;;;;;;;;;;;;;::...  .uヘ人iイ  .  (. .」_ ノ    ...::::;;;;;;;;;;;;;;;;;;;;;;;;;;;;;;;;;;;;;:
  ;;;;;;;;;;;;;;;;;;;;,:..     (∨ヘ      |....|: .)    .:::;;,,;;;;;;;;;;;;;;;;;;;;;;;;;;;;;;;;;;;;;;
  ;;;;;;;;;;;;;;;;;;::::....    .|;|レ'      .(_;);;.| -〜、 ..:..:..:,,;,;;;;;;;;;;;;:::::: :::: ::::
  、 ._  _.:;〜⌒^^⌒⌒´⌒` ̄ ̄ ....::,...⌒~^⌒ ̄ ̄`〜._:::;:..:::
   ⌒ ⌒    ....::::::::::.:::.::;: :::... .:::: :::.  ..::  :::::::: ;::::::;:;.;:;,;,.,,; ...::⌒
  :;;;:::;::;: :::;:;;:::::::..::::::::::::::::;::::::::::::::::::::::::::::::::::::.  :; ..;::::::;::;: :::;:;;:;:::::;: :::;:;::
と〜けたこおりのな〜かに〜♪恐竜がい〜たら〜たまのりし〜こ〜みたいね〜♪
538ご冗談でしょう?名無しさん:02/12/09 22:35 ID:???
>>534
主翼の取り付け角次第じゃないの?
539ご冗談でしょう?名無しさん:03/01/04 04:10 ID:ohTj2Yyx
あげ
540ご冗談でしょう?名無しさん:03/01/04 18:01 ID:7GKm6cp/
飛行機って離陸するときジェットエンジンのみで加速してるんですか?
タイヤには駆動装置ついてないの?
541ご冗談でしょう?名無しさん:03/01/04 23:36 ID:???
>>540
飛行機にはレシプロ機も有るでよ
542ご冗談でしょう?名無しさん:03/01/05 01:09 ID:???
1/4朝日の夕刊「ののちゃんのふしぎ玉手箱」にある飛行機の説明はイイ!
取材協力したセンセイが良いのか。
543山崎渉:03/01/11 19:29 ID:???
(^^)
544ご冗談でしょう?名無しさん:03/01/13 10:52 ID:quK/u93a
亜音速と超音速とでは航空機の設計方法が根本的に異なり
一般的なジェット旅客機のように、亜音速飛行を前提としたものでは機体先端部分が丸みを帯びており
超音速飛行を前提としたもの(コンコルド)では先端が尖っています
これは亜音速では機体に作用する抗力の支配的なものが形状抵抗であるのに対して
超音速では造波抵抗が支配的な抗力になるからである、
と聞いたのですが
先端が尖っている事が造波抵抗を軽減させる事になる、というのが理解できません
だれか教えて下さい
545ご冗談でしょう?名無しさん:03/01/13 11:09 ID:???
機長が舞空術を使ってるから。
546ご冗談でしょう?名無しさん:03/01/13 12:50 ID:Pflrhqcn
>>544
先端がとがってる場合の方が,鈍頭の場合よりも衝撃波が弱くてすむから.
547^^^^^^^:03/01/13 12:52 ID:Y3cNgvQ/
548ご冗談でしょう?名無しさん:03/01/16 13:22 ID:t5b5KbAM
概出だったらすみません
水平方向に移動している翼にあたる(影響を受ける)空気ってのは
翼上面を通る空気の場合、翼が近付くにつれて上昇しはじめ、前縁を過ぎた途端に下降するわけですよね?
このとき
ある一つの空気中の分子を考えると、翼が近付いて来た事によって静止した状態から徐々に上昇しはじめ
前縁通過後、下降して翼後方に流れていく事になりますが
トータルでは上昇量よりも下降量のほうが多いのですか?
つまり、最初いた位置よりも翼通過後は下がっている?
549ご冗談でしょう?名無しさん:03/01/17 00:29 ID:???
>>548
まぁ、言いたいことはわかるが、細かいところでちょっと違うな。
まず空気の分子は「静止」していない。多数の分子の速度を平均
するとゼロになるだけ。翼が通過すると空気は平均として下向きの
速度を得る。で、このとき翼には上向きの力「揚力」が働く。
翼が通過した後には空気の下向きの流れが生じているんだな。
550ご冗談でしょう?名無しさん:03/01/17 09:17 ID:dy8XjVxU
>>549
レスどうも
静止しているってのはあくまでも仮定で書きました
てことはトータルでは空気は下向きの運動量を得た事になるので
逆に翼は上向きの運動量を得たという解釈でいいんですね
551ご冗談でしょう?名無しさん:03/01/18 06:16 ID:???
>>550
運動量保存とゆーよりも運動量の変化=力積。
水平飛行でも空気は下向きの運動量を得るからな。
552山崎渉:03/01/18 12:20 ID:???
(^^)
553ご冗談でしょう?名無しさん:03/01/25 12:12 ID:???
554ご冗談でしょう?名無しさん:03/01/25 12:23 ID:???
おまえらいつまでやってるんだよ!!!
こんな簡単な問題で詰まってるのか?
ベルヌ(以下略)
555ご冗談でしょう?名無しさん:03/01/26 10:05 ID:dTC8z9/B
このページとかどうかな
飛行機はなぜ飛ぶのか――「ベルヌーイの定理」説に挑む
紙飛行機をうまく飛ばす科学 (1)(2)(3) ――― 飛行機が飛ぶわけ
飛行機が飛ぶわけ―――「ベルヌーイの定理」説をめぐる論争を解く (1)(2)
http://www.sanwaprn.co.jp/taruta/paperplane/
556ご冗談でしょう?名無しさん:03/01/26 10:22 ID:???
557ご冗談でしょう?名無しさん:03/01/27 02:52 ID:???
>>556
このスレをカナ〜リ参考にしてるな (w
出来は悪くは無いが「ちゃんと物理わかってるのかよ」と思わせる記述が結構ある。
NS方程式が難しくて揚力はよくわからん、と書いた紙飛行機センセーよりマシだがな。
558ご冗談でしょう?名無しさん:03/01/28 03:57 ID:???
>>548
大気と飛行機の翼との相互作用だけじゃ揚力は発生しないべ。
地球が絡んでくる。地球も含めて初めて運動量保存の法則が成り立つ。
だから、大気が得た運動量=-翼が大気から得た力積
という式は正しくない。
559ご冗談でしょう?名無しさん:03/01/28 23:00 ID:???
>>558

ネタならクサすぎ。
マジならバカすぎ。
どっちにしても物理はニガテなようで…
560ご冗談でしょう?名無しさん:03/01/29 03:06 ID:???
>>559
大気が地球から受ける力積を考えないのか?
561ご冗談でしょう?名無しさん:03/01/29 23:41 ID:???
>>560
上空を飛んでいるときには考えないよ。空気は周りに沢山あるから
離れたところで上向きの流れがあればトータルで運動量は保存する。
グランドエフェクトの時は地面を考えるけど、>558は違うだろ。
562ご冗談でしょう?名無しさん:03/01/30 03:30 ID:???
>>561
無重力状態でも揚力は発生するの?重力定数に無関係なら,一体何に依存
するの?
563ご冗談でしょう?名無しさん:03/01/30 03:49 ID:???
>>562=558?
発生するよ。空気の運動量の変化に依存するよ。
564ご冗談でしょう?名無しさん:03/01/30 17:51 ID:???
>>563
空気の運動量変化=揚力なんだろ?
これが重力定数に無関係な値を取るというんだったら何に依存するんだい?
空気の密度かな?
565ご冗談でしょう?名無しさん:03/01/31 02:57 ID:???
重力定数ってなんだろ?
これが運動量に関係するって何の話だい?
久々の電波かな?
566ご冗談でしょう?名無しさん:03/01/31 16:06 ID:???
>>565
翼周りの空気と翼との相互作用だけで揚力が発生していると近似するのは
正しくないと思われ。静力学的性質を無視し過ぎ。飛行機はその下の空気
に「乗っている=荷重を預けている」という性質も持つ。下に空気を投げ下
ろして...その反動で...大気圏境界でのロケットの行動じゃあるまいし。
投げ下ろそうとする空気も、地面でBoundされてるから自由に投げ下ろす
ことは出来ないわけね。つまり運動量だけで説明しようとすると、投げ下ろす
空気の見かけ上の密度を多めに見ておかなければならないことになる。
ついでに言えば運動量保存則で説明するのが何故まずいかといえば、運動量
保存則は飛行機に揚力が発生しないということも含んでいるから。運動量だけ
だとどの位の揚力が発生することを決定できない。
567ご冗談でしょう?名無しさん:03/02/01 02:04 ID:???

     デムパ、キタ━━━━━(゚∀゚)━━━━━!!
568ご冗談でしょう?名無しさん:03/02/01 03:10 ID:???
分からなければ何でもデムパ(藁
素直にわかりません。間違っていましたといえばいいのにね。(藁藁藁
569ご冗談でしょう?名無しさん:03/02/01 17:39 ID:???
>>566がデムパな理由
揚力と浮力の区別が出来ていない。
空気はつながっていて自由に動かせないと思い込んでる。
ヘリコプターの回転翼の下側に生じる気流は考えられない。
運動量といえば「保存則」のみで、力積を知らない。
理解できないことは自分の妄想で補完して、それを正しいと思い込んでいる。

以上から単なるシッタカではなくキティちゃんに認定。
570ご冗談でしょう?名無しさん:03/02/01 18:37 ID:???
>>569
問題は重力加速度(物理定数でこれくらいは知ってるだろ。)に対抗できる
くらいの空気の投げ下ろしをやってるかどうかだろ。どの位空気の要素が独立
に動けるかってことだが、空気の独立性を認めたら、真空部分が出来ることも
認めなけりゃならないな。あと大気圧も認めるだろ、当然。そうなると
大気圧と空気を投げ下ろして出来た「真空」との間に翼が入ったら強い浮力
が発生してるのはわかるよな。これは飛行機を語るのに無視できる要素か?
571ご冗談でしょう?名無しさん:03/02/02 01:35 ID:???
>>556
ポイントを一点に絞るべきだったよね
572ご冗談でしょう?名無しさん:03/02/02 04:21 ID:???
>>570がデムパな理由
ちゃんと勉強していないから用語の使い方が不適切。
独自にあみ出した変な造語が他人にも通用すると思い込んでいる。
文章がシリメツレツ。最後の2行は意味不明。
「浮力」と書いてしまっているところで自爆だが。

一度ホバリングしているヘリの直下に入ってみな。
妄想も吹き飛ぶぜ。
573ご冗談でしょう?名無しさん:03/02/02 05:58 ID:???
(1)
飛行機の周囲の空気が、飛行機によって力を受けて、その運動量だけが変わることに
より飛行機の落下を防ぐだけの上向きの力を与え、下の空気や地面に飛行機の荷重
をなんら伝えない。伝えるとしても、風となって、地面にぶつかることによるもの
で、相当に時間が経ってからである。

なのか、
(2)
それとも、飛行機の荷重の殆どが下の空気に伝わり、飛行機はいわば静力学
的に重力と釣り合っている状態であり、周囲の空気に与えた運動量変化が
飛行機に作用する力の鉛直方向の成分に寄与する割合は一般的に言って
相対的に小さい。

のでしょうか?
個人的には(1)は飛行機とロケットを混同しすぎだと思います。(2)はその
理由がわかりにくいです。何故速度0でそのようなことが起こらないのか?
574ご冗談でしょう?名無しさん:03/02/02 22:50 ID:???
>飛行機とロケットを混同しすぎ

どこが?
使うガスが外部か内部で違うが。
575ご冗談でしょう?名無しさん:03/02/03 06:43 ID:???
>>574
飛行機の飛ぶ力は静力学的な力に近いでしょう。やっぱり。
羽が空気を引き裂いて「穴」を作ろうとするので、それを埋めるように
力が働く。その力の源は大気に働く重力。
巨大な力だけど、余り仕事をするわけじゃないので、エネルギーロスが少ない。
水平飛行するハリアーのパワーが水平停止垂直上昇「ホバリング」ハリアーに
必要なパワーよりずっと小さいのは何故か説明できる?
576ご冗談でしょう?名無しさん:03/02/08 02:57 ID:???
>>575
それって浮力。
揚力じゃないね。
ハリアーの話は既出。
577ご冗談でしょう?名無しさん:03/02/08 18:54 ID:???
578ご冗談でしょう?名無しさん:03/02/08 23:53 ID:32H20jl5
飛行機はどうして飛べるの?子供から聞かれたときに、こういう説明をしてます。

・上下の形が違う物体を、水平な流れの中に置く。
・流れが当たって、後ろに流されようとする力が働く。これは納得できるね。
・上下の形が違うから、上か下かにも流されようとする力が働く。これも納得できるよね。
・上下の向きを、力が上に向くようにしよう。
・昔の人が、ありとあらゆるいろんな形を試して、上向きに働く力が、後ろに働く力の
 15倍にもなる形を見つけてくれた。その形を、翼型っていうんだ。
・翼型、これはすごい発見だった。後ろから1の力で押してやれば、15の重さの
 ものを空気中に浮かべることができる。飛行機までもうちょっとだ。(続く)
579ご冗談でしょう?名無しさん:03/02/08 23:54 ID:32H20jl5
(続き)
・別の昔の人は、空気を吸い込んで後ろに吐き出して、前に進もうとする力を生み出す
 機械を、一生懸命作った。今その機械はすごく進歩して、灯油を燃やして、自分の重さの
 5倍くらいの前に進む力を出せるようになった。この機械をジェットエンジンっていう。
・ジェットエンジンと翼と、進むのに邪魔にならないよう進行方向に細長くした、
 人が乗ったりする胴体を組み合わせよう。胴体や翼には、
 向きを変えたりするための舵もいっぱいつける。重さは大したことないよ。
・ジェットエンジンの重さが1、前に進む力が5、浮かぶ力が75になる。
 胴体と翼と人間と灯油を合わせた重さを74以下にすると、この機械は空にうかぶ。
 これが飛行機だ。ただ、74だとぎりぎり地面から浮かべるだけなので、
 本物の飛行機は値段が高いけど軽い材料をいっぱい使って、全部の重さを15
 くらいに抑えている。これくらいだと、エベレストより高い空にも行けるし、
 灯油をいっぱい積めば、太平洋だって渡ることができる。
・ちなみに飛行機は、いい材料をいっぱい使って、精密に作られているので、
 1tあたり1億円くらいする機械になっている。自動車のだいたい20〜50倍
 の値段だね。
580ご冗談でしょう?名無しさん:03/02/10 00:55 ID:wT8eQuqV
>>556のリンク先を見て思ったのだけど、

他の物理屋さんが片手間に流体力学をかじると、渦度と渦の違いを分かってない
ことが多いような気がするね。

どんな激しい渦でも、剛体回転してれば渦度はゼロだし、せん断流れでは流れは
平行だけど、あらゆる領域に渦度がある。

流体力学は古い学問なので、使ってる数学もクラシックな数学だ。だけど逆に、
いくら難しいレトリックを使っても、分かってる分かってないは誤魔化せないよ。
581580:03/02/10 01:21 ID:RiyDD31x
>>578-579
言い忘れた。たぶん工学屋さんだとおもうけど、もう来なくていいよ。
こういうのを見て分かった気になるバカが増えるから。
582ご冗談でしょう?名無しさん:03/02/10 03:53 ID:???
>>580
すぐに人を憶測でレッテル貼り付けるヤシもそれほど賢い知能を持ってる
とはいえないのでは?
>>578-579の議論は確かにこのスレで議論されてることを余り熟知してやった
カキコとは思えないが、一生懸命、一般的な説明を考えてくれてることは
評価してやったっていいと思うぞ。物理版の説明としては相応しくないのは
間違いないが。(その翼型というのが本当に試行錯誤の成果なのかは、結構
重要。そうでない方法との相違は、統計力学と量子力学の方法論の違いと同じ
とさえ言える。)
他人を罵倒することがまかり通るスレは沈む沈む。
583ご冗談でしょう?名無しさん:03/02/10 13:58 ID:???
>>578
それで、過去ログは呼んでみたかい?
584ご冗談でしょう?名無しさん:03/02/11 19:17 ID:???
う〜ん、>>578-579は単にスレ違いなだけで、変なことは書いてないぞ。
>・上下の形が違うから、上か下かにも流されようとする力が働く。
これの理由がスレの話題の中心かな。

アブないのは>>575の方だな。自らの狭い知識だけで誤った結論に達しても
それを正しいと信じて疑わないタイプ。このスレにも沢山出没してた。

>>580みたいのも結構いたな。流体力学を習いたてのコドモ。
空気の「剛体回転」とか書いて、そのマヌケさに気がつかない。
585ご冗談でしょう?名無しさん:03/02/12 00:38 ID:???
非対称な形状をしたモノが流れの中で受ける力を、非対称抗力とでも
呼ぼう。
この力を重力の存在によって影響を受ける部分とそうでない力に分解
できるとする。
前者を浮力、後者を揚力と定義しよう。式で書けば非対称抗力をF
揚力をL、浮力をBとする。gを重力加速度、Vを流体の速度として
F(V,g)=B(V,g)+L(V)

飛行機は、当然非対称抗力を受けて重力と釣り合って飛んでいる。
この非対称抗力は、重力に依存するのしないの?かが、議論の中心

「しない」が衝動とか粘性によって、流れがモノの境界を認識して流れと
モノの間に相互作用が働き揚力となるという。ベルヌーイの定理を使って
揚力を理解することには基本的に反対している。上の式で飛行機に働く力
はLが主体でB項はVの大きさにかかわらず小さいとしている。

「する」が、この非対称力の正体は浮力であり、重力加速度の大きさに関係
しているとする。流れは、翼の境界を認識できず、翼から慣性によって離れて
流れようとするが、その際に翼に働く大気圧のバランスから翼に流れが剥れ
ようとしている場所めがけて力が働く。よく為されるベンチュリ管の原理
(ベルヌーイの定理を用いた)による説明とは独立していることには注意
(上部の流速が変化していてもしてなくてもこの説明には矛盾しない。)
勿論この説明では、上の式でL項は常に0.つまり非対称力は常にBであり
V=0の時に限り浮力と呼んでいる。
586ご冗談でしょう?名無しさん:03/02/13 09:40 ID:???
ずっと前に既出のような気がするなぁ、「揚力は浮力だ」ってヤシ。
新手の電波か?はたまた妄想の煉獄から脱出できないでいるのか?
まぁ、飛行機は空気に「浮く」(水には沈みそうだが)ってことを主張したい
ようだから放置でもいいのだが、ちょっとだけ。

>翼から慣性によって離れて流れようとするが

そう、
翼に沿って流れた空気は下向きの運動量を得たまま離れる
と考えるのがフツウ。

>その際に翼に働く大気圧のバランスから翼に流れが剥れようとしている
>場所めがけて力が働く

ここが妄想。そんな気がしてるだけ。
翼に対してどのような力がどのように作用するのかは示さない(せない)。
(無理なコジツケをここでひけらかすのは止めてね)

あと、浮力と比重(密度)の関係を調べておくのは、夏休みの宿題にするね。
587ご冗談でしょう?名無しさん:03/02/13 10:59 ID:???
>>586
書いてるじゃん
>その際に翼に働く大気圧のバランスから翼に流れが剥れ
>ようとしている場所めがけて力が働く。

>上の式でL項は常に0.つまり非対称力は常にBであり
>V=0の時に限り浮力と呼んでいる。
この文ちょっとおかしいね。
「通常V=0の時に限り浮力と呼んでいる」
に直したほうがいいね。でも意味は通る。
飛行機を重力に対抗させている力はB(V)で、それもVが十分大きい時のもの
だってことになるね。>>585はV=0以外の場合も、B(V)のことを浮力と
呼んでるみたいだね。
空気の内部エネルギー変化が伴う、膨張が絡んでいるんでマクロレベルでの
周辺大気の運動量変化と飛行機の得る力積の間に簡単な関係があるかどうか
>>585の「する」派説では「ない」と言わざるを得ないだろうね。
実際、飛行機のDown washは断熱膨張を伴っているせいか、湿度が高い日
なんて水分が霧になってはっきり見えるじゃない。運動量で簡単に割り切れる
世界だとは思えないな。
588ご冗談でしょう?名無しさん:03/02/13 11:15 ID:???
>>586
> 翼に沿って流れた空気は下向きの運動量を得たまま離れる
> と考えるのがフツウ。
フツウかどうかは知らんが、これもそんな気がしてるだけなワナ。
589ご冗談でしょう?名無しさん:03/02/13 23:07 ID:???
陽気の所為か、電波の影響か、また変なヤシが湧いてきた…
自分の知らないことや理解できないことには否定的とゆー
マヌケなスタンスが共通項だな。

>>587
大気圧のバランスがどーなるとどっち向きの力がどこに作用するのかね?
まぁ、587は内部エネルギーと運動量の区別もつかないくらいのメデタさだから
電波のシンパってところか。熱力学の理解には程遠い罠。

>>588
可視化した翼まわりの流線の図を見たことが無いとは、イトアハレ。
590ご冗談でしょう?名無しさん:03/02/14 01:56 ID:???
>>589
見たことあるってば。
考えるのがフツウ、ってんじゃ"翼の上側の方が長いんだからそっちの方が速く流れると
考えるのがフツウ"てのと変らんぞと言いたかっただけ。
591あんがいムズいな:03/02/14 02:14 ID:???
航空宇宙板の似せパイロットと(ホントかどうかわかんないけど)本物パイロットの話も
参考になるかもね

体感している者だけが真実を知る なーんて
592ご冗談でしょう?名無しさん:03/02/20 03:09 ID:ZwDScaN9
ここの一部の人が言っている、揚力に重力が関係している、
というのが、やっぱりどうしても分からない。

流体力学の授業で、NS方程式の厳密式で、まず単純化のために落とすのが重力項で、
ちょっとでも習った人ならみんな経験あると思うんだが、もしかしてここの人たちは
定説を疑う、もっと高度な議論をしてるのでしょうか?
593ご冗談でしょう?名無しさん:03/02/20 16:29 ID:???
>>592
NS方程式は流体力学の方程式だけど、数学的な興味が先行して物理的な
付帯条件を極端に単純化している.(方程式自体の成立条件や境界条件や
圧縮性の無視など)
その段階で流体の問題から派生した問題であることすら忘れてしまってい
ると考えてよい。
それで得られた結果が現実の現象を説明していなくても何の問題も無いし、
逆にその方程式で得られた解が現実に起こると即断するのも間違い。
まぁ、方程式の解が存在するかしないかのぎりぎりのところで揚力が発生
するかしないかが決まるみたいなので、こういった議論が無意味とも言えない
でしょう。
594592:03/02/21 01:37 ID:rw4Ig4e8
>>593
了解です。ちゅうことは、議論の前提として、方程式は仮定や単純化を使った、
説明であって自然そのものではないので、とりあえずNS方程式は忘れると。
で、流体が流れてると。
その中に、>>585さんのいう非対称物体がおいてあると。
この物体に、非対称抗力と呼ぶ力が働いて、それを水平方向と垂直方向に分解して、
水平方向に働く力を揚力と呼称すると。
で、この揚力に、地球の重力は関係してるか、いなか、ということで、
いいでつか?
595592:03/02/21 01:42 ID:rw4Ig4e8
んで、一つの思考実験として、模型飛行機を作って、ビルの上から
垂直に落として見ると。この模型飛行機は、本物の飛行機みたいに、
翼型が上下非対称になってますと。

で、この実験の重要ポイントとして、垂直に落とすので、重力の影響は、
飛行機から見た座標系では、水平方向に限られてますと。重力は、飛行機を
前に引っ張ってるだけですと。

さて、この飛行機は、@まっすぐ落ちる、A翼の揚力の方向にずれながら
落ちる、…みなさんはどう思います?
596ご冗談でしょう?名無しさん:03/02/21 01:59 ID:HINToUKu
私流体力学専門ではないが、NS方程式など水くらい密度の高い
ものから、空気ほど密度の低いものまで、使ってるようだが?
空気ほど、密度の低い物に、NS方程式など使う厳密な根拠どこにあるの?
一応局所平衡の仮定が成り立つ程度の密度なんだろうけど?
最終的には、吹流し作って実験だろ?それとも、厳密な方程式
あるの?教えて?多分、ちょっとかっこ付けた方程式もってきても
厳密解が得れても、部分的にしか表現できないのが現状じゃないの?
597ご冗談でしょう?名無しさん:03/02/21 02:18 ID:yCYmqO1Q
>>595 それ、飛行機の離陸の時とどう違うの?
598ご冗談でしょう?名無しさん:03/02/21 02:32 ID:bKOgkbM+
>>596
空気の密度が薄い?
何を根拠に、あなたが密度が薄いと言われるのか分かりません。
あなたが仰ってるのは、単位体積あたりの重さとしての密度?
それが、連続体近似に何の関係があるの?
例えばあなたの言ってることは、極端に言えば、
同じ体積に、同じ数の分子が入っていて、でも分子量が小さくなると、
連続体近似が成り立たなくなるということではありませんか?

気を悪くしたら謝りますが、根本から考える機会なので、
ぜひ教えて下さい。
599ご冗談でしょう?名無しさん:03/02/21 02:54 ID:aAsimC5M
>>598 あのですね!要は、熱力学で言う所の局所平衡の仮定が成立
が最低限度の要求ですよね!そうで、ないとあなたの言う所の
連続体近似?も成り立たないはずです。
私の疑問は、空気ほどの密度で、どの程度局所平衡の仮定が成立してるのか?
を知りたい訳です。水に比べると、成立が難しいのではないか?
と思うちゃう訳です。
600ご冗談でしょう?名無しさん:03/02/21 03:29 ID:zivYOtij
>>599
598の投稿をした者です。教科書的な答えで恐縮ですが、昆虫、人間、飛行機、
の周りの流れであれば、十分連続体とみなせると言われています。
なにしろ、連続性を気にしなければならない相手が、空気の平均自由工程なので。

ただし、あなたが仰るように、空気でもNS方程式が成立しなくなる
ケースもたくさんあります。例えば、低い軌道を飛ぶ人工衛星の周りの、
希薄な空気の流れや、HDのような記録媒体と読み取りヘッドとの間の、
極小隙間の流れなどです。このような場合はNS方程式ではなく、
ボルツマン方程式が使われるとのことです。(残念ながら私は後者は
よく知りません)お答えになっていれば幸いです。
601ご冗談でしょう?名無しさん:03/02/21 10:57 ID:???
>>595
その考え方は正しくないと思う。空気に重力が作用しているか否かが問題で
あって、重力の向きは関係無い。飛行機が重力による落下運動から空気と
の相互作用でどれだけずれるかが問題。
602ご冗談でしょう?名無しさん:03/02/21 21:42 ID:46vZuYCf
>>599
598の投稿をした者です。教科書的な答えで恐縮ですが、昆虫、人間、飛行機、。。。。。
の周りの流れであれば、十分連続体とみなせると言われています。
なにしろ、連続性を気にしなければならない相手が、空気の平均自由工程なので。

ただし、あなたが仰るように、空気でもNS方程式が成立しなくなる
ケースもたくさんあります。例えば、低い軌道を飛ぶ人工衛星の周りの、
希薄な空気の流れや、HDのような記録媒体と読み取りヘッドとの間の、
極小隙間の流れなどです。このような場合はNS方程式ではなく、
ボルツマン方程式が使われるとのことです。(残念ながら私は後者は
よく知りません)お答えになっていれば幸いです。
603ご冗談でしょう?名無しさん:03/02/24 01:12 ID:3z+I6t8D
理屈としては合ってても、翼が生み出す空気の投げ下ろしの運動量が、
飛行機を浮かせるだけの巨大なものだというのが、どうも納得できない、
もやもやした感覚は確かにある。

それはたぶん、翼の近傍だけの、わずかな質量の空気を意識しているからだろう。

想像だが、翼は多分、その周りの想像以上に広大な空間の、空気の流れに
影響を与え、そのために巨大な力で上に持ち上げられているのではないか?

もし航空屋さんが見てたら、その辺(漏れの想像だが)がヴィジュアルに
分かる、風洞試験とかの絵があったらうpきぼんぬです。
604ご冗談でしょう?名無しさん:03/02/24 04:21 ID:???
>>603
投げ下ろされる翼の上の空気も、運動している以上その下の空気や地面
にいずれは衝突する。
実際には音波の速度で力が伝わって、下の空気と一体化して行動する
筈。もし大量の空気が翼から力が伝わって落ちてくるのならば、
そう考えたほうが自然だね。
となると、地面で空気はバウンドされているので、地面との間に
働く力が無視できなくなる。少なくとも空気の運動量変化は単純に
計算出来ない。
仮に空気と翼の間に引力めいたものが働くことを先見的に認めると
しても、それを空気の運動量変化をもって実証するのは難しそうだね。
605ご冗談でしょう?名無しさん:03/02/24 04:49 ID:ujTaMVWq
>>604
翼が影響を与える空間の、下半分がほとんど地面に占有されるような、
低い高度を飛んでいる場合は、確かにそういう影響はあるだろうが‥‥‥。

ゴメソ。
きみの理論だと、1気圧、1Gでも、地面が(仮に)何十億kmも下にあった場合、
飛行機は飛べないということを言いたいのかしら?

それとも、プールで力いっぱい立ち泳ぎして、体の大部分を水から上に出したら、
その分地球が、ちょっとだけ下に下がってる、ということを言いたいのかしら?
(後者は同意です)
606ご冗談でしょう?名無しさん:03/02/24 14:08 ID:???
>>604
1気圧1Gで、空気塊が何故か地面に接していないで、なぜか膨張もせず同圧かつ
塊の状態で地面から浮いた状態で全体として落下運動しているところに、同様に鉛
直方向には落下運動している飛行機が飛び込んできて、空気力を得て、重力と釣り
合い鉛直方向には等速運動(つまり重力が打ち消される)になったとする。
この時に空気塊全体の運動量変化がきちんと飛行機が単位時間に重力によって
獲得する力積に一致していれば、このスレで主張されている
「運動量によって単純に理解しましょう」説はある程度実証されたことになる
でしょうね。しかし根本的に何故翼と空気の間に引力が働くのかという説明力
は0に等しいと思うが。
607ご冗談でしょう?名無しさん:03/02/25 03:55 ID:9BtMD3Do
>>606
翼と空気との間に働く引力‥‥‥‥‥‥‥。

単純に、翼の上面から押す力 < 翼の下面から押す力
と考えたらダメなんでしょうか?

空気はただひたすら、ぐいぐい押してくるだけだと思うんですが‥‥‥?!
608ご冗談でしょう?名無しさん:03/02/25 04:23 ID:???
>>605
デムパやkittyにはマジレスしてはダメだよ。いくら説明してやっても全く理解しないし、
それどころか意味不明な妄想を性懲りも無く垂れ流してスレを汚していく。しかも
その毒気は結構強力で、>>592の最後の行にあるように変な疑問も誘発してしまう。

揚力の原因は空気の運動量変化で説明して、翼のどこにどれくらいの力がかかる
のかは空気を連続体近似してNS方程式で求める、というこでいいと思うのだが…
609ご冗談でしょう?名無しさん:03/02/25 04:29 ID:???
>>607
>空気はただひたすら、ぐいぐい押してくるだけだと思うんですが‥‥‥?!

飛行機スレを全部読んだほうが良いな。
610607:03/02/25 04:50 ID:t6+GlaBo
>>608
さん、サンクスコ。

>揚力の原因は空気の運動量変化で説明して、翼のどこにどれくらいの力がかかる
>のかは空気を連続体近似してNS方程式で求める、というこでいいと思うのだが…

‥‥‥で、いーんだよねー!
611ご冗談でしょう?名無しさん:03/02/25 12:27 ID:???
生=死に至る病
飛行機=墜落に至る機械
612ご冗談でしょう?名無しさん:03/02/25 13:29 ID:???
音速を基準として十分地面から遠い場合遅翼周りの空気の運動量変化
を見れるならば、飛行機への重力の作用が認知出来、日常世界の常識
「重力の間断なき作用が高空でも成り立つ」が延長出来るのではない
かという主張なんだろうね。
しかし運動量変化など見ることも出来ないだろうし、第一根本的に
何故ある特定の場合に限り揚力が起こるのか?(起こらないこともある)
という疑問は何ら解決しないだろうしNS方程式からも直接的にそれは
出てこないんじゃないのか?
さらに言えば、大気圧という気体特有の現象を無視し過ぎている。
大気圧の翼周りのバランスが崩れるだけでも揚力は発生し得るし
それは大気の運動量を殆ど変えない。何ら外部に仕事をするわけでもない
のでエネルギー消費も0.静力学的安定で重力と釣り合っている。(吸盤の原理)
吸盤の原理で説明される要素をどうして無視するのかがわからない。
613ご冗談でしょう?名無しさん:03/02/25 15:36 ID:???
このスレで議論されていたのは、ベルヌーイの法則という比較的簡単な法則
(だが抽象的)を使って揚力を説明するのは、どうも変だとかそういうもんだった。
確かに密度0の空気に圧力というものが定義できるのかという素朴な疑問など、
流体力学とニュートン力学の間には微妙なギャップがあるので
流体力学の基本的な法則を使った説明がしっくり来ないという人も多いかも知れない。
NS方程式は流体力学とニュートン力学の間のこういったギャップを克服した完全な方程
式とは思えない。(だからだといってボルツマン方程式とかに走るのも同じ結果を招く
だろうね。)
だから流体力学的な考え方捨てて、すべてニュートン力学だけで説明しようというの
も過激でしょう。すべて二項関係だけに還元しちゃう方法がニュートン力学。
ベルヌーイの法則による暗示的説明の否定=作用反作用の法則だけによる説明
は間違いだって気がするね。
614ご冗談でしょう?名無しさん:03/02/27 02:35 ID:???
ある意味デムパ以上に始末が悪い。

>大気圧の翼周りのバランスが崩れるだけでも揚力は発生し得るし
>それは大気の運動量を殆ど変えない。

大気の運動量をほとんど変えずに、どーやって翼周りの圧力バランスを崩すんだ?
飛行機は吸盤でぶら下がっているわけではないぞ。
ダウンウォッシュが生じている現実をどうして無視するのかがわからない。

>ベルヌーイの法則による暗示的説明の否定=作用反作用の法則だけによる説明
>は間違いだって気がするね。

気がするだけだろ。揚力の説明をするのに、連続体近似から離れられないのは何故だ。
空気の微小要素が互いに斥力(応力)を及ぼしあうのが現実だと思っているのか?
615ご冗談でしょう?名無しさん:03/02/27 03:19 ID:???
>>614
ん?ということは圧力バランスが崩れることが揚力発生の要因だということを
認めているみたいな感じだな。
連続体近似による暗示的説明の否定がすぐに空気と飛行機の間の二項関係だけの
説明に結びつけるのが問題なんじゃないのか?
ある意味、飛行機の大部分の荷重は飛行中、下の大気を媒介にして大地が支えてい
るということも出来る。
616ご冗談でしょう?名無しさん:03/02/28 02:16 ID:???
案の定、コマッタちゃんだな。
空気の連続体近似から脱却できない己の愚かしさを棚に上げて、
「作用反作用だけによる説明は間違いだ」と言うあたり、失笑を禁じえないぜ。

>ある意味、飛行機の大部分の荷重は飛行中、下の大気を媒介にして大地が支えてい
>るということも出来る。

出来ねーよ。地表付近を飛ぶことしか考えてないのかよ。

質問だ。
1気圧の無風状態のとき「空気」は止まっているか?
617ご冗談でしょう?名無しさん:03/02/28 03:22 ID:???
>1気圧の無風状態のとき「空気」は止まっているか?
無風状態とはどういうことか、止まっているとはどういうことか
その辺りの定義をはっきりさせないと。
恐らく「空気分子がピュンピュン飛んでいるのが見えないのか?俺は見えるぜ。」
とでもいいたいのだろうが。

作用反作用の法則さえ成立すれば、後は慣性の法則とか成り立つか
成り立たないなんかはど〜でもいい「物理学」でも揚力が発生する
ことが証明できるみたいなことを言っておるようだが、全く似たよ
うな理由で、「下の大気を媒介にして大地が支えている」という
説明は、連続体近似とは関係なく、十分に粒子数が多い質点系でも
言えるだろうね。大地が大気が下に行くのを押さえつけているから
こそ、大気圧が発生し、それが翼の下に作用して、飛行機を支える。
地表付近じゃなければ、大気圧が存在しないとでも言うのか?
618ご冗談でしょう?名無しさん:03/02/28 03:38 ID:jcuQ1JVN
‥‥‥とりあえず、デムパはデムパとして、

過去スレの繰り返しになるかも知れないが、
数式を使わずにエレガントに、揚力が生じる理由を、お互い披露してみないか?
619ご冗談でしょう?名無しさん:03/02/28 04:20 ID:CV7Kf9cL
>>603 >デムパやkittyにはマジレスしてはダメだよ。いくら説明してやっても全く理解しないし
お前が意味不明だろ?あのなあ!熱力の局所平衡の仮定を持ち出したのはのはなあ!
お前ら流体屋が、やってるのは連続体近似!(近似だろ!)どの程度!局所平衡
の仮定、仮定が成り立ってのか?わかんないと!方程式から来た誤差か?
それとも、近似から来た誤差か?わかんないじゃないの!ってのがおれの疑問な訳。
本当!初学者だな!あと、俺流体力学、だいたいニュートン力学みたいに捉えてる
が悪いか?特別な発想なんてないだろ!ベクトル解析使ったくらいだ!
だいたいさ、どの程度局所平衡の仮定が成り立つかも評価出来ないで!
長期予測しようと言うのが間違ってるよ!解とのズレが、カオスとか
なんたら言う前にもっと根本的な誤差要因あるだろ!
流体屋でなくても持つ当然の疑問だと思うが?本当しあわせだな!
人が分かって無いと決めつけれる文盲のアホは!
それが、いやなら全部ボルツマン方程式で解け!
流体力学みたいな近似つかあずに!
620ご冗談でしょう?名無しさん:03/02/28 04:53 ID:???
>流体力学みたいな近似
近似は時には実際以上の情報を含むこともあるから馬鹿には出来ない。
ま、流体力学は、解の不存在を拡大解釈する傾向があるかも知れないが。
これは数学的に偏り過ぎているだけの話
621ボルツマン:03/02/28 04:58 ID:???
私の名の付いた方程式も所詮は近似ですが何か?
622ご冗談でしょう?名無しさん:03/02/28 05:12 ID:LxIHRLms
>>621 お前みたいのを、糞厨房と言うんだ!
   ボルツマン法定式からお前は、流体力学の方程式も導出できんのか?
   じゅあ、もっと上級の方程式書いてみろ!
   じゃあ、お前は、第一原理計算してろ!経路積文でも使って
   ラグラジアンでも、づっと考えてな!
623ご冗談でしょう?名無しさん:03/02/28 05:13 ID:???
今宵はニギヤカだな。試験が終わったのか (w

>>617
「発生した圧力が伝播する」とゆーのは連続体近似だ罠。質点系で考えているのに
オモイッキリ連続体近似になってるので、ロジックは破綻。って、わかるか?
いいかげんマヌケさを露呈し続けるのを止めて、もっと勉強してきなさい。

>>618
「お互い」とは?
流体力学によるエレガントな説明はこれまで皆無だな。
デムパのノイズはスゴイが。

>>619
吉野家モドキみたいだが、まぁ、言いたいことはわからんでもない…
でも「揚力が発生する理由はよく解らんが、翼回りに作用する力を詳しく求めたい」
とゆー立場では、流体力学は威力を発揮するぞ。で、あまりに強力なので、連続体
近似が現実だと思い込んで理解しようとするからダメなんだが。
近似が実際以上とか言ってる>>620もそのクチか。

>>621
ボルツマンのお墓を探すのに時間がかかり、ふと気がつくと辺りは薄暗くなってきて…
さらにあの厳めしいお顔。ウィーンでちょっと怖い思いをしますた。
624ご冗談でしょう?名無しさん:03/02/28 05:15 ID:wOxlXAxg
     ∧_∧∩ / ̄ ̄ ̄ ̄ ̄ ̄ ̄ ̄ ̄ ̄ ̄
    ( ´∀`)/<先生!こんなのがありました!
 _ / /   /   \___________
\⊂ノ ̄ ̄ ̄ ̄\
 ||\        \
 ||\|| ̄ ̄ ̄ ̄ ̄||
 ||  || ̄ ̄ ̄ ̄ ̄||
http://saitama.gasuki.com/kaorin/
625ご冗談でしょう?名無しさん:03/02/28 05:27 ID:660M8tcx
>>623 俺が、このスレで欲しかったのは、こうゆうレスなのだ!
   安心して、眠る。揚力は、よく、流体屋で議論して
   解決してくれ!
626ご冗談でしょう?名無しさん:03/02/28 05:56 ID:E2ZzcGmT
>「発生した圧力が伝播する」とゆーのは連続体近似だ罠。
どこにも発生したなんて書いてないじゃん
さらに言えば、大地の存在は大地の剛体的存在と読み替えるべし。
627ご冗談でしょう?名無しさん:03/02/28 06:11 ID:E2ZzcGmT
>>626に追加させてもらえば、>>617は飛行機がほぼ一定速度で比較的
長時間水平飛行できることを説明するのに、例えそれを質点系で捉え
質点を構成する空気粒子の間に相互作用を考えなくても、大地を構成
する質点が強い相互作用、つまり剛性を持っていることが決定的に
重要だと言っており、空気の圧力に相当する物理量の定義とかをして
いるわけでもないしそれらが伝播するなどとも書いていない。
628ご冗談でしょう?名無しさん:03/02/28 06:13 ID:UZIX2Mn+
>>626 623さんが、正しい事いっていようがそうでなかろうが
   私のレスに期待どうりに、反応が返ってくれば、このレスには
   、用事は無い。他の事は、私にとっては、どうでもよいのだ!
   揚力考えとくれ!ガン枯れ!
629ご冗談でしょう?名無しさん:03/02/28 06:23 ID:E2ZzcGmT
ま、>>617は大地の相対的剛性条件の他にも
実飛行時間に対し空気の経路空間体積が圧倒的に大きいということ
と大地の質量が圧倒的に大きいということも追加すべきだ。

何かある一定時間の間に飛んでいると解釈されるか、否かは
結局は方程式のパラメータ比の問題なんじゃないかという気
がするな。振動の過渡現象解析に似た感じ。
630ご冗談でしょう?名無しさん:03/02/28 07:38 ID:Rlis6CU+
趣味でサンプルムービーのサイト作りました
http://homepage3.nifty.com/digikei/sample/sample.html
631ご冗談でしょう?名無しさん:03/02/28 15:50 ID:???
質点系で考えることと、連続体で考えることの差というよりも、「有限」の立場で考えるのか
「無限性を否定しない」立場で考えるかの違いだと思うけどな。ここでの不毛な論争は。
「無限性を否定しない」立場は、大抵局所有限の立場で考える。その局所の取り方でまた
議論が分かれる。
飛行機が飛ぶということをどのように捉えるかで、局所有限の領域の取り方が変わる。
「長い」時間飛ぶ場合は、空気が力を媒介して大地に伝えているという考え方は間違いじゃない。
空気の運動量変化だけを見る立場では非常に短い時間空気との相互作用で重力による落下運動からずれる現象も「飛ぶ」に含めることになる。
「飛ぶ」ってどういうことなの、まずこれを明確にしなければ>ALL
632ご冗談でしょう?名無しさん:03/03/01 01:46 ID:8MmmiVmh
確かに、飛行機が、地球を半周するような長いフライトをすれば、
大地の影響があることは認める。ただ、そこまでの話をすると、
コリオリ力とか、そういうのまで含めなきゃいけないから、そりゃ果たして、
飛行機の飛行の問題なんか?話が違うだろ?と思ってしまう。

上下非対称物体が流れの中に置かれてれば、流れの方向と垂直方向の、
何らかの力を受けるだろう。別にこれはNS方程式とか知らなくても、
直感的に理解できる。んで、その垂直方向成分の力が、

流れに重力場が働いている場合→発生する
    〃  働いてない場合→(?)

の(?)で意見が割れてるわけだが、どう考えても、漏れには、
『発生する』としか思えん。
633ご冗談でしょう?名無しさん:03/03/01 02:11 ID:8MmmiVmh
@質点の力学 (まあ、楽勝だね)
A質点系の力学 (+あらら、物体の数が増えちゃったよ)
B剛体の力学 (+おっと、回転しはじめた)
C弾性体の力学 (+ぐにゃぐにゃ!変形してるうぅ)
D非圧縮性完全流体の力学 (+変形した上、バラバラになっちゃったよ)
E非圧縮粘性流体の力学 (+ねばねばまで加わった)
F圧縮性粘性流体の力学 (+のびたり縮んだり、不連続面まで!もうわけ分からん!)

の、Fが確立されているのに、Fは近似だ近似だ!こんなのダメだ!といって、
Fに比べればまさに厨学生レベルの、Aで揚力を説明しようという。

まあ、基礎からやろうという意欲は認めるが、そこまでやるんだったら、
アリストテレスまで戻ってやってみては?物体は地水火風の四大元素から成る、
‥っていう、あのあたりから。
634ご冗談でしょう?名無しさん:03/03/01 02:23 ID:aUiKOs/9
気合いだ、気合い。
635ご冗談でしょう?名無しさん:03/03/01 03:16 ID:???
俺がまだ学生だった頃、兵隊経験のあるじいさん先生が板。
天候無視、根性で飛び立った飛行機は、堕ちた、そうだ。
636ご冗談でしょう?名無しさん:03/03/01 04:15 ID:xnOBpftz
結局わかないのに、ヤツは飛んでるの?
637ご冗談でしょう?名無しさん:03/03/01 04:17 ID:6mnVi9vr
凧揚げと、何ら変わりの無い原理で飛んでいる。
638ご冗談でしょう?名無しさん:03/03/01 04:25 ID:/veWuATb
>>633 とんだ誤爆だな!みてておもろいなあ。
   俺は、628以降発言してないが!
   気持ちは、分かるが!流体法定式は、ニュートンレベルだな!
   お前の言うAを理解する能力ありゃFも同じに見えて当然!
   お前には、見えんのか?(いいたい事は分かるぞ!)
   別に、俺は揚力の事で発言した訳じゃないし。
   ついで、言えば揚力を明確に説明デキン点ではお前も同じ
   だろ?まあ、揚力がんがれ!
639638:03/03/01 04:50 ID:sPy4IYOs
誤解の無い様に言っとくが、俺は、揚力の説明に、このスレで指一本
触れてないが?そのくらい読み取れ!
お前の、言う所のAで、説明出来る奴がいるかどうかは知らんが?
説明出来たらそれでもOKだ!出来るかどうかは知らんが?
それが、物理だからしょうがあるまい。
640ご冗談でしょう?名無しさん:03/03/01 04:56 ID:???
おぉ、真打登場か。

>>633
質点系では粘性を考えられないのかな。
流体力学だけやけに分類が詳しいのは、ご愛嬌ってところか。
で、その「確立」された流体力学って意味は
「流体の物理量を求めるための方程式が与えられた」とゆーことだろ。
確かにある条件で方程式を解けば揚力が発生していることは示せる。
だがここで問題にしているのは、揚力の発生する理由。
計算して力を求めただけでは揚力を説明したことにはなってないだろ。
わかるか?

>>638
そんなに煽っちゃダメだよ。
オレは633が揚力のエレガントな説明を披露してくれるのを
ちょっとだけ期待している。
641ご冗談でしょう?名無しさん:03/03/01 05:45 ID:4/a3F3m0
>>640
漏れはエレガントどころか、泥臭い説明しかできないが、

@水平な流れの中に、上下非対称な物体が置かれた。
Aすると、その周りの流れも、物体の影響で上下非対称になる。
B流体、上下方向に運動量発生。(上流ではそんなものなかった)
Cこの上下方向運動量はどこから?犯人は?‥‥もしかして物体?(しかいない罠)
D流体、物体から上下方向に押されてることが判明。
E作用反作用の法則で、物体も、流体から上下方向に(反対向きに)
 押されてることが判明。(→揚力)

くらいしか思いつかん。
この説明だと、別に流体が、連続体でもパチンコ球の集まりでも、
どっちでもいいんだけどな。
でも、わざわざ精度の悪いパチンコ球で考える必要もないだろうと思う。

あと説得が難しいのは、特定の形(翼形)の場合、異常な大きさの、
上下方向力が発生すること。ただ、こりゃ定量的な話なんで、
これを説明するには、実験してみせるか、NS方程式で数値的に当たってみるほか
ないと思う。
642ご冗談でしょう?名無しさん:03/03/01 05:52 ID:???
>>640
>ある条件で方程式を解けば揚力が発生していることは示せる
流体屋だったら眉唾に思うだろな。
真の流体屋だったら、方程式がまともに計算できたり解けたりする
代物じゃないことは十分知ってる筈。

もしまともにぶち当たる場合でも、「ある時間、揚力が発生し
ている確率は0ではない」ということを命題とするだろうね。
同様に質点系でも、膨大な数を相手にするんだったら、同様の
命題になる。こういった問題は初期条件や時間スケールよって変
わってくる筈。
揚力を説明するのには、少なくとも時間スケールを固定する必要
があることは、流体の発想が無ければ出にくいことかも知れない。
時間スケールを変えると、揚力は発生しない(0に漸近)する
ことも有り得る。
それとは逆に、質点の集合体(非連続体近似)で考えないと
初期条件に拠らずどの場合でも揚力が発生すると即断しやすい。
蛇足だがいずれの場合も揚力とは、重力に相当するような大
きな力を意味しているわけで、微小な空気分子が翼に当たった時
の反動が翼に与えている力を無視しているわけではないよ。
643641:03/03/01 05:52 ID:4/a3F3m0
ああ、あと補足ね。
厳密に言うと、上下非対称でも、前後対称だと、流体の運動量が中間地点でそっくり
そのまま回収されて、物体に上下方向の力が働かないという、パラドックスが起こる
ような気がするが、実際には、

 ⌒ ←のような上下非対称&前後対称物体に、前から風を当てると、揚力は発生する。

これって実は、流れの前後対称性が破れてしまうからで、その原因がいわゆる粘‥(ry
644ご冗談でしょう?名無しさん:03/03/01 06:00 ID:???
Cこの上下方向運動量はどこから?犯人は?‥‥もしかして物体?(しかいない罠)
流体を質点の集合体として捉えてみると、おかしな発想だということが
わかる。
どうして質点に個性を考えないのか?質点同士の相互作用を考えないのか?
ま、質点が併進運動をしているとみなせるぐらい、流速が早い場合はこの
推論も妥当性が無いわけじゃないが。
645641:03/03/01 06:03 ID:4/a3F3m0
あと余談だけど、
なんでみんなそんなに流体力学を眼の敵にするの?。・゚・(ノД`)・゚・。

もしかして古臭い学問の(ような気がする)割りに単位とりにくかったり、
式がごちゃごちゃしてるから?

でもね、あのごちゃごちゃした奇怪な式も、もとはありふれた、

@質量保存則
A運動量保存則
Bエネルギー保存則

から誘導してるんだよ。怪しいもんじゃないっての。
646ご冗談でしょう?名無しさん:03/03/01 06:08 ID:4/a3F3m0
>>644
いや、別におかしくないでしょ?

そりゃ、質点には個性があるし、例えばビンの中に入れた空気が、
いきなりみんないっせいに片側に偏るってこともあるかも知れない。
ただ、そんな怪奇現象を見るためには、宇宙の終わりまでビンを見てないと
ダメだよ。
647ご冗談でしょう?名無しさん:03/03/01 06:11 ID:xy77mOtf
趣味でエロサンプルムービー専門のサイトはじめました。
http://homepage3.nifty.com/digikei/sample/sample.html
気合入れてどんどん更新していきます
648ご冗談でしょう?名無しさん:03/03/01 06:32 ID:4/a3F3m0
ああ、あとね、
ID???さんも、漏れみたいなヤシにも分かるように、別にエレガントでなく、
泥臭い言葉で、揚力の説明してくれんかのう?
ID???だと、誰が誰だか分からんので、人違いだったら真に申し訳ないが。

ただ、質点個性論はカンベンな。あれを使ったらほんと何でもありになっちゃうから。
(突然群れ集まってきた空気分子が、ジャンボジェットを垂直離陸させて、しばらく
フワフワ浮かせた後、突然離れて行って、ジャンボジェット墜落、とかね)

あと、質点で流体をやりたいんだったら、ボルツマン勉強してみては?
良くは知らんのだが、希薄気体って、日常の常識と違う、
いろいろ不思議な現象が起こって面白いらしいよ。
ただ数学は難しいけどね。(漏れは式見ただけで挫折)
649ご冗談でしょう?名無しさん:03/03/01 06:35 ID:???
>>646
644だけど
というかね、流体が物質と相互作用をした結果が流体の局所的運動量変化で
あるというのが即断だといいたいのね。流体を構成する質点どうしも
相互作用するわけでしょう。既に障害物に衝突した質点が、未だ衝突してい
ない上流の質点に、相互作用を通して間接的に力(障害物情報)を伝えてし
まうってこともあるわけで、
上流から来る質点がすでに衝突した質点との相互作用の結果で、障害物が
あることを知らないのに障害物を避けるような経路を辿ることが十分有り
得るわけね。
t−>∞で障害物と何ら相互作用しないってことも考えられる。
そこに流体の面白さがあると思うんだけどね。
質点どうしの相互作用を考えないのは、流体の考え方じゃないとも
いえる。
650ご冗談でしょう?名無しさん:03/03/01 06:45 ID:4/a3F3m0
>既に障害物に衝突した質点が、未だ衝突していない上流の質点に、
>相互作用を通して間接的に力(障害物情報)を伝えてしまうってこともあるわけで、
>上流から来る質点がすでに衝突した質点との相互作用の結果で、障害物が
>あることを知らないのに障害物を避けるような経路を辿ることが十分有り
>得るわけね。t−>∞で障害物と何ら相互作用しないってことも考えられる。

すいません。漏れが間違ってました。<(_ _)>
649さんが正しいです。そこまで考えが及びませんでした。
負けを認めます。生意気なことを言ってどうもすいませんでした。
どうかカンベンして下さい。ほんとすいませんでした。
では、この辺で寝させてもらいます。<(_ _)>
651ご冗談でしょう?名無しさん:03/03/01 06:58 ID:???
>>650

649だけど
君は流体粒子か?どうせ一度しか衝突しないのに、すぐに勝負や善悪の
問題にしたり、生意気言うなあやまれとか自分の非を問われていると
解釈したりしてすぐに離れていく。
それでいて流体粒子って奴は、自分がやられたことを同じことを他の
粒子に勝手にやり返すから始末が悪いと言われるんだな。
漏れもそうであることは否定できないが(W
652ご冗談でしょう?名無しさん:03/03/01 07:11 ID:K6ccdvZV
>>642
>ある条件で方程式を解けば揚力が発生していることは示せる
これ、数値計算の事いってんじゅあ無いの?
誰も、紙とえんぴつで解けるとは思って無いと思うが?
何度も言うが俺は流体屋じゃないからよく分からん。
>>645 別に変な式とは思って無いが?何度もいうようだが!
あと、ボルツマン法定指揮、別に希薄気体の為だけに用意された
物では無い。俺のレス見た?
あと、641さん、説明ありがとう!分かったような気はする。
(よく、分からんですまん)
別に、俺は、繰り返すが!揚力はどうでもいいのだが!
揚力ガン枯れ!
揚力ガン枯れ!
653ご冗談でしょう?名無しさん:03/03/01 07:43 ID:NP1paZEN
>>650 まあ、気にせんでいいんじゃない?
    考えてるだけましだ。
    少なくとも、分かったような気はした。
    649さん、のをよむまで。
    揚力ガン枯れ!
    
    
654653:03/03/01 08:29 ID:NP1paZEN
>>649 >流体が物質と相互作用をした結果が流体の局所的運動量変化で
ある。何度の言うが流体屋では無いが!
ちょっと遊んでみますか。逆に、局所的運動量変化無しに、定常的な
揚力が生じる事あります?
655653:03/03/01 08:54 ID:NP1paZEN
>>650 確かに649さんの言うようあきらめ早すぎ。
   もう一度、頭の中にある。もやもやしたもの整理して。
   649さんと話してみたら?649さんなんて、たちのいい方
   だよ!もう、俺もネムルから、揚力ガン枯れ!
656ご冗談でしょう?名無しさん:03/03/01 17:59 ID:???
649だけど、まだ伸びてたのか.失礼。
>>654
先ず、定常的な揚力というもの自体が怪しいわけで、それすら存在しないのでは
ないかということがまずある。揚力によって重力が打ち消された状態
で飛行機が昇降運動をしている時などは、局所的運動量変化を間違いなく空気に
与えている思われ。完全に水平飛行している場合でも完全に0ということは
有り得ないと思うけど、どのくらいの運動量変化になっているのかは答えに
相当幅が出ると思われ。
空気の粘性の影響が重要で、それによって答えは影響を受けるのは間違いない
でしょう。次世代の極超音速旅客機構想では、マッハ6〜7で超高空を飛ぶらしい
けれども、迎え角は30度とかの凄い急角度を取るらしい。この飛び方は
運動量変化の反動を利用して重力を打ち消す方法だと思う。
657ラッキーアイテム:03/03/02 00:29 ID:TqK2jBca
658ご冗談でしょう?名無しさん:03/03/02 22:59 ID:???
あれぇ、揚力の流体力学的考察はもう終わりなの。失速しちゃいました?
このままでは「流体力学では揚力は理解できない」って結論だけど…

>>641
>あと説得が難しいのは、特定の形(翼形)の場合、異常な大きさの、
>上下方向力が発生すること。ただ、こりゃ定量的な話なんで、
>これを説明するには、実験してみせるか、NS方程式で数値的に当たってみるほか
>ないと思う。

スレの最初からの問題が正しくこれなので「流体力学では計算して示せるだけ」
では済まされないでしょ。「何で飛べるのか」の答えにはなっていない。
もう少しガンガッて続けてほしいぞ。
ちゃんとしたレスが増えると変なデムパはこなくなるので良いカンジだし。

>>656
>この飛び方は
>運動量変化の反動を利用して重力を打ち消す方法だと思う。

他の飛び方では運動量変化以外に揚力を発生させるファクターがあるような
書き方なので、詳しい説明をキボンぬ。
659ご冗談でしょう?名無しさん:03/03/03 03:01 ID:???
「理解出来る」ということの定義の問題じゃないか?
流体だとか、質点系であるとかの違いの問題も大事だが(連続体モデル
と離散モデルの間にある数学的な「深いレベルの」相違)

動力学的な考察なら良いが、静力学的な考察ではダメ(圧力差ではダメ)
ということを巡る論争?がテーマなのではないかい?
660ご冗談でしょう?名無しさん:03/03/03 04:09 ID:???
>>659
工学的には静力学的考察でOK。同時に動力学的な考察も必要だけどそれは
音速に近いかそれを超える高速飛行の場合で低速の場合は静力学的考察が
絶対必要。
ただ物理学の場合、静力学的考察は少なくとも19世紀半ば以降はタブー
に近い。現代物理学は静力学的考察を意図的に無視することで成立して
きた。静力学的現象は、2体問題に還元して行われる動力学的考察を通
して「推測」されるべき「仮説」であり、最初からそれを本質であると
仮定して考察してはならないことになった。そこには不確定性原理に似た
事情が伴うからである。静力学的存在の物に静力学的存在を当て嵌めて
観測した瞬間それは静力学的挙動をしなくなるからである。動力学的実在
は細分・抽出というプロセスに対してその挙動を大きく変えないのに対し
静力学的実在は、細分抽出した瞬間別の挙動になるという観測の問題に
伴う本質は現代物理学ではタブ―とされている。現代物理学は物の性質は
抽出・細分の操作に対して不変であることが前提とされる。
661ご冗談でしょう?名無しさん:03/03/03 21:21 ID:tbfyEWPP

水平飛行では揚力を一定(=重力)にしないといけないから、
スピードに応じて翼の迎角を調整するんだよね。
で、スピードが速くなったにも関わらず抗力が小さくなることが
あるようだけど、つまりスピードが上がって方が推力が小さくて
すむってことになるけど、こんなことってあるの?
662ご冗談でしょう?名無しさん:03/03/03 22:07 ID:FeQax4Wr
すでに出てるかもしれないけど、揚力はベルヌーイの式によって導かれるものではなく、作用反作用の式によって導かれるそうな。
663ご冗談でしょう?名無しさん:03/03/04 00:01 ID:???
>>659
>動力学的な考察なら良いが、静力学的な考察ではダメ(圧力差ではダメ)

「翼の上下面に圧力差があるので揚力が生じる」では揚力を“理解”した
ことにはならんだろ。単に力の引き算ができただけ。ちゃんと流体力学的に
圧力差ができる理由を言わなきゃならない。なのにこれまでに出てきたレス
では「計算してみないとわからない」ってのばかり。(もちろんデムパは除外だ)
流体力学による説明で何か新しい切り口は無いのか?
664ご冗談でしょう?名無しさん:03/03/04 00:07 ID:???
>>662
おもいっきり既出だな。
朝日新聞にも揚力を運動量変化で説明をした記事が載っていたが、何故かウェブの
バックナンバーからは削除されている。(ののちゃんの不思議玉手箱)
ギョーカイから圧力がかかったのか (w
665ご冗談でしょう?名無しさん:03/03/04 01:38 ID:CO7fkxt4
>>663
641氏の投稿で、途中まではうまく説明がされていますが、最後の、
『特定の形(翼形)の場合、異常な大きさの、上下方向力が発生すること。』
これが正しくないので、皆さんの誤解を生んでしまったものと思われます。

ここを正確に説明すれば、
『特定の形(翼形)の場合、異常な大きさの、L/Dが発生すること。』
となります。異常な大きさのL/Dが生まれる原因は、簡単に言ってしまえば、
『翼形というものが、非常に尖った後縁を持つために、非常にDの値が小さくなる』
ことに尽きます。
このことは、初心者向けの流体力学の教科書(例えばシャピロの教科書)等に
しばしば描かれる、同じDの値を持つ、円柱と層流翼とを比較した図をみれば、
一目で理解できると思います。これを見ると、円柱の断面積の、たぶん数百倍
〜数千倍の断面積の層流翼形が、同じDになるというのに、本当に驚かれるの
ではないかと思います。
666ご冗談でしょう?名無しさん:03/03/04 18:41 ID:???
>>2 
>今はネオ空洞説とネオ循環説待ちかな。
ネオむぎ茶の復活を待つしかないか
667ご冗談でしょう?名無しさん:03/03/05 03:00 ID:???
>>665
>『翼形というものが、非常に尖った後縁を持つために、非常にDの値が小さくなる』
>ことに尽きます。

尽きないと思われます。
ここでは、Lが生じる理由を、直接説明するべき
ではないかと思います。
668ご冗談でしょう?名無しさん:03/03/05 04:03 ID:???
流体力学的な説明というのを、連続体・あるいは有限個の粒子の初期値問題
と捉えた場合、勝手に初期値を選んだ場合、何故、「高い」確率で揚力が発
生するようになるのか?と捉るんだったらまだこのことに関しては余り議論
されてない。それは恐らく流体の方程式を解いて揚力が維持されてもおかし
くないぞ、ということを示すことよりずっと難しい話になる。
というのも、まずこれは飛行機の加速の仕方も重要なファクターとして自由
度として入ってくるし、人間の操縦という意志も入ってくる問題だから。

空洞説みたいな説?がこのスレの最初にあったけれども、あれが揚力が維持
されることの一つの直感的な説明だとしても、何故加速中に揚力が発生して
いくのかということに関しては何も言っていない。(空洞説が正しいか正しく
ないかは別としてベルヌーイの法則による説明も、揚力が維持されることの説
明にはなっているけど、圧力の分布が維持される理由がわかりにくいので空洞
説に較べても一層わかりにくい。とおいらは思っている。でもプロペラでも
スクリューでもキャビテーション(空洞)という現象が付き物なので、同じ
原理である筈の飛行機やヘリの翼も全くキャビテーションに無縁というわけでも
あるまい)
669ご冗談でしょう?名無しさん:03/03/05 23:44 ID:???
>>668

揚力を流体力学的に説明するのはむずかしいってことかな。
変な条件を考えずに定常状態の風洞実験を考えてくれれば
良いんだが。

キャビティーって「真空」じゃないだろ。水蒸気の希薄(?)気体で
満たされている。周りの水と比べる場合には無視しても良いと思うが。
で、飛行機やヘリの翼には始めから気体が接触しているわけで、
キャビテーションとは無縁だな。
さらに翼周りの流線を見れば「空洞」など無いのは明らかだし、
ベルヌーイの定理は適用できないし。他の説明はないの?
670ご冗談でしょう?名無しさん:03/03/06 02:49 ID:???
>さらに翼周りの流線を見れば「空洞」など無いのは明らかだし
見ただけで明らかだと決め付ける態度は科学的とは言えませんね。
それが科学的だというなら、次の言明も科学的と賞賛されるでしょう。
「トイレ逝った後手を洗わなかった。お尻を拭くときちょっとついちゃった。
壁に擦りつけてごまかした。見た目にはまったく汚れていない。したがって
指先には大腸菌はまったく付いていない」
671665:03/03/06 02:55 ID:4MI8Evtv
>>667さん、
641氏の説明で、Lが発生する理由は既に説明されていると思います。
私は、それに加えてDが小さくなることを説明し、この2つを合わせて、
翼型というものがなぜL/Dが大きくなるかについて、みなさんに説明いたしました。
もし論理の誤謬があれば、どこがそうなのか、具体的にご指摘いただきたくお願いたします。

またよろしければ、あなたの考えるLが発生する理由を、お聞かせ願えますか?
672ご冗談でしょう?名無しさん:03/03/06 04:04 ID:7bUQc/n3
単純にだな。
F1のウイングはなんのために付いているのかを考えよう。
ウイングが空気を上に流す事により、車体はその分下に押さえられる。
だからあれだけのスピードでもグリップを得る事ができるわけだ。

空飛ぶ物体にウイングを逆の形に取り付ければ、上向きの力が得られるのは自明。
つまり作用反作用。
673ご冗談でしょう?名無しさん:03/03/06 05:43 ID:???
>>672
単純な頭だね。
あの程度のウィングで
空気の運動量を変えて
車体の過重の半分以上が
かかっているタイヤの摩
擦係数を左右する位の揚力
を打ち消せるんだったら
ほぼ同じかそれ以上の水平成分の
空気抵抗を受けるはず。
それにはエンジンを大きくして
パワーを高め、ガスも余分に積まなければ
ならず、ウィングの分の過重増加による
性能悪化も含めると運動性能には
ほとんど寄与しないのでは?
実際にはそのようなことが概して無いから
ウィングを付けているわけだね。
674ご冗談でしょう?名無しさん:03/03/06 06:09 ID:zbRS5MXZ
>>673
本気で言ってるんじゃないよな?
675ご冗談でしょう?名無しさん:03/03/07 02:25 ID:???
>>671

流体力学的には翼の上下面の圧力差が揚力になっていると思います。
ですから「飛行機はなぜ飛ぶのか」という問いに対しては、その圧力差が
生じる理由を説明すべきだと思います。

>>641>>672にあるような、空気の運動量変化による揚力の説明はこのスレの
始めから出ていて、簡潔な回答を与えています。しかしながらベルヌーイの定理
の誤用による間違った揚力の説明が巷に蔓延っている為か、「揚力は圧力差」と
いう固定概念から離れられず運動量変化の説明が理解されないことが多々ある
ようです(正しく理解するためには熱力学や統計力学の素養も必要ですが)。
そのような場合に逆に圧力差の生じる理由を訊ねると、物理法則を無視した珍説の
オンパレードになります。滑稽です。

揚力を圧力差で理解するための、納得のいく説明を期待しています。
676ご冗談でしょう?名無しさん:03/03/07 07:13 ID:981rTBiA
だいたい風速40m/s(144km/h)で屋根の瓦は数100mも飛ぶわ、看板も飛ぶは、木が途中から折れて倒れるわするんだぜ。
自分がそれをやろうとするとどんなに大変な事か・・・・
考えてみてちょうだい。
空気の力を馬鹿にしちゃいかんよ。

飛行機の速度は台風もびっくりの約270m/s(約1000km/h)。
飛行機が浮いても不思議じゃないでしょ?

計算するか・・・
空気の密度(1.247 kg/m^3)、飛行機の速度(約270m/s)、翼の面積(約500m^2)を考慮して・・・
空気の力は0.5*m*v^2で「約45t/ms^2」
それが500m^2の面積の翼に及ぼす力は、なんと約22500tm/s^2(22500kN)だ。
ちなみに飛行機の重さは約400tです。
数%も利用すれば十分浮けるね。
677-y( ̄Д ̄)。oO○:03/03/07 07:25 ID:SYq8yFGs
エッチなアップローダーですよ
http://homepage3.nifty.com/digikei/ten.html
678676:03/03/07 07:27 ID:981rTBiA
よく考えたら、ちょっとおかしかった。
誰かこの式を発展させてください。
679ご冗談でしょう?名無しさん:03/03/07 17:27 ID:???
>>676
それ言うなら秒速数mで流れる海流に何万トンもあるタンカーを浮かせたまま
放置するとたとえそれが流れに向かって走ってても、いずれは海流と同じ速さで
秒速数センチで流れてしまいますが。瓦が飛ぶのはそれと同じ理屈。
680ご冗談でしょう?名無しさん:03/03/07 18:48 ID:???
>>675
流体力学的に考えた場合は圧力差を原因としなければならない
という考え方こそ固定観念では無いのかな?
空気の流れが、翼に接触しなければ、近接作用力である圧力は
応力として翼に作用することが出来ない。上面で空気の流れが
何らかの原因で翼に接触出来なかったら圧力差は生まれてしまう。

確かに流体の方程式を研究する場合、空気の流れが常に翼に接触
していると考える。しかしそれは飛行機が飛ぶとか飛ばないとか
を説明するというよりも、流れが安定なのか過渡的に変化するの
かを研究するためのもので別問題。
681665,671:03/03/08 00:59 ID:JCDnpofF
>>675
ベルヌーイの定理を使わずに、翼表面の圧力を説明するのは、
難しいですね。私にはちょっと見当がつきません。
なぜベルヌーイの定理を使ってはいけないのか、
よろしければ経緯をお教えいただけますか?

641氏は、運動量保存則の観点から説明しており、これも不同意。
また、ベルヌーイの定理(エネルギー保存則)の使用も不同意、となると、
残っているのは質量保存則くらいですが‥‥?これでどうやって説明しろと?

他者に説明を要求するまえに、せめてあなたが一体、
どういうアプローチを求めているのかくらいは、説明すべきと思います。
682ご冗談でしょう?名無しさん:03/03/08 03:37 ID:YVwF5Kyg
>>681
おれは>>675じゃないけど。
どういうアプローチでもいいから、聞いてみたい。
昔、大学の教授が言ってたけど・・・
「飛行機の翼の上と下では空気の流れの速さが違います。上のほうが同じ時間に長い距離を移動するから。翼によって上下にわけられた空気は最後に合流しないといけないのでこうなります。」
ほんとか?
なんで合流しないといけないんでしょうか?

ベルヌーイの式を簡単に言うと
速さ+圧力+高さ=const でしょ?
速さが大きくなると圧力が下がるってのはわかるけど・・・

上の教授はこれを自信満々に言ってましたよ。
俺が「とある外人の学者が、飛行機が飛ぶのは作用反作用の効果ですって言ってました」って言ったら、めちゃめちゃムキになって反論してくるので、それ以上はなにも言わなかったんだけど。

おれはこの教授の教えと、その外人の学者の言い分を二つとも知った上で、作用反作用を採用しました。
ベルヌーイの式の説明を信じる人は、それしか教わってないからなんじゃないの?
683ご冗談でしょう?名無しさん:03/03/08 03:42 ID:YVwF5Kyg
大体、圧力差だけで飛行機が浮くか?
翼の上が真空だとしても、たかが知れてると思うんだけど。
684ご冗談でしょう?名無しさん:03/03/08 04:14 ID:???
>>693
時間スケールとか質量分布に対応する空間スケールを適切に選んでそれを
固定して考えないと。
圧力差派も作用反作用則はまったく否定していないでしょ。流体力学の
方程式は簡単の為にそれを仮定しないだけで、流体力学は作用反作用則は
否定しない筈。作用反作用則に明確に矛盾する解が現れたらおそらく実現
されない解として切り捨てられるでしょう。(明確に矛盾することを示す
のがえらく難しいのだが)
作用反作用則が成立するしないに関わらず揚力の存在が示せれば
ベターと考えている人は居るみたいだけど。

同様に作用反作用だけで説明したい派も圧力差の存在を否定していない。
この人達も重力の存否に関わらず揚力が存在することを主張している
点が圧力差派と意見が分かれている点。

確か6%程度(地上の気圧差に換算して
150ヘクトパスカル程度の気圧差が500平米の翼の上下に発生してい
れば300tの力が発生している計算が元スレで行われていた。
史上最大の台風(890hp)と、史上最大の高気圧(1060)
の場合の気圧差程度ね。翼の上が真空なんかになったら、恐ろしく小さな
翼で浮くことになる。
685ご冗談でしょう?名無しさん:03/03/08 05:02 ID:Mf9gfXpK
1気圧って、ニット缶をグニャっとつぶせる程度の力じゃないんですか?
686ご冗談でしょう?名無しさん:03/03/08 05:05 ID:Mf9gfXpK
>>679

>>676は、そのタンカーが動かせる原理で飛行機も浮くと主張してるような気がします。
687ご冗談でしょう?名無しさん:03/03/08 10:46 ID:???
>>682
作用反作用則は,翼と空気が,大きさが等しく,互いに逆向きの力を受けている
ことを説明するが,そういう力が発生する原因については説明しない
ので,別に説明を探す必要があると思う。

ちなみに,上下の気流の同時合流について,サイトを探すと,たとえば
ttp://www002.upp.so-net.ne.jp/a-cubed/flight/chapter2.html#sec2.1
とかがありました。
688ご冗談でしょう?名無しさん:03/03/08 20:16 ID:HIFPNKOX
>>687

>そういう力が発生する原因については説明しない

そんな簡単な事がほんとにわからないのか?
数字を扱うのは得意でも、現象を頭でシュミレーションする力がないんじゃないのか?
晴れた日には洗濯物がよく乾く、ってのと同じくらい当たり前の事だと思うんだけど。
それともそんな事でも数字で証明しないと納得できない?
689ご冗談でしょう?名無しさん:03/03/08 21:37 ID:XYniN4be
一気圧の空気って空気を構成する
どのくらいの頻度?で分子同士がぶつかり合いますか?
飛行機を考える際にぶつかり合わないと近似しても問題ないのでしょうか?
690ご冗談でしょう?名無しさん:03/03/08 22:48 ID:1glX7uvn
ID???

の人は、全員同じ人とみなしてよい?
691ご冗談でしょう?名無しさん:03/03/08 23:41 ID:???
このスレは極端な考え方をする奴が多いみたいだ。
ID:???のレスは同一人物によって為されたものが存在する
というのはOKだとしてもすべてが同一人物によって
為されたと考えるのはかなり極端な考え方じゃないか?
ちなみに、このスレで議論されている、作用反作用則と
流体力学の基本法則は、独立に近い概念だということは
もっと意識したほうがイイ。流体の基本方程式は、流体を構成
する物質や(時には力場)が作用・反作用則に従っていようが
矛盾するものじゃない。だけど慣性の法則の法則は出てくる。
慣性の法則と重力の存在だけで揚力がある場合はある程度の時間
維持されることは説明出来そう。
この場合、作用・反作用則は成立していようがいまいが関係ないし、
成立していても矛盾するわけじゃない。
ただし、何故揚力が存在する場合が有り得るのかについては何も
言えない。
作用・反作用については、この点明快で空気が翼内部に侵入できない
から相互作用が起こらざるを得ず、作用・反作用の法則だけ認めれば
後は単純ということになる。
692ご冗談でしょう?名無しさん:03/03/08 23:42 ID:???
だけど慣性の法則の法則は出てくる

===>

だけど慣性の法則は出てくる

訂正
693ご冗談でしょう?名無しさん:03/03/11 03:46 ID:???
結局、
「翼の揚力はなぜ発生するのか?」
に対する回答は
「空気に下向きの運動量を与えた力積の反作用」
とゆーことになるのかな。
圧力差では未だにマトモな説明を与えられてないようだし。

このスレで最近プチブレークしてるウェブページ>>556もダメだな。
揚力の原因については何も触れていない。その元凶は、循環Γがゼロでない状態を
一様流+循環流としている点。流体力学の教科書には大抵そう書かれていて数学的
にも間違いではないけど、「粘性のために翼の周りに循環流が生じる」と言われても、
直ぐにその物理的根拠はわからん罠。
元々の循環の定義は、流れが閉領域に入出する際に速度ベクトルがどれだけ曲がった
かを表す量なわけで、質量を乗じれば流体の運動量の変化そのものになる。
この話は元スレで既出。何故か件のウェブページにも載ってる (w
一様流の中に平板を斜めに置くときの揚力の説明で、「板の周りに循環流が生じて」云々
とか言われたら「なんでだよ?」って訊きたくなるなるよな。でも流体力学のテキストに答は
載ってない。循環流の考えを止めて、素直に「板に流れが当たって曲がったから」なら
直感的に理解できるだろ。
つまり揚力の説明をするには流体の運動量変化、作用反作用で良いわけだ。
わかるか?
694ご冗談でしょう?名無しさん:03/03/11 04:25 ID:???
>元々の循環の定義は、流れが閉領域に入出する際に速度ベクトルがどれだけ曲がった
>かを表す量なわけで、質量を乗じれば流体の運動量の変化そのものになる

二次元ベクトル場で、なおかつv=rot vが恒等的に成立するベクトル場ならば、
ある曲線に沿う循環は確かにその内部の運動量の和に等しい。
ただ断じて運動量変化ではない。循環が0でないことは、仮にそれが運動量に
一致していたとしても、運動量が変化することを意味しない。
さらに言えば一般に次元は2で考えるべきではないし、第一、vとrot vの間に
は何の関係も無い。循環と運動量の間にははっきりいって何の関係も無い。以上
695ご冗談でしょう?名無しさん:03/03/11 23:49 ID:???
ストークスの定理を知らないのか、運動量がベクトルだってことを知らないのか、
あるいは周回積分がニガテなのかはわからんが、流れが曲がっているのに
「運動量は変化しない」と言われても困っちゃうなぁ。

まぁ、流体力学のテキストでは循環の物理的意味を直接(周回積分のまま)考察する
ことはなくて、数学的に同等な rot v の面積積分に置き換えて説明してるからな。
循環=循環流と思い込んでも仕方ないのか。回転しているボールがカーブするのを
説明するには良いが、「翼の周りをグルグル回る流れが生じるから揚力が発生する」
とゆーのはダメダメだろ。
696ご冗談でしょう?名無しさん:03/03/12 01:15 ID:NMN8gWJZ
>>695
‥‥そっ、そういうことだったのかぁぁぁぁっ!
流力習い始めて○年、式を展開するのは上手になったけど、
理解は全然してなかったよ。ありがとうです。

695は693だよね。お願い質問させて。あのさ、693の迎え角ついた平板の話でね、
圧力分布で揚力を説明しようとすると、翼型よりもかえって平板の方が難しくて、
素直に「板に流れが当たって曲がったから」は禿同なんだが、でもやっぱり、
(運動量変化の流体全領域での積分)=(圧力分布の板全部での積分)
になってて、どっちで説明しても手間の違いだけで、等価じゃないかと思うんだが、
どうよ?漏れ自身、右辺での説明はいくら考えてもできないんだが、
キミだったらできるんじゃないか?と思った。変なこと頼んでたらゴメソ。
697ご冗談でしょう?名無しさん:03/03/12 04:00 ID:???
>>695,696
あたかも同一人物であるがごとく奇妙に噛み合うレスの応酬、恋人の会話のように
楽しいところまことに恐縮ですが、やはり循環の定義に戻ってみて考えても次元が
1次元下のある曲線上の運動量の総和とみなせるのがやっとで、これに物理的意味
を見出すのは非常に難しいと思います。
この量が0でないことと、周辺の流体の運動量が変化したということの間に
直接のつながりを見出すことは出来ません。
誤同人同士仲睦まじい会話で、革命的な理論構築で世界を驚かせる夢にでも
お浸り下さい。但し変な関係にはならないように。ここはネットですから。
698ご冗談でしょう?名無しさん:03/03/12 06:00 ID:???
第一循環はスカラー量です。ベクトルじゃありません。(W
699山崎渉:03/03/13 13:31 ID:???
(^^)
700ご冗談でしょう?名無しさん:03/03/13 19:42 ID:???
>>696
オレの専門は流体力学じゃないんだが、平板周りの流れは円柱周りの流れの等角写像と
かでやるように教わったが、それじゃダメなのか。
あと、
>(運動量変化の流体全領域での積分)=(圧力分布の板全部での積分)
これは「計算の手間が同じ」とゆーことかな。
翼周りの圧力分布が何故あのようになるのか、今のところ明確な説明は無い。
「飛行機はなぜ飛ぶのですか」と訊かれて
「それは計算してみないとわからない」ってホンキで答えてるヤツ。

>>697-698
要するに周回積分が出来ないだけね。
ちょっとウザイから、次の練習問題が解けるまで出てくるなよ (w

問題 x-y平面上の速度場(vx, vy)を考えるとき
1) 流れがx軸方向に一様(vx=v, vy=0)のとき、原点を中心とする半径1の円周上で
   循環を求めよ。vは定数。
2) 流れがy軸を境に角度αだけ曲がるとき、すなわち
    x<0のとき vx=v, vy=0、x>0のとき vx=v cosα, vy=v sinα のとき、原点を中心
   とする半径1の円周上で循環を求めよ。v, αは定数。
3) 中心が原点にある一辺1の正方形の周囲で、上の場合の循環を求めよ。
701ご冗談でしょう?名無しさん:03/03/14 03:37 ID:???
>>700
あとこの問題も追加したほうがいい。
u-v平面で常に流れが「曲がっている」特殊な2つの速度ベクトル場
1)(Vx,Vy)=(x^2-y^2,2xy)
2)(Vx,Vy)=(x/(x^2+y^2),-y/(x^2+y^2))
の循環を半径1の円周上で求めよ。
一方は0で他方は0では無い。この違いはどうして生まれたのか?
1)はストークスの定理が使えるが、2)は使えない。(何故か)
702701:03/03/14 03:38 ID:???
訂正:u-vじゃなくx-y平面ね。
703ご冗談でしょう?名無しさん:03/03/14 06:10 ID:6hrY6cHN
循環があるから翼が浮くっていうより
浮いてる翼まわりに循環があった、という言い方のほうが正しいんじゃないの?
こんなん結果論だろ
ベルヌーイだのクッタジューコだの定理が出て来たが結局こじつけだろうが

翼の上面と下面で何が起こって”浮いて”いるのかって考えたら
上の方が圧力が低いから、
コレしかねーじゃん
704ご冗談でしょう?名無しさん:03/03/14 08:02 ID:???
>>703
浮いている翼のまわりに循環はあるべきであり、実際もそれに近い流れの分布
になるだろう
というのが真相に近いでしょう
705bloom:03/03/14 09:00 ID:wBXreAr7
706ご冗談でしょう?名無しさん:03/03/14 10:17 ID:2Bq1HUKt
なんで、ヘリコプターとやらは胴体が回転しないんだ?
707ご冗談でしょう?名無しさん:03/03/14 13:16 ID:yyM3HSdT
胴体が回転しないように、ケツにもう1個縦のプロペラがある。
708ご冗談でしょう?名無しさん:03/03/14 13:46 ID:???
メインローターの排気を誘導して、しっぽからモーメントを打ち消す向きに
噴射する方式もある
709ご冗談でしょう?名無しさん:03/03/14 14:35 ID:???
循環は流れの「質」を表現する量で、「量」を表す性質は持っていない。
運動量変化は「量」に微妙に影響を受ける量。
710ご冗談でしょう?名無しさん:03/03/14 15:12 ID:6hrY6cHN
翼の上面より下面の空気が活発に動いてるから押し上げられるのです

結局ここに行き着く
これ以外にない
単純な事じゃないか
711ご冗談でしょう?名無しさん:03/03/14 21:35 ID:???
また春厨の季節がやって参りましたぁ。

>「質」を表現する量で、「量」を表す性質は持っていない。

う〜ん、激しい電波を感じるぞ。
>>700の計算(暗算で解けるよな)ができない香具師は循環を語るの禁止だ (w

>>703
コレしかねーじゃん、と言われてもなぁ…
上のほうが圧力が低くなる理由がわかってるなら説明してごらん。

>>710
「空気が活発に動いてる」ねぇ。
温度が高いのかよ、って、熱力も知らないか。
712710:03/03/15 00:17 ID:C/t6I66H
>>711
たまには定理とか数式に頼るのやめて直感で判断したら?
定理に頼るような場面でもないだろう?

まず、なぜ翼は下に落ちないのか?
重力を打ち消すだけの下面からの力を受けている

その力とはどうやって発生するのか?
下面圧力が上面圧力よりも高いから

なんで上面が圧力小さいのよ?
翼が水平方向に移動すると、移動した事によって上面が真空になってしまう
でもそうはさせじと上から空気が流れ込む、翼も引き付けられる
しかし流れ込むにも時間がかかるからどうしてもその領域は空気が薄くなってしまうからて低圧となる

んで圧力が高いって何よ?
空気が頑張っている

んで十分だろ
713710:03/03/15 00:27 ID:C/t6I66H
>>711
熱力って何?聞いた事ない

人の事春廚とかいう技量がオマイにあるのか?
あるんだったらオレより分かりやすく説明してみれ
714711ではないが:03/03/15 01:13 ID:???
>>713
熱力学の略で、ねつりき。
715ご冗談でしょう?名無しさん:03/03/15 01:28 ID:KuPKhsm8
>>712
の上面真空説で思いついたんですが、

 <____  ←進行方向

こういう形の物体を、左の方に動かすと、<の後ろに真空ができようとして、
上の方にぎゅう〜っ!と持ち上げられるのでしょうか?
飛行機の翼よりは空気抵抗は多いだろうけど。
低レベル質問ゴメソ。
716ご冗談でしょう?名無しさん:03/03/15 02:40 ID:???
物理板にいるのに「真空」と書いてみたり、「熱力」を知らなかったり、
自分の妄想に絶大な自信を持っていたり、どこから見ても春厨だ罠。
ちょっとおナマなオマイにはベルチウの称号を差し上げませう (w

>>715
持ち上がるわけ無いよ。変な妄想を真に受けてはダメだよ。

誰か「<の後ろの真空」にハマって息苦しくなっているコドモを
助け出しておやりよ。
717ご冗談でしょう?名無しさん:03/03/15 04:47 ID:VrfDakFR
>>716
レスさんくすこ。そうだよね。持ち上がるわけないよね!
んじゃこんなのは?

<\/> ←進行方向
  ̄ ̄
上の方が経路が長くなってるので、後縁で上下同時合流しようとすると、
上の方が速く流れなければならなくなって、ベルヌーイの定理で圧力が
下がって、上の方にぎゅう〜っ!‥‥と。これは絶対持ち上がるだろ!
718ご冗談でしょう?名無しさん:03/03/15 05:34 ID:???
>717
翼の上面にギザギザを付けたというわけね。持ち上がるでしょうけど
速くは飛べないだろね。速さに揚力は鋭敏に影響を受け、もちろん速ければ
揚力は安定する。(音速より小さい場合だけど)表面を滑らかにして加速を
良くして出来るだけ短い距離で安定した揚力を得てから離陸したいのが普通だから
そういうことはしないんだろうね。
719ご冗談でしょう?名無しさん:03/03/15 08:38 ID:???
妄想の産物、「翼上の真空」や「同時合流」が出てくると
やはりソコハカとなく「春」を感じずにはいられませんね。
元スレとかはもう読めないのかな?

>>717
持ち上がらん罠。
もしオレが空気だったら、前縁の<で右上に流れて御去らば。
\/この凹みに沿って流れるなんざ、真っ平御免よ。

しかし>>675の予言通りだな (w
>圧力差の生じる理由を訊ねると、物理法則を無視した珍説の
>オンパレードになります。滑稽です。
720715,717:03/03/15 14:56 ID:QxtCyHf+
>>718,719 レスありがd。
んじゃまたこんなのは?

<○○○○○> ←進行方向

(各○は、モーターで時計回りに回ってて、
 キャタピラみたいのが巻きつけてある。)

これは絶対浮かぶと思うな〜。
低レベル連続カキコゴメソ。
721712:03/03/15 15:42 ID:C/t6I66H
>>716
また定理か、オレはベルヌーイなんて言葉一言も出してないぜ
それにオレの説明に何の指摘もせずスルーで逃げるなよ
お前こそ春廚じゃねえ?結局何の説明もできてないじゃん

”真空”ってのは分かりやすい説明のために誇張して書いただけ
それにいちいち反応して言葉尻捉えるって、、ほんとレベル低いなあ
722ご冗談でしょう?名無しさん:03/03/15 15:47 ID:0qZhc7e6
水面ぎりぎり、地面ぎりぎりに飛ぶと、揚力が増すらしい。
2倍ぐらいになるとか。
723ご冗談でしょう?名無しさん:03/03/15 15:58 ID:C/t6I66H
ペリカン?
724ご冗談でしょう?名無しさん:03/03/15 16:08 ID:???
>>720
??
浮かぶんじゃなくて、落ちるんじゃないの?
フォークボールと同じ回転の向きだから。
725ご冗談でしょう?名無しさん:03/03/15 16:09 ID:RLUmIrHn
ペリカンって、ガウ攻撃空母だよね。
ザク×3機だったらマジ載ると思う。
ボーイングにもガンオタいるのかな?スレ違いスマソ。
726ご冗談でしょう?名無しさん:03/03/15 20:44 ID:sd6bBVvF
>>724
ttp://www.nec.co.jp/community/ja/edu/galileo/7th_02.html
しかしな〜。フォークボールが曲がるのも、よく分からんな。

ベルヌーイの定理ってのは、流体が、流路が狭くなってたりすると、
連続性を守るために速く流れようとして、圧力の形で蓄えてたエネルギーの
貯金を下ろして運動エネルギー(速度)に変えることだろ。そういう訳で、
速度が速くなってるところは、圧力が低くなると。

しかし、フォークボールは、ボールの回転が流体の加速をアシストしてるので、
べつに圧力の貯金を下ろさなくでもいいんでは?流体は、ボールの摩擦に引き
ずられて加速し、全エネルギー増えてラッキー!と思ってるような‥‥?
727ご冗談でしょう?名無しさん:03/03/15 20:59 ID:CQTMagAL
>>722
鳥人間コンテストで証明されてるね、それ
728ご冗談でしょう?名無しさん:03/03/15 21:06 ID:CQTMagAL
フォークボールは純粋に重力と空気抵抗で落ちるんじゃないの?
むしろこっちの挙動のほうが自然なボールの挙動に近いと思う。
人間の投げたボールは野球で言う直球になるから、それを基準に考えるとフォークボールは変化球になるんだと思う。
物理の神様から見ると、直球のほうが変化球になるんじゃないかな。
729ご冗談でしょう?名無しさん:03/03/15 21:13 ID:CQTMagAL
>>724
>>720の言いたい事は、上面のほうが空気の流れが速いという一点だと思う。
流れが速いほうが圧力が下がるいうベルヌーイ派の意見があるから。
ちなみに俺は、>>720の形は揚力を生まないに一票。
>>717もむしろ落ちるほうに一票。
>>715はどうかな。よくわかりません。
730ご冗談でしょう?名無しさん:03/03/15 22:25 ID:EDG8k7xH
飛行機が浮く原理と、竹とんぼ(ヘリコプター)が飛ぶ原理は同じです。
もし竹とんぼの羽を、断面が長方形の完全な平板で作っても、非効率ながら十分に飛びます。
羽の回転の進行方向側を丸くして、反対側を薄くするのは単に空気の渦を生み出すのを押さえるためです。
なぜ渦が生まれると竹とんぼは飛びにくくなるのか、それは流体力学をやってる人ならわかると思います。
野球の直球を投げる時に、逆回転がより多くかかってるとミットまで失速しにくいのと同じです。
ゴルフボールにいっぱい溝があるのと同じ理由です。
731726:03/03/15 23:01 ID:Zn22vFAD
>しかし、フォークボールは、ボールの回転が流体の加速をアシストしてるので、
>べつに圧力の貯金を下ろさなくでもいいんでは?流体は、ボールの摩擦に引き
>ずられて加速し、全エネルギー増えてラッキー!と思ってるような‥‥?

誰か反論してくれ!実際、カーブやらシュートやらが曲がる以上、
自分でもこの考えはおかしいと感じてるんだが、なぜおかしいのか、
自分では分からないという生殺し状態になってる。
732ご冗談でしょう?名無しさん:03/03/15 23:06 ID:???
   ,.´ / Vヽヽ
    ! i iノノリ)) 〉
    i l l.´ヮ`ノリ <先生!こんなのがありました!
    l く/_只ヽ    
  | ̄ ̄ ̄ ̄ ̄|
http://saitama.gasuki.com/tochigi/
733ご冗談でしょう?名無しさん:03/03/16 00:54 ID:???
圧力差、厨猫まっしぐら (w

>>726
そう、ベルヌーイの定理から離れると揚力の本質が見えてくる。

まず、そのページの流線の図は誤りで、下流側ではボールの回転の影響を受けて
流れが下向きに曲がる。粘性があるからな。真直ぐなのはウソ。社会に貢献してねーぞ>NEC
ボールは流れに対して垂直な方向に運動量を与えたわけだから、その力積とは逆向きの
力を受けることになる。つまり流れに対して垂直な力が作用するからカーブする。

これを*無理*に圧力差で説明しようとすると、適用条件を満足しているのかどーか
アヤフヤなベルヌーイの定理を持ち出さないといけなくなる。
734ご冗談でしょう?名無しさん:03/03/16 02:27 ID:Ka8UNgbk
クッタジューコフスキー?


ぷっ、逝っていいよ( ´,_ゝ`)
735ご冗談でしょう?名無しさん:03/03/16 02:48 ID:???
>>733
でもどうしてボールは、自分が大気に対して平均的に移動していることを
認識したのかな?
ボールが空気を引っ張って、ある方向に運動量を与えてその反動を受けた
(とも言える)というのなら、別に大気が平均的にボールに対して流れて
いる必要性は無い筈。必要性があるのならば、ボールは大気の流速をどのように
して認知したのかを説明しなくてはならない。
それが説明できないというのなら、ボールは飛んでなくても、滑らかな地面の上
で軸を静止させた回転運動でも、同じ現象が発生することを認めなければならな
いはず。この場合力を受ける方向が本質的に不定で、フラフラとカオスのような
運動をする筈。決定論的な力学の基本原理に対する重大な挑戦的現象だ。
「空洞派」の人たちは空気がボール表面の一部に出来た「真空=空洞」
を埋め合わせようと勝手に曲がろうとするが、ボール表面との摩擦の関
係で対称性が崩れて結果としてボールに作用する大気圧のバランスが崩
れていると考えるだろうね。
736ご冗談でしょう?名無しさん:03/03/16 03:00 ID:YLxH6/uu
翼の上下に分かれた空気は同時に合流する?
・・・・・大嘘つき!!
737ご冗談でしょう?名無しさん:03/03/16 03:25 ID:???
NS方程式のみを信奉する工学屋はココには来るなよ。
計算しか出来ない香具師は板違いだぜ (w

>>735
何が言いたいのかわからんが、「静止」している大気中をボールが直線運動すれば、
ボールから見れば一様流だ罠。また流速がゼロの時は、回転しているボールが空気に
与える運動量は等方的。だからボールに作用する合力もゼロで動かない。
乱流になった場合はフラフラするかもな。
738ご冗談でしょう?名無しさん:03/03/16 03:52 ID:???
>>737
なるほろ、ぢゃボールは大気との相対速度を認識出来るわけだな。
静止して回転している時は、周囲の空気を平等に引っ張るので、ト
ータルで与えた運動量はゼロ。
しかし大気の中を直線運動していると、なぜかそうせず与えた運動
量はゼロで無くなり反動で直線運動からズレてしまう。
状況によって、空気とボールの間に働く力が変わるというのなら
どうして分子や原子にも同じことが起こりえると考えないんだ?
もっと言えばなぜ分子や原子のの存在を認めるんだ?
原子や分子の相互作用のように、状況に依存しない力が支配的な
体系が実際の自然であるとするのが、物理学のオシエじゃ
なかったのか?
739ご冗談でしょう?名無しさん:03/03/16 04:37 ID:0SNo+hFE
クッタジューコによると物体まわりに循環流れがあれば物体に揚力が働くという
ならば、直進はせず、仮にその場でボールが超高速回転をすればボールは浮くのか?
740ご冗談でしょう?名無しさん:03/03/16 05:31 ID:???
>>738
さっきから君は何を言っているの?それは釣りなの?
741ご冗談でしょう?名無しさん:03/03/16 05:48 ID:cfILuqtQ
>>739
浮かねえな
742ご冗談でしょう?名無しさん:03/03/16 05:57 ID:cfILuqtQ
このスレにあるレスの中で、馬鹿レスNo1決定戦をやると面白そうだな
743ご冗談でしょう?名無しさん:03/03/16 08:56 ID:LZlE696y
デムパでない方、ID???をやめれ。
そうするとこのスレの本質が見えてくる。
744ご冗談でしょう?名無しさん:03/03/16 16:59 ID:0SNo+hFE
ID???て何よ?
745ご冗談でしょう?名無しさん:03/03/17 00:33 ID:4hMY1g3m
ベルヌーイの定理の、間違った適用が世間に蔓延してるぞっ、
‥‥ってのは、このスレを見てよく分かった気がする。

でもやっぱり、最終的に物体に働く力は、圧力(静圧)の総和になるわけでしょ?

ベルヌーイの定理のほかに、圧力変化を説明できる方法って、何かある?
それともその前の、運動方程式の適用の仕方が間違ってるのかなぁ?
もうわけ分かんなくなってきたよ。。・゚・(ノД`)・゚・。
746ご冗談でしょう?名無しさん:03/03/17 00:52 ID:???
>>739
クッタ・ジューコフスキーの定理は 揚力=流体の密度×流速×循環
それと 循環≠循環流 だぜ。
747ご冗談でしょう?名無しさん:03/03/17 00:53 ID:???
>ベルヌーイの定理のほかに、圧力変化を説明できる方法って、何かある?

連続体近似を止める。
748ご冗談でしょう?名無しさん:03/03/17 01:09 ID:ezjnFsS4
とりあえず、素人に飛行のメカニズム聞かれたら得意げにクッタジューコを説明しとけばいいわけだ
749745:03/03/17 01:09 ID:4hMY1g3m
>>747
いやん!連続体近似させてぇ!連続体‥マンッ、マンセー!

ベルヌーイの定理自体は、もうこれ以上どう直しようもないくらいシンプルな、
エネルギー保存式なので、これから導き出すpが間違ってるとすれば、
これに代入するvの段階で既に間違ってるのではないかと思うのですが、

‥んで、翼型とか、ボールの回転とかを説明するとき、たいていの場合、
運動方程式はちゃんと解いてなくて、「同時合流」とか怪しげな仮定で
vを定性的に想像してるだけなので、たぶん、この辺がおかしいんじゃ
ないかと思うんですけどね。
750ご冗談でしょう?名無しさん:03/03/17 01:26 ID:uIBcx3dh
>>747
何かあればすぐ流体力学が連続体近似をしているって決めてかかるみたい
だが、初期値問題を考えるので無い限り、連続性が深刻に影響してくる
ってことはあまり無いでしょう。
空気を微小な有限個の粒子からなる離散的質点とみなした場合でも、
圧力という概念はポテンシャルという概念で置き換えることが出来る。
この場合のポテンシャルは、粒子の密度から決まる量。有限個の粒子
からなるモデルを考えてもベルの定理に相当するものがエネルギー保
存式という形で出てくるだろうし。
問題の本質は、粒子の行動を緩く規定するエネルギー保存則だけで細かい
粒子の挙動に依存する性質がどうして導き出せるのかってことでしょ。
ベルによる説明に奥歯に物が挟まった感じを受ける人が多いのはそこらへ
んが関係してるんでしょうね。

似た問題に円周を地面の上に立てて、頂上からボールを転がす問題がある。
ボールの位置エネルギーと運動エネルギーの和は初速運動エネルギーと
初期位置エネルギーの和であることは間違いが無いが、ボールがどこ
まで円周の上を転がりながら動き、どこから円周から外れて自由落下運動に
なるのかは、エネルギー保存則だけじゃ説明できない。初速に鋭敏に依存
する初期値問題を考えなくてはならない。

ま、しかし翼の揚力の場合は、粒子間に相互作用を考えても、考えなくても、
空洞派の理論みたいに、前方から飛び込んできた粒子が翼上面で一度衝突して
重力の影響で落下し再衝突する位置が翼の運動によってずっと後ろにずれて
多くは再衝突しない。その為、翼の下で衝突する粒子の撃力が優勢になっているとい
う状況でなんとなく理解出来るピクチャーになっているんじゃないかな?
751745:03/03/17 01:34 ID:KwYkwRzz
>>750
すごくいいんだけど、最後のパラグラフの説明だけだと、715の、

<____  ←進行方向

が浮いてしまうことになる(これがキモチ悪い)ので、もう少し補足願います。
マクロでもミクロでも、翼上面での「加速」という概念がいるんじゃないかなぁ?
外してたらゴメソ。
752ご冗談でしょう?名無しさん:03/03/17 01:55 ID:???
>>750
もっと頑張りましょう。

空気を連続体で近似しない流体力学を是非構築しておくれよ。
翼は空気に対して仕事をしているのに「エネルギーは保存する」と言われてモナー。
ボールがどこで離れるかって、そんなに難しいのか?
空洞派の「理論」かよ。しかも重力の影響だと。F1のウイングは上向きの重力なのか?
753745:03/03/17 02:28 ID:RDgzfH+O
>>752
んっ?翼は空気に対して仕事はしてないでしょ?

その理屈だと、風吹いてる日に、こいのぼりの棒立てたら、
棒は風にエネルギー与えてことになるよね?
プロペラや風車は、空気に対して仕事のやりとりをするけどね。

流れの中に何か邪魔者(断熱材製)があって、その後ろで剥離して、
全圧が下がると、一瞬流れのエネルギーが減ったように錯覚するけど、
それは邪魔者が、圧力→熱という変化を起こしただけで、
流れのエネルギーは変わってない、と思うけど?‥‥‥ていうか、
供給源があるか、どっかに何かの形で持ってくかしない限り、
エネルギーは増えも減りもできない。不可能でつ。
754745:03/03/17 02:39 ID:RDgzfH+O
‥っと、ちょっと言い方がキツくなってしまったよ。

別の例えをすれば、川の中に棒いっぱい立てたら、川の流れの運動エネルギーは
下流にいくほどどんどん増えるのかよ?ってこと。上流に棒いっぱい立てたら、
下流の水車は勢い良く回るのかよ?ってことよ。

棒が立ってるだけじゃダメだ。プロペラでぐりぐりかき回さないと、
川の勢いは決して強くならない。連続カキコスマソ。
755ご冗談でしょう?名無しさん:03/03/17 02:59 ID:???
>>753-754
風洞実験&連続体近似にやられすぎだ (w

飛行機に乗っているときは、フツーは翼が動くんだがな。
翼が通過した後にはダウンウォッシュが生じているが、
どーゆーふーに空気のエネルギーが保存してるんだよ。

棒をいっぱい立てたら、川の勢いは弱くならないのかい?

756745:03/03/17 03:34 ID:rZEjKFLi
>>755
遅くまで付き合ってもらってアリガd。うむ。言葉が足らんかった。

キモチはよく分かるんだが、運動エネルギーは座標系のとり方で全然変わるでしょ?
飛行機に固定した座標系をとると、翼は力を受けながらもピクリとも動いてないので、
周りを流れてる空気と、仕事のやり取りが全く発生しなくなる。飛行機から見ると、
唯一、プロペラ(かジェットエンジンでもいいけど)だけが仕事をする。

一方、止まってる空気の中に、飛行機がどえらい勢いで飛び込んできて、かき回して、
どえらい勢いで去っていく、という見方も当然あるんだが(この場合、飛行機自体が、
動座標系のプロペラに相当する)、ただ、それで考えるのってメチャクチャ大変だし、
結果は動座標系で考えたのと同じになるわけだから、そのままスライドすりゃいいじゃん。
‥というところが言葉足らずですね。

川の勢いは弱くなるよ。???棒が川に仕事するわけじゃないんだから。
くどいけど、棒を立てても、川の勢いは強くならない。
757ご冗談でしょう?名無しさん:03/03/17 08:14 ID:ly/qlu+/
川が棒に仕事をしてるんだね。
同じように空気も翼に仕事をしている。

一つ馬鹿な質問します。
想像力を働かせてください。
笑えるから。

飛行機の羽を一方はいつも通り、もう一方はそれの上下対称につけたとして・・・
胴体を軸に高速で回転させたとしたら、扇風機の変わりになるかな?
758ご冗談でしょう?名無しさん:03/03/17 13:09 ID:FCazuNnT
川が川に仕事をしてるんじゃない?
水流が弱まった分は熱になってるんでしょう?
759ご冗談でしょう?名無しさん:03/03/17 14:00 ID:???
>>755
最近このスレで多少脚光を浴びてきた空洞派の説明でも、エネルギー消費
は別に否定されてないだろうしあくまでも近似の理論なんだろうから、現実
の翼ではそりゃ多少のエネルギー消費はしているんだろうね。つまり翼は大
気に対して仕事をしてしまう。

しかし、どの位のエネルギーを与えているのかが問題になる。
一般に軽いモノを勢い良く動かすよりも、重いモノを
ゆっくり動かしたほうが、力積は同じでも、与えるエネルギーの量は
全然違ってくる。もちろんエネルギー消費は重いモノをゆっくり動かした
ほうが小さい。

空気に下向きの運動量を与えた反動が揚力であるとしても、
消費するエネルギーは小さくて済む。滑空時のように殆どエネルギーを
消費しない水平飛行が可能な理由を、大質量の空気にゆっくりとした
下降気流にするという解釈をすればこれも説明できる。
ただ、ここで問題になるのは現実の空気がそんなに高い粘性を持っている
のか?ということに尽きる。大きな粘性を持っていなければそのような
ことは起こらない。シリンダーに空気が詰められている状態で、ピストン
を引っ張ると、中の空気は膨張せず、最初の体積を保ったままピストン
に引き寄せられて運動量を持ち、ピストン内部には一部真空が出来ている
ということが起こらなければならない。これはもちろん実験結果に合わない
がこれをどうゆーふーに説明する?
760745:03/03/17 23:22 ID:HX/MB2m7
んんっ!いいね〜。川っ!棒っ!
川と棒で、こんなにみなさんレスつけてくれるとは思わんかったよ。

例えばね、もっと別のたとえだと、重い米袋、両手で頭の上に持ち上げてる。
米袋とっても重いと。手がぷるぷる震えると。すごい『力』が手にかかってるから、
手、ぷるぷるする訳ね。でも、じーっとそのままの姿勢で、米袋動かさない限り、
米袋は手に『仕事』しないし、手も米袋に『仕事』していない。

川と棒の例えもこれに近いんだけど、一つ高度になってるのは、棒の後ろで流れが
ちょっと弱くなることね。『仕事』のやりとりがないのに何でこんなことが起こる
かというと、棒のせいで、758さんがいうように、水流が弱まった分が熱になってると。
エネルギーの総量は変わってないけど、運動エネルギーの一部→熱エネルギーへの、
エネルギーの劣化(感覚的な言葉だけど)が起こってると。セルフ解説スマソ。
761ご冗談でしょう?名無しさん:03/03/18 02:13 ID:???
空洞とか真空の好きなキティちゃんは放置でいいかな。
しかし相変わらずの妄想ぶりだなぁ。

>>760
川と棒はどーでもいいよ。
飛行機が通過した後のダウンウォッシュが存在する=空気は運動エネルギーを得ている
にもかかわらず、「空気は仕事をされていない」と主張する根拠を示してね。

ベルヌーイの定理を持ち出すと大変だな (w
762ご冗談でしょう?名無しさん:03/03/18 02:16 ID:9U60IWi+
>>757じゃないけど・・・

扇風機が回ると風を送るよね?
ということは、止まってる扇風機に風を当てれば、扇風機は風の力でまわりだすよね?
これって揚力と同じ原理じゃない?
やっぱり扇風機の羽の前面と後面に圧力差ができてまわるのかな?
763745:03/03/18 03:01 ID:8/p6oFdC
>>761
うーん、飛行機の窓から見た、動座標系では、という意味なんだけど。(756参照)
棒恐縮ですが、そりゃあ池に棒立てて、人間がえいやっと動かせば、当然棒は
湖に仕事してる罠。

それよりも、キミが構想してる、「空気静止、飛行機移動」の飛行力学の理論を
聞いてみたいな。
764745:03/03/18 03:25 ID:8/p6oFdC
あとちょっと、ダウンウオッシュについて、極端な例で補足すると、

飛行機が速度Uで飛んでると。飛行機の窓から見てて、前から空気がUで
流れてきて、翼で直角に真下に向けられ、下方に速度Uで流れてくとしよう。
飛行機から見たら、空気の運動エネルギーは保存されてるように見えるよね。

ただ、静止系から見ると、飛行機の通過によって、止まってた空気が、
斜め下45度に、√2×Uで流れるように見えるわけだから、空気が飛行機から
エネルギーをもらったように見える。キミが言ってるのはこれで、これは全く
正しいよ。

だからようは、どこに目線を置いて、そこから見て動いている物体は流体に仕事を
している、という整理でいいんじゃないか、と思ってる。
765ご冗談でしょう?名無しさん:03/03/18 03:50 ID:???
>>763-764
つまり、静止系ではベルヌーイの定理=エネルギー保存則は成り立たないってことかい?
まぁ、フツーに運動量で説明すれば良いものを、エネルギー保存則でやろうとするから
厄介なことになってるんだがな。

元スレとか前スレを読んでごらん。巷に蔓延する「圧力差」の壁を取り壊すのが
如何に大変なのかがよくわかるし (w
766745:03/03/18 04:01 ID:8/p6oFdC
>>765
そのとおり。成り立たない。なんでかというと、ちょっと今手元に教科書が
ないんだが、確か定常とか、いろいろ条件があるのよ。ベルヌーイ。

それよりも静止系での飛行力学、きぼんぬです。
‥てかキミもたまにはなんか説明しなさい。
767ご冗談でしょう?名無しさん:03/03/18 04:13 ID:???
つまり、現実の静止系だと使えなくなるよーなベルヌーイの定理を金科玉条にしている
説明はダメダメってことでしょ。

だから元スレ嫁って (w
768ご冗談でしょう?名無しさん:03/03/18 04:35 ID:???
>>764
飛行機が空気とは無関係に直線運動していて、空気が飛行機の直前で
飛行機から見て避けるが如く勝手に90度向きを変えたというシチュ
エーションね。
それを地上から見てると、空気の運動エネルギーが増えているじゃないか
だから、それは飛行機が空気に為した仕事が転じたものなんだという
短絡は間違いだってことね。
ま、そりゃそうだ。バイクとダンプが狭い一車線道路を逆方向に
走ってて、バイクがダンプを避けようと、路肩に向けて直角に曲がった
場合、ダンプがバイクに仕事をしたとは言わないのと同じだね。
もちろんダンプの運転手も仕事をしたとは感じない。静止系から見た
場合、バイクが内部のエネルギーを放出して運動エネルギーを増加さ
せたと解釈するのが普通。ダンプはその際に特別エネルギーを消費
しないのだから。
でも、>>761あたりは、飛行機は空気に対して仕事を実際しており、
空気の運動エネルギー変化に絶対的な影響を与えていると主張している。
空気が飛行機を見て勝手に内部のエネルギーのやりとりだけで曲がるこ
とはあり得無いという固定観念がそうさせているのだろうけれど。
にしても、>>764の例えはちと極端過ぎ。
769745:03/03/18 04:38 ID:0mnBYMWM
現実の静止系って何?
どんな座標系とっても、全部現実だよ。

あと、元スレは読めないので、キミが自分の言葉で、
自分が何を言いたいのか、説明してくれると有難いです。
770ご冗談でしょう?名無しさん:03/03/18 05:03 ID:???
771ご冗談でしょう?名無しさん:03/03/18 16:04 ID:UFwFS4eg
で、今でもまだ翼の揚力はベルヌーイの式で導かれると信じている人はいるの?
もうそれはいないよね?
772ご冗談でしょう?名無しさん:03/03/18 16:07 ID:M2rPDbny
773ご冗談でしょう?名無しさん:03/03/18 16:38 ID:???
>>771
ベンチュリ管(旧型エンジンの燃料吸入装置(キャブレタ))の
原理と飛行機の揚力の説明を同時に行うのにはベルヌーイの定理
を使うのが巧かったというだけでしょ。当たらずとも遠からずの
説明になっている。
正確かどうかはこの際度外視。エンジンのキャブの場合、燃料の
相変化も伴うわけだから、式どおり乗るわきゃないことは
すぐわかる。感じとか気持ちの問題だからね。
ま、同時到着するから圧力が下がるというのが一番わかりにくい
点で、論理関係特有のもの。物理的因果関係的には圧力が下が
ったから早く流れて同時到着するというのがわかりやすい。
その圧力が下がる理由も少しわかりにくい。
ちなみに上面の圧力が下がり過ぎて、早く流れすぎて同時到着
しなかった場合、ダウンウォッシュになるわけね。
同時到着するから圧力が下がっていなければならないというのは
それ自体は正しいけど、同時到着していない場合でも、圧力
が下がっていることがあり得るということをマスクしちゃう効果
があるよね。物理的自由度を殺してしまう点が問題かな?
774ご冗談でしょう?名無しさん:03/03/18 16:48 ID:???
同時到着し、圧力が下がっていることを観測することでベルヌ
ーイの定理が発見以来、今までの所成り立っていることが検証
出来る:OK

ベルヌーイの定理が成り立つから圧力が下がっていなければ
ならない:NG

物理法則も所詮は結果論として解釈するのが妥当なんぢゃないのか?
775ご冗談でしょう?名無しさん:03/03/18 18:33 ID:f+K4sLzN
>ちなみに上面の圧力が下がり過ぎて、早く流れすぎて同時到着
しなかった場合、ダウンウォッシュになるわけね。

これは違うと思う。
776ご冗談でしょう?名無しさん:03/03/18 18:50 ID:???
ふつー上面の空気の方が先に後縁に達するだろ。
777ご冗談でしょう?名無しさん:03/03/18 18:52 ID:???
>で、今でもまだ翼の揚力はベルヌーイの式で導かれると信じている人はいるの?

翼面の空気の流速と圧力はベルヌーイの式に従わないのか?
778ご冗談でしょう?名無しさん:03/03/18 19:58 ID:8ihaOJdV
>>777
従うだろうけど、揚力を生む原因とはあまり関係ないような気がする。
風呂のお湯はベルヌーイの式に従うけど、体が暖まるのはそれでは証明できない。
それと同じような感じで。
779ご冗談でしょう?名無しさん:03/03/18 23:52 ID:???
>空気が飛行機を見て勝手に内部のエネルギーのやりとりだけで曲がるこ
>とはあり得無いという固定観念がそうさせているのだろうけれど。

固定概念かよ。マヌケな流体屋には空気と翼の相互作用とゆー概念が無いらしい。
流れてる空気が翼に衝突しそうになると、バイクライダーのように「おっと危ねぇ。あらよっ」
と回避行動を始めるとでも思ってるのか?インテリジェンスを持った空気ねぇ。オカルト板に逝きな。

>どんな座標系とっても、全部現実だよ。

空気同士が応力を及ぼしあっているとする連続体近似は全然「現実」じゃないぜ。

しかし、圧力差が生じる理由をマトモに説明できる流体屋はいないの?
780745:03/03/19 00:50 ID:SBj0F99X
>>779
@んとね、768さんは独自の境地に達してらっしゃるので、漏れ745へのレスとは
別にしてくれるとありがたいです(^^)。
あとたぶん、768さんは流体屋さんではないので、「マヌケな流体屋」と言っては
気分を害されると思われ。どっちかといえば、779さんに近いのでは?

Aどんな座標系とっても全部現実というのと、連続体近似と何の関係が(;゚д゚)…!?
とりあえずちょっとおちけつ。

B流体屋、出席をとります。(゚▽゚)ノ ハーイ!
781745:03/03/19 02:25 ID:RpJxW3cu
。・゚・(ノД`)・゚・。
誰もいないのね。流体屋。
782745:03/03/19 02:27 ID:RpJxW3cu
@じゃあ、ベルヌーイ嫌いな人が多いようなので、。・゚・(ノД`)・゚・。
運動方程式(オイラー)からのアプローチでやってみるかのう。
流体の運動方程式は、ごちゃごちゃしてて、ほんと見ただけで
イヤになるんだけど、定常・粘性なしとして、主要項だけに注目すると、
すごい単純なことを言ってる。(式自体は教科書見るかググッてね)
ようは、
『流体の速度ベクトルは、流れながら、圧力勾配の低いほうに、
 向きを変えたり、伸びたりする』
‥のでつよ。※ここで高中低は静圧

  高高高             高中低
→ 中中中 \         → 高中低 ―→ 
  低低低  ┘          高中低

 こんなのとか‥         こんな風にね。
783745:03/03/19 02:28 ID:RpJxW3cu
Aずれた‥で、気をとりなおして、



→ / ̄ ̄ ̄ ̄\
→(/ ̄ ̄ ̄ ̄\>



こういう典型的な翼型(^^)があって、左から右に空気が流れてるとする。

翼の上面を流れる空気は、翼に添って、時計回りに向きを変えてく。
(ただし翼前縁の局所的には、反時計周り)
翼の下面を流れる空気も、翼に添って、時計回りに向きを変えてく。
こんな風に、速度ベクトルが向きを変えていくために、翼周りにどんな
圧力勾配が生まれてるかというと、上流の空気の圧力を「中」とすれば、
784745:03/03/19 02:29 ID:RpJxW3cu
Bこんな風になる。

中中中中中中中中中中
中中低低低低低低中中
中高/ ̄ ̄ ̄ ̄\低中
中(/高高高高\> 中
中中中中中中中中中中
中中中中中中中中中中

こうなってないと、流体は時計周りに向きを変えられなくて、ひたすら
まっすぐ進んで、翼の周りのあちこちに真空ができてしまう訳だ罠。
‥また、この結果として翼上面では空気が早く流れ、下面では遅く流れる。
785745:03/03/19 02:30 ID:RpJxW3cu
Cちなみに、この「運動方程式からのアプローチ(時計回り理論)」だと、
「翼の上面下面の経路長の違い+ベルヌーイの定理」で説明できない、
薄板問題、

→ \ こんなのや、

→ ⌒ こんなのに、揚力が生まれるのにも、適用ができます。

(糸冬)
786ご冗談でしょう?名無しさん:03/03/19 03:29 ID:6Oa1fQ1a
>745
もつかれー。

そして低学歴より一言。

745氏の解説の通りだと解釈してた。
するとこのスレではいろいろ聞いたことのない用語で議論されている。
結局この理屈でいいの?だめなの?
だめだとしたら、どこかの部分が飛躍してるってこと?
787ご冗談でしょう?名無しさん:03/03/19 04:38 ID:8dcXrYL3
>>745
重箱の隅をつつくなら、その翼型は少々問題ありそうだけど、これを言ったら嫌みか

あと圧力差が生まれる理由に時計回りに向きを変えるからというのがあるけど
その部分について俺は、もし空気が直進すると翼上面は真空を作ってしまい、翼下面は空気がたまってしまうような型だから、上面のほうが圧力は弱くなってしまうと勝手に思ってるんだけど、どうかな?
もちろん実際は真空など生まれなくて、上面の空気は翼に沿って流れるんだけど
しかも、剥離して渦を生まずにすむような型に作ってあるしね。

結局、空気がぶつかった反動で浮くってのと、圧力が高くて浮くってのは同じような理屈ですね。
どちらも空気が押しているのには違いがないのだから。
788ご冗談でしょう?名無しさん:03/03/19 05:02 ID:???
   ______________
 /:\.____\
 |: ̄\(∩´∀`) \  <先生!こんなのがありました!
 |:在  |: ̄ ̄ U ̄:|
http://saitama.gasuki.com/shinagawa/
789ご冗談でしょう?名無しさん:03/03/19 06:11 ID:8dcXrYL3
>>745
同意
790ご冗談でしょう?名無しさん:03/03/19 06:20 ID:p0u9sq5X
投げ下ろし効果説は、飛行機が背面になっても飛べることを説明
できる?

ベルヌーイで説明するやつは、それがどうしても納得できなくて疑問
を持っていたのだが。
791ご冗談でしょう?名無しさん:03/03/19 06:27 ID:???
>もし空気が直進すると翼上面は真空を作ってしまい、

翼空気が直進しようとすると翼上面に真空が発生してしまう。実際空気は
質量を持っているから慣性で直進しようとする。だから真空が出来やすい。
空気が圧縮(膨張)しなければ実際、真空が出来る時間が多いでしょう。
(出来ても、それは不安定で翼下面から空気が回り込んできてそれを埋める。
翼下面と上面の空気は翼上面で合流し、右斜め上に流れていく状況になり
これを剥離と呼んでいる。もちろん失速状態か寸前の状態)
実際には空気は膨張するので、真空にはならず低圧状態になる。速度成分に
ついては、下面より常に早くなるとも遅くなるとも言えないが、一定時間の
統計をとればほぼ同時到着が満たされている状態になる筈。
792ご冗談でしょう?名無しさん:03/03/19 07:04 ID:JAO9DISY
>>790
ベルヌーイで説明という奴より、断面対称同時到着(翼断面どこ取っても
飛行機の進行方向と同じ向きの流れになってて、状況は同じで、上面
下面で分かれた空気が同時到着する)に固執する限りベルヌーイの定理
で計算した圧力は下面が低くならざるを得ず説明出来なくなるってこと
でしょうね。背面飛行を説明する限り、断面対称同時到着は捨てざるを
得ないってことね。おいらとしては、存在が確実な物に乗っかって因果
的時系列的に論証していく空洞派(=真空派?≒745さんとか)の説明
よりも、どっか循環論法を含んでいるようなベルの説明のほうが実は気
に入ってる。
空洞派(時系列説明派)の論法は正確かも知れないけどちと危ないもの
感じるね。
(同時到着説は愛嬌ね。要するに原因はわからないけど、とにかく圧力が
下がりその為に速度も上がりそれが次の時刻においても、圧力が下がった
ままである重要な原因になってるってのがベルの説明の本質)
他にも要因としては迎え角を付けていることによる空気の衝動力も速度が
速ければ決して無視出来ないし、飛行機によっては急上昇し、上昇速度
が落ちていない段階で背面飛行に移り、暫らくの間背面飛行状態を保って
あたかも背面飛行しているように見せかけている奴もあるかも知れない。
揚力が激減しても、上昇速度がついている限り暫らくの間は上昇し続ける。
下降しだす寸前で元に戻して水平飛行ね。
地上のギャラリーなどは、飛行機の高度が高いと上昇しているか否かを
肉眼で確認するのは実はかなり難しいってことアリアリだと思うよ。
前縁フラップがノーマルな機械で下降しながら背面飛行って出来るって
奴があるんだったら、この部分は取り消すけど。
793ご冗談でしょう?名無しさん:03/03/19 09:48 ID:???
何だよ、マトモな説明をするのかと思って期待して読んじまったぜ。
流体屋のどこがダメなのかが際立っただけだな。

>>784
>こうなってないと、流体は時計周りに向きを変えられなくて、ひたすら
>まっすぐ進んで、翼の周りのあちこちに真空ができてしまう訳だ罠。

つまり、
 圧力差が生じるのは流体が向きを変えるから。
 流体が向きを変えるのは圧力差があるから。
ってことだろ。
一番肝心な圧力差の生じる理由がこれじゃ、全然説明になっていない
が、わかるか?

結局、飛行機が何故飛ぶのかは方程式を解いてみないとわからない
とゆー、これまでの流体屋の「公式見解」を繰り返しただけ。
他の説明はないのかよ?
794ご冗談でしょう?名無しさん:03/03/19 10:53 ID:???
圧力差が生じるのは、流体が向きを変えようとしないから。だろ。

まあ、「2ちゃん(や他のインターネット)に書き込んでる流体屋が
駄目駄目なのは否定しないが。
795ご冗談でしょう?名無しさん:03/03/19 12:52 ID:???
>>792
知り合いのパイロットに聞くと、航空機の種類によって
は背面飛行はほとんど普通の飛行と同じようにできるそ
うだ。もちろん燃料の効率は落ちるそうだが。

そうなると、「飛行機はなぜ飛ぶのか」という問いに対
する本質的な答えは、翼が上下対称であっても説明でき
るモデルである必要があるんじゃないかな。翼の「上」
という言葉が出てくる説明は全部駄目だ。それを「下」
に置き換えた途端に破綻してしまう。

上下対称で説明できない説は、今の飛行機の翼の形を採
用したほうがほうが効率が良くなることだけの理由であっ
て、疑問の核心には答えてないような気がする。
796ご冗談でしょう?名無しさん:03/03/19 13:55 ID:UjrxrGtE
そういやハエって引っくり返してもそのまま上に飛んでいくらしいね
797ご冗談でしょう?名無しさん:03/03/19 13:59 ID:SM164+bu
★男はココを見るべし★女と金と???★
↓↓↓↓↓↓↓↓↓↓↓↓↓↓↓↓↓↓↓↓
http://www.pink-angel.jp/betu/linkvp2/linkvp.html
798ご冗談でしょう?名無しさん:03/03/19 17:38 ID:ovv0X9Z7
>>795
普段、おもいっきし急降下をかけたい時にやる事を、背面でやればなんとか飛べるんじゃないかな・・・
想像だけど。
どちらにしてもベルヌーイでは説明できないわな。
799ご冗談でしょう?名無しさん:03/03/19 19:14 ID:???
翼が上下対称であっても、迎え角が付いている以上非対称な流れ
発生しているのは自明でこの場合も揚力は圧力と速度の間の制約
関係(むしろその高い確率の原因は、空気の慣性による流れの軌道
が翼から離れようとすること)で説明できる。
しかし前縁で上面に空気が多く回りこみやすいこの形状では揚力は
発生しにくいし不安定になる。背面飛行もこれとほぼ同じ状況だが
前縁が滑らかな形をしているので、多少は安定するのではないか?
800ご冗談でしょう?名無しさん:03/03/19 22:29 ID:???
背面飛行は、正直どーでもいい。揚力の本質的な問題じゃない。
断面がカマボコ型の翼で、上下同時到着、ベルヌーイの定理で上部低圧
ってのをシタリ顔で語るヤシに、「背面飛行って知ってるかい?」と訊ねてイヂ
める時には役立つけどな (w
迎角のある平板でも揚力は発生するわけだから、まぁ、平たい翼をひっくり返して
適当な迎角で飛んでもオッケーだろ。抗力が増えすぎて失速する可能性はあるが。
801ご冗談でしょう?名無しさん:03/03/19 22:44 ID:WjLbSlEZ
>>799-800

つまり、揚力が生まれる本質的な理由は「迎角がついているから」で良いの?

中学の実験で、仰角0度に固定したかまぼこ型の翼に風を当てたら
浮いたように記憶しているのだが・・・。記憶違いかなあ。
802ご冗談でしょう?名無しさん:03/03/19 23:42 ID:???
>>801
翼にキャンバー(反り)があるときは,迎角ゼロのときも
揚力が発生する。この場合も翼に沿って気流がダウンウォッシュ
になる反動で揚力が発生すると思われる。
803ご冗談でしょう?名無しさん:03/03/20 00:14 ID:???
>>801
「本質的な問題じゃない」って書いてあるぜ。

>>802
おぉ、イイ感じの説明だ。

安直に圧力差を持ち出すと破綻するってのが浸透してきたのかな。
そろそろ流体屋の出番だが、どーゆー言い訳をするのか、チョト楽しみ。
804745:03/03/20 00:48 ID:II7LWd1r
背面飛行って何?

もしかして、飛行機が裏返って飛ぶときに、ただ水平に裏返っただけじゃ、
自重と翼が生み出す下向きの揚力で、あっという間に高度を失って、
地面に叩きつけられてしまうので、それを打ち消すだけの強い迎え角と、
エンジン推力の上向きの分力とで、強引に水平を保って飛ぶ、飛び方のこと?

それが翼が揚力を生み出すメカニズムに、何か関係があるんですか?
全く分からないので、説明きぼんぬ。
805745:03/03/20 01:01 ID:II7LWd1r
‥‥あと、ちょっと不安になってきたので、このスレにいる人に質問。

・水平な流れの中に、上下対象物体が置かれている。この物体は、
 流れから、重力と反対の方向の力を受ける。
・水平な流れの中に、上下非対称物体が置かれている。この物体は、
 その形状にかかわらず、流れから、重力と反対の方向の力を受ける。

‥これ読んで、うんうん、そーだそーだ、とうなづいた人、

↓ここに行けば、楽しい友達が、いっぱい待ってるよ。(^^)/~~




















ttp://etc.2ch.net/denpa/ 
806ご冗談でしょう?名無しさん:03/03/20 02:50 ID:QYUXPHBG
迎え角と推進力だけで飛行機の重さを支えられるのですか?
807ご冗談でしょう?名無しさん:03/03/20 03:03 ID:???
>>805
コワレたのか?
808ご冗談でしょう?名無しさん:03/03/20 03:12 ID:???
作用・反作用の法則があるから、揚力が発生するのですか?
揚力が発生しないのは、作用・反作用の法則が成り立たないから
なのですか?
作用・反作用は相互作用をする質量を持った物質の間でいつでもどこでも
成り立つ関係ぢゃないんですか?(ミクロの世界じゃそうじゃないらしい
けど)
809ご冗談でしょう?名無しさん:03/03/20 04:12 ID:???
>>804
>もしかして、飛行機が裏返って飛ぶときに、ただ水平に裏返っただけじゃ、
>自重と翼が生み出す下向きの揚力で、あっという間に高度を失って、
>地面に叩きつけられてしまうので、それを打ち消すだけの強い迎え角と、
>エンジン推力の上向きの分力とで、強引に水平を保って飛ぶ、飛び方のこと?

それが本当だったら、翼が生み出す揚力というのは仰角とエンジン推力に
負けてしまう程度のものということになる。だったら、「飛行機がなぜ飛
ぶのか?」の理由はそっちにした方が良いんじゃないか?、っていう話だ。
810ご冗談でしょう?名無しさん:03/03/20 04:40 ID:???
>>804
背面飛行がそういう飛び方だとして...
強い迎え角を付けたというのは、引っくり返ってはいるが迎え角を
付けることによって翼に揚力が発生するからそうするんじゃないの?
何か関係があるのかというからには、反対向きになると迎え角付けて
も揚力が出ないって考え方みたいだ。
となると背面飛行出来るからにはエンジンの推力がメインということに
なるけど、エンジンの推力が何G出せるのかはしらないが、機首の角度
から言って並の飛行機のエンジンじゃ垂直方向成分で1G出すのは難し
そうだ。30度という強烈な迎え角でも、垂直分力は1/2.2G近い
推力出せるエンジンが必要になる。少なくとも普通の飛行機じゃ絶対
無理な話になる。
だから>>809の結論は早計で、>>804の話が本当だとしても特殊な飛行機
の事例を強引に一般化しようとしている。
811745:03/03/20 04:58 ID:0f4wHKw/
>>793
@教科書チックに、『未知数の数がn個で、方程式の数がn個なので
‥境界条件を与えると‥』と説明してもあまり面白くないし、
絶対納得もしないだろうから、また例え話にすると、

バネの先にオモリをつけて、引っ張ってから離すと、振動を始める。
これは、1変数1方程式の‥なんていうのかね?系っていうのかね。
812745:03/03/20 04:59 ID:0f4wHKw/
Aバネとオモリが方程式で結ばれてるように、流体では、
圧力と速度が方程式で結ばれ、あと速度どうしも方程式で結ばれてる。
(密度とか温度の変化は割愛)

バネとオモリの系の挙動を決めるのは、最初にどんだけ引っ張ってから
手を離すかだ。(引っ張らなきゃずっと止まったまま)
それと、バネとオモリが、水平面にいるのか、傾斜面にいるのか、
曲面にいるのか。

流体と物体の系を決めるのは、上流からどんだけの速度と
圧力で流れてくるかだ。(流れてこなきゃずっと止まったまま)
あと、流れの中にどんな物体がいるのか、水平板なのか、傾いた板なのか、
翼型なのか。
813745:03/03/20 05:00 ID:0f4wHKw/
B何がいいたいかというと、現象を決定してるのは、方程式の形と、
境界条件とか初期条件とか、そういうもんだってこと。
変数同士で、どっちが原因でどっちが結果か何て、質問自体が無意味だろ?
いっしょに変わるんだよ。

>つまり、
> バネが伸び縮みするのはオモリが動くから。
> オモリが動くのはバネが伸び縮みするから。
>ってことだろ。
>一番肝心なバネが伸び縮みする理由がこれじゃ、全然説明になっていない。
>わかるか?

‥って、言ってるのと同じ。
814745:03/03/20 05:10 ID:0f4wHKw/
>>810
ああ、どうもありがとうです。
その通り。

言いたかったのは、

『揚力は、物体の形にかかわらず、重力と反対方向に発生する』

‥んなわきゃねーだろ!ってことでつ。
815ご冗談でしょう?名無しさん:03/03/20 05:41 ID:jGamWNZB
★男はココを見るべし★女と金とサンプルムービー★
↓↓↓↓↓↓↓↓↓↓↓↓↓↓↓↓↓↓↓↓
http://www.pink-angel.jp/betu/linkvp2/linkvp.html
816ご冗談でしょう?名無しさん:03/03/20 05:45 ID:IMUENDQk
1000までには結論がでそうだな
817ご冗談でしょう?名無しさん:03/03/20 06:07 ID:???
ベルヌーイの法則から類推される
揚力の翼周辺分割気流末端同時到着原因低圧発生説
を毛嫌いするヤシは、少なくとも>>808の質問に答えてあげてやれば?
818ご冗談でしょう?名無しさん:03/03/20 06:23 ID:???
>>817

>>808 とベルヌーイの法則は全然関係ないだろう。
同時到着説は、現に気流が同時に到着してないのだから
間違っているのは確かだよ。
819ご冗談でしょう?名無しさん:03/03/20 08:22 ID:???
>>818
で、作用・反作用の法則を揚力の原因として説明したい人にとって
>>808の質問はどう答えるの?
820ご冗談でしょう?名無しさん:03/03/20 10:16 ID:???
>>819
法則があるから力が発生する、なんて流体屋みたいな考えがイケない。
揚力が発生するのを作用反作用の法則で理解(説明)しているんだよ。
揚力がゼロになるってのは条件がよくわからんが、別の力と打ち消しあって
いるのかもしれんな。

これで、いいか?
821ご冗談でしょう?名無しさん:03/03/20 10:31 ID:???
>>811-813
流体屋がダメダメだってのをダメ押ししてどーするんだよ (w

>B何がいいたいかというと、現象を決定してるのは、方程式の形と、
>境界条件とか初期条件とか、そういうもんだってこと。

よーするに、与えられた条件で方程式を解いてみないと何が起こっているのか
わからない、そーゆーことだろ。

>変数同士で、どっちが原因でどっちが結果か何て、質問自体が無意味だろ?
>いっしょに変わるんだよ。

こーゆーことを言っているのかい。
中学生「翼にはなんで揚力が発生するんですか?」
流体屋「翼の上側の圧力が下がっているんだよ」
中学生「なんで圧力が下がるんですか?」
流体屋「圧力が下がるように空気が流れているからだよ」
中学生「なんでそのように空気は流れるのですか」
流体屋「流れは圧力で決まるんだよ...そんな質問は無意味だかやめなさい」
中学生「(なんだ知らないのか。使えねぇヤシ)ありがとうございました」

バネとオモリの例えは、ほぼ意味不明だな。特にバネの伸びとオモリの変位を別に
考えてしまっているあたり。まぁ、物理がニガテなのはよくわかったが。

そろそろカンネンして連続体近似を諦めたらどーだ。
ラクになれるぞ (w
822ご冗談でしょう?名無しさん:03/03/20 10:32 ID:???
>>820
>法則があるから力が発生する、なんて流体屋みたいな考えがイケない

といいつつ

作用反作用の法則(があること)で揚力が発生するのを理解している
というのはいけないな。
823ご冗談でしょう?名無しさん:03/03/20 10:53 ID:???
アタマのヨワいつっこみだな。もっとガンガレ。

法則があるから力が発生する≒方程式があるから揚力が発生する→○な流体屋
法則で現象を理解する→フツーの物理屋
824ご冗談でしょう?名無しさん:03/03/20 11:06 ID:???
で、作用反作用の法則があるから揚力が発生するんだろ?>>823≒○な物理屋さん?
漏れは揚力が発生しなくなた日には、作用・反作用の法則の存在もアヤしいと
思っているが。
825ご冗談でしょう?名無しさん:03/03/20 15:49 ID:???
飛行機ってのは上から紐で吊るされているんだよ。解ったかい?
826ご冗談でしょう?名無しさん:03/03/20 16:13 ID:???
827ご冗談でしょう?名無しさん:03/03/20 19:06 ID:???
安易に圧力差を否定するやつは、初歩適な風洞実験さえしたことないやつだよ(w
828ご冗談でしょう?名無しさん:03/03/20 19:12 ID:???
>821
すると、おまいさんが隣に住んでる消防に相対性理論を理解させることが
できなかったら、相対性理論は間違ってることになるのか
829ご冗談でしょう?名無しさん:03/03/20 19:20 ID:???
まあ、前にもこのネタでスレが続いたけど、結局、なんで翼上面の流速が
速くなるのか説明できる人いなかったね。
830745:03/03/21 02:05 ID:WVuJEbec
>>821

>中学生「翼にはなんで揚力が発生するんですか?」
>流体屋「翼の上側の圧力が下がっているんだよ」
>中学生「なんで圧力が下がるんですか?」
>流体屋「圧力が下がるように空気が流れているからだよ」
>中学生「なんでそのように空気は流れるのですか」
>流体屋「流れは圧力で決まるんだよ...そんな質問は無意味だかやめなさい」
>中学生「(なんだ知らないのか。使えねぇヤシ)ありがとうございました」

‥だー、かー、らー!
誰もそんなこと言ってないっての。どうしてわざと曲解するの?。・゚・(ノД`)・゚・。
「変数同士で、どっちが原因でどっちが結果か」
なんて質問自体が意味ないんだよ。原因は初期条件とか境界条件なんだってば。

またバネで例えたら、バネ@の先にオモリ@つけて、オモリ@の先にバネAつけて、
バネAの先にオモリAつけて、引っ張って離すと。
もまえは、オモリ@、オモリAに、「原因」「結果」というラベルを
つけないと、気がすまないのかよ。
どっちのオモリににどっちのラベルをつけるのかは知らんが。
831745:03/03/21 02:07 ID:WVuJEbec
>>829
正確に言えば、いくら説明しても理解できない人(一部)がいた、ということ。
832745:03/03/21 02:07 ID:WVuJEbec
@あとさー、勘違いしてもらうとこまるのだが、
一般の流体屋は、『同時到着』は間違いだと思ってるよ。
‥てか、流体力学の教科書には載ってません!そんなトンデモ法則は。

流体力学の専門家でない人が、個人HPとかで勝手に妄想してるんです。
『同時到着』を定理チックに記述すると、

「水平流れの中に置かれた任意断面周りの流れは、前縁で上下に分かれた後、
 後縁で同時に合流しなければならない」

とでもなるんだろうが、このトンデモ定理がもしホントだと、
流体屋が計算するときに、導入する変数の数に応じた方程式を準備
するわけだけど、このトンデモ定理のせいで、ものすごいキツい
拘束条件が1コ増えてしまうので、逆に計算できなくなってしまう。

‥はっきり言って、そんな法則があったら困るんです。迷惑なんです。
833745:03/03/21 02:08 ID:WVuJEbec
Aんで、さらに困るのは、このスレにいるアンチ流体力学派が、

『同時到着マチガイ』
    ↓
『ベルヌーイの定理マチガイ』
    ↓
『流体力学マチガイ』

という、論理的に誤ったデマゴギーをまきちらしていることなのだが、
おまいら、なんでそんなこと言うのかと。問い詰めたい。
小一時間問い詰めたい。‥個人HPで専門外のヤシがなんか誤用して
いることなど、知らんよと。んじゃ、流体屋の漏れが自分のHPで、
特殊相対論とか量子力学とかを、誤って誤用したら、特殊相対論とか
量子力学の体系自体を否定するのかよ、と言うことだ。
834ご冗談でしょう?名無しさん:03/03/21 02:50 ID:???
厨房工房の思いつきで出来るくらいの物なら、とっくに出来てる。
古人曰く、下手の考え休むに似たり。
失せろ。
835ご冗談でしょう?名無しさん:03/03/21 18:55 ID:6P7OMMHE
>>834
翼上面に低圧部ができるメカニズムは,712,784,787等で
説明されている思うが。要は,翼上面を流れる気流が,空気分子の慣性の
せいで,翼上面の後ろに回り込むのが遅れて,翼上面の気圧が低くなると
いうこと。
説明が残っている点は,なぜ翼下面の高圧部から翼後端をまわりこんで
翼上面の低圧部に流れ込まないのかというところ。
836ご冗談でしょう?名無しさん:03/03/21 19:44 ID:???
>>835
実際回り込む場合があります。その際は失速になるか失速寸前の状態です。
速度が遅いとそうなる可能性が高くなります。
早すぎても今度は上面が低圧に成り過ぎて何かの拍子に突如回り込むこと
が起こりえます。
大抵の場合下面の空気は翼末端で相当に高い速度を保っており、上面から
合流してきた空気との間にせん断応力(粘性)を作用させて多少減速します。
しかし慣性がある為に上面まで流れ込むことはありません。上面の低圧部
の影響を受けて多少の減速と方向変化はありますが。
いつ失速になるかは、基本的には翼の形状、速度、周辺大気の密度や
温度などから決定されるのでしょうが、確率的にしか決められないとしか
言わざるを得ません。ある瞬間に下の空気が上面に回り込むか回り込まな
いかは確率的にしか決定されません。
より小さな時間スケール・空間スケールの揺らぎがあるタイミングで拡大さ
れて激変を起こす可能性があるからです。
同様に回り込みが発生している状態でそれが解消されることも確率的には
起こりえます。
十分高速で飛んでいても、短い時間、狭い領域では回り込みが起こっていても
おかしくはないでしょう。
837ご冗談でしょう?名無しさん:03/03/21 20:16 ID:???
空気の慣性が本質的であるということは
元スレ(http://cheese.2ch.net/sci/kako/988/988866699.html
の247辺りで既に指摘されてる。
それで重力と拮抗し、上昇加速度は下面に作用する動圧の分力を
援用するということや、
超音速時には風圧(動圧)のほうが支配的になり、それを神経質
に利用していることまで(間接的だが)指摘されてる。
もう2年近くも同じこと議論してるんだね。
838ご冗談でしょう?名無しさん:03/03/21 21:29 ID:6P7OMMHE
>>837
でも,下面から上面に回り込まない理由として,慣性があるから
だけでは弱いでしょう。
もし,翼後端で気流が無限大の流速で回り込み,淀み点が翼下面上に
できれば,循環がゼロになり,揚力が生じない。
過去スレで,空気の慣性を使った説明は何度か出てるが,たとえば
スレ1をみると,

277 名前: 27 投稿日: 2001/05/14(月) 12:30
  圧の高い翼の下側から上側に流れ込まない理由は何でしょう?
281 名前: 27 投稿日: 2001/05/14(月) 13:31
>>279
>圧力差のある上下空間の中間部を翼という名の膜で閉鎖してる
>から。だから翼は上に引っ張られる。

そんな都合の良い「膜」なんて存在しないでしょ?
あるんだったら、ちゃんと説明してみてね。

という感じで終わり,下面から上面に回り込まない理由の説明が
でてこない。
839745:03/03/22 02:17 ID:OqcTWPoY
>>835
その通りです。漏れが書いた784の説明の、一番の弱点を
よく衝いていただきました。この回り込みへの疑問から、
循環で揚力を説明するメインディッシュを、じゅーっ、と焼いて、
喰った喰った〜( ゚д゚)ウマー‥とつながるわけですな。

>>836
本職の方ですか?的確な解説ありがとうございます。
ええと、失速や回り込みが確率論的にしか決定できないというのは、
乱流境界層?とか、渦領域の内部が乱流?‥の影響なんでしょうかね?
この辺まったく不勉強なので、またご解説いただけるとうれしいです。
840ご冗談でしょう?名無しさん:03/03/22 08:04 ID:???
>>838
だから回り込むこともあり得ると>>836は言っています。そして回り込まない
時は大きな揚力が発生している時と考えて良いのでしょう。
逆に聞きたいのですが、なぜ常に回り込まなければ不思議だと思うのでしょうか?
回り込むことも、回り込まない場合も両方あり得て、予測するのは難しいのですが
一定期間毎に交代交代でどちらかの状態が実現するという描像は不自然でしょうか?
慣性と粘性によって、次から次へとやって来る流体同士の間にある種の「伝統」
が発生してどちらの場合も、ある程度の時間その状態が持続される筈ですが、
何らかの拍子に、別の状態に遷移する。(「伝統」は粘性や圧力分布が伝える)
ただ、回り込まない時間が長いのは何故か?という問いに対しては、中々答える
のは難しいでしょうね。回り込む場合に比べて回り込まない場合のほうが「伝統」
が維持されやすいということなんでしょうけど。
あと、循環という概念は3次元で翼を考える時にも有効でしたっけ?もしよろし
ければ、その辺りの解説をしている文献在りましたらお教え下さい。
841835=838:03/03/22 10:27 ID:???
>>840
前の議論を踏まえない話をしたことは,すみませんでした。
もし,下面から上面へ回り込めば,揚力は発生しないので,揚力があるときは
どういうメカニズムで回り込まなくなっているのかについて説明が必要だと
思いました。
言い換えると,最初の循環ゼロの状態から,どのように循環が発生するかとい
う点です。
出発渦を使った説明だと,淀み点が翼上面上にあるときは,境界層内で,圧力
が低い翼後端から圧力の高い淀み点に向かって流れがある状態となるため,境
界層が剥離して淀み点が翼後端に移動する。その際,循環が−Γの出発渦が気
流内に生じ,逆に大きさΓの循環が翼周りに生じるというものだったと思いま
す。
840さんのお話だと,上記の話は3次元では間違っていることになるのでしょ
うか。
842835=838:03/03/22 10:30 ID:7Aupn8pW
(つづき)
2次元の場合,淀み点が翼後端に移動すると,翼後端で上面,下面それぞれの
流速が等しくなるよう合流することになると思っていたのですが,836=840
さんだとして,836では,
>大抵の場合下面の空気は翼末端で相当に高い速度を保っており、上面から
>合流してきた空気との間にせん断応力(粘性)を作用させて多少減速します。
とあり,それは間違いなんでしょうか。
境界層近似は,翼表面の境界層外部は非粘性流体として扱えるということだと
理解していましたが,「せん断応力(粘性)を作用させて多少減速」するとい
う話だと,境界層近似が成立していないことになりませんか。

なお,私は専門的なことはわからない素人なので,文献等を提示することは,
すみませんができないです。
843ご冗談でしょう?名無しさん:03/03/22 11:24 ID:???
>>842
境界層近似はあくまでも近似で、どのような揚力の発生の場合をも尽くし
ているというわけでもないでしょう。循環が0でない(ある条件のもと
での2次元翼モデルで揚力が発生しうる為の条件)流れがどのようにして
生まれるのかを解説したもので、それと組み合わせて考えないと意味ない
んじゃないの?
844ご冗談でしょう?名無しさん:03/03/22 13:45 ID:???
物理のニガテな連中が寄って集って井戸端カイギってところだな。

空気に慣性があるから翼上部が低圧になるってのは、>>838にも書いてあるように
説得力が無い。とゆーか、ほぼ間違い。
「空気は真っ直ぐに進むから、翼の“カゲ”になっている部分の圧力が下がる」
って、短絡的な発想はすぐに否定できるな。もしそーならば、流れと翼が最も離れている
後端で圧力は最小になるが、そんな圧力分布になっていないことは流体屋でも知ってる (w
結局、速度と圧力は方程式を解いてみないと求まらない。

あと、翼の断面と垂直な方向の流れ(って、ほとんどゼロ?)が、揚力を発生させる
のに重要なファクターではないので、2次元で理解するのに問題は無い。流体屋の
好きな「翼端渦」は3次元でないと説明できないが、揚力発生の主因ではない。
循環と循環流をコンドーしていると「これぞ揚力発生の源」と思いたくなるよーだが。
実際の飛行機を設計するときは3次元で計算をして欲しいけどな。

で、コマったちゃんの流体屋だが、このスレにいるデムパ以外は「圧力差」は否定
していないし、「ベルヌーイの定理」や「流体力学」が間違っているなどとは、ミジンも
考えていないぞ。デマとはオマエの書いた>>833のことだぜ。
方程式を解かなければ何もわからないのに、揚力を説明するとき安易に
「圧力差」持ち出して「わかったつもり」になっているのがマヌケ
だと指摘しているのだが、わかるか?
揚力がなぜ発生するのかとゆー問に
「それは方程式の条件が原因になっているからです」
と言われても「ハァ?」だろ。

もし出来るのなら、中学生と流体屋の会話で揚力を説明してごらん。
845745:03/03/22 14:25 ID:T9+ywnGo
>>844
ゴメソ!デマとばして正直すまんかった。
じゃあ以後、833チックな話は、デマとして『一切禁止』ってことで。
ご指摘ありがとうです。
846745:03/03/22 14:43 ID:T9+ywnGo
836氏が現れないようなので、

>2次元の場合,淀み点が翼後端に移動すると,翼後端で上面,下面それぞれの
>流速が等しくなるよう合流することになると思っていたのですが,
淀み点に接近するにつれて、上面の高い速度と、下面の低い速度が、それぞれ
0を目指して漸近して、最後の最後に0になって一致するのでは?と思ってます。
淀み点から離れてたら、やっぱり速度は違うんじゃないかな〜?

>境界層近似は,翼表面の境界層外部は非粘性流体として扱えるということだと
>理解していましたが,「せん断応力(粘性)を作用させて多少減速」するとい
>う話だと,境界層近似が成立していないことになりませんか。
もう翼から離れた後なので、境界層近似は別に関係ないのでは?と思ってます。

836氏のご意図と違ってたらすみません。
847ご冗談でしょう?名無しさん:03/03/22 14:43 ID:???
>>843
ということは,実際の翼では境界層近似から大きく外れてるということです
か。
>大抵の場合下面の空気は翼末端で相当に高い速度を保っており、上面から
>合流してきた空気との間にせん断応力(粘性)を作用させて多少減速します。
は,クッタの条件を満たしてないですよね。
848835:03/03/22 15:23 ID:???
>>846
境界層の外の気流という前提ですが,
上下のぶつかる部分が,翼後端の淀み点で,そこで上下に
流速差があり,その速度差によるせん断応力のために下面の
気流が減速するという内容に私には読めました。
そうすると,翼周りのごく薄い境界層の外部では粘性を
無視できる境界層理論(?)と違ってくると思ったのですが。
もし,上下が合流する前に下面の気流が減速するということであれば,
せん断応力以外には圧力勾配しか力の働く要因はないので,下側で
気流の進行方向に圧力が増加する状態が現れていることになるが,
そうなっているということなんでしょうか。
849745:03/03/22 16:26 ID:xv+Mr0ka
836氏ではないんですが、836氏がクッタの条件を知らないとは思えないので、
翼上面のu1、翼下面のu2が、翼後端の淀み点から出る流線上で、クッタの条件から
同じ速度のuになっているんだけど、その流線の上下にはやはり(有限の)速度勾配
が発生していて、そこにせん断応力が発生している、というイメージではないか、
と思います。

でも835氏の心配されていることも分かります。835氏は、836氏が、u1、u2のまま、
翼後端以降の流線上で無限の速度勾配があって、そこにせん断力が発生していると
主張されている、と心配なさってるんですよね。(違ってたらすみません)
それは確かにあり得ないです。
850745:03/03/22 16:32 ID:xv+Mr0ka
あと、もしご賛同いただければですが、継続して書き込みをされる場合は、
できればコテハンにしませんか?もしお嫌でなければですが。

そろそろ誰が誰か、全然分からなくなってきたですよ。。・゚・(ノД`)・゚・。
851835:03/03/22 17:30 ID:???
>>849
コテハンの件については,いい名前があるか考えてみます。
836氏に聞くべきことなのですが。。。
境界層に沿ってすぐ外を流れる気流u1(上面側),u2(下面側)を考えると,
せん断応力が働くのは境界層の中だけなので,境界層の外のポテンシャル流
領域を流れるu1,u2にはせん断応力は働かず,u1,u2の上下の流線もポテンシャ
ル流領域にあるので,やはりせん断応力は働かないと思うのです。
境界層を越えて出入りが可能なので,単純に境界層にそって気流が流れるという
いいかたが正しいかはわからないのですが。
852ご冗談でしょう?名無しさん:03/03/22 19:13 ID:???
>>4
>第3スレまでくるとは思わなかったけど、やるんだったら
>
> 「流体力学で揚力を説明してみよう」

流体屋さんたちに厨房でもわかるような揚力の説明をお願いします。
第一スレから読んでいますが空気の運動量変化による説明はなんとなくわかりますが
圧力差による説明はどれも途中でわからなくなってしまいます。
853745:03/03/22 23:55 ID:6Y952VpQ
>>851
835さん、やっと仰ることが分かりました。
確かにそう考えると???ですね。

てか、漏れがクッタの条件を理解してない?!
クッタの条件って、後縁淀み点の近傍の、
相当広い範囲でu1≒u2って意味なのかな?
854ご冗談でしょう?名無しさん:03/03/23 01:23 ID:???
食った条件は、境界層領域に特異点があって、その周辺にはありとあらゆる速度
が実現されているというもの。それがあるか無いかで揚力の有無が決まる。
翼後端点にそれが一致していると考えるのが普通。それが後ろにずれている
場合は、境界層が翼表面だけでなく翼後ろにも生じていると解釈出来る。
この場合の揚力の計算は単純じゃなくなってくるだろうね。数学的には
リーマン面というものを考えなくてはならなくなったりする。
855ご冗談でしょう?名無しさん:03/03/23 01:45 ID:???
>>854
>食った条件は、境界層領域に特異点があって、その周辺にはありとあらゆる速度
>が実現されているというもの。それがあるか無いかで揚力の有無が決まる。

ってゆーか、粘性を無視した完全流体だと循環が一義的には求まらないので、
翼の後端で上下面の流れがなめらかに離れることにしよう、とゆーのがクッタの
条件だろ。そーすれば流速が有限になって循環を計算することが出来る。
クッタの条件の有無で揚力の有無が決まっているわけではない。
実在気体を完全流体で近似をしているから、クッタの条件がないと揚力が
「計算」できないだけ。
856ご冗談でしょう?名無しさん:03/03/23 02:00 ID:???
>>855
理化学辞典丸写し的にはそうかも知れない。が、循環=0だと2次元
無限領域内の翼型領域には揚力は発生しない。
857ご冗談でしょう?名無しさん:03/03/23 02:06 ID:???
>>856
理化学辞典のコピペなら間違いではないだろ (w
循環=0でも揚力が発生すると誰か書いたのかい?
858ご冗談でしょう?名無しさん:03/03/23 02:23 ID:???
>>844
>もし出来るのなら、中学生と流体屋の会話で揚力を説明してごらん。

>>852
>流体屋さんたちに厨房でもわかるような揚力の説明をお願いします。

この説明が出れば飛行機スレも目出度く大団円を迎えることができるな。
是非コテハン745=流体屋にやって欲しいぞ。
できるか?
859ご冗談でしょう?名無しさん:03/03/23 07:52 ID:???
>>844
密度が0に近い(つまり慣性が0)の流体でも、揚力が発生すると思っているのかな?

>>857
クッタ条件を満たす流れの場合、周囲の流体の運動量など計算出来るのか?定常状態なのだから
計算してもトータルな運動量変化はゼロだぞ。やっぱり圧力差じゃなきゃ説明はムリなんだろな。
有界領域に限って理論を適用したとしても、
[流線上での単位時間毎の運動量変化]=[翼にかかる圧力の総和]ってこと示さなければ
運動量変化だけで理解させるのは難しいだろうな。
ポテンシャル流を2次元で展開しようとすると、運動量変化は特異点に集中してしまうという
基本的問題がある。クッタのモデルに固執すると、翼後端に運動量変化が集中することに
なって、そこに湧き出し(ホワイトホール)とか吸い込み(ブラックホール)を考えなくてはなら
ないので奇妙な世界になる。
860859:03/03/23 08:25 ID:???
>>857じゃなくて>>855へのレスね。
861ご冗談でしょう?名無しさん:03/03/23 09:02 ID:???
>>844

作用・反作用の法則は、力を相互に及ぼしあっている孤立した2つの質点同士に働くアトミック
な力には一定の規則があるぞってことだけ。複数の質点からなる体系でもこういったアトミック
な力の総和が働いているという考え方は、その考え方を一段飛躍させたもの。さらに飛躍させる
と、質点系とみなすのには問題が無いわけでもないものにまで適用するようになる。
流体力学とか剛体力学なんてのがそういう例だろね。
それは一応棚にあげておいて...
この法則はそもそも2つのアトミックな質点の間にどのくらいの大きさの力がどのくらいの時間
働くかについては何も規定しない。ゼロでも何ら問題が無い。引力が働くべき状況で斥力が出て
きたとしても問題が無い。力がいつ何故働くかについても何も言わない。
だから重力と釣り合う位の揚力がある一定の時間持続的に発生している事実を説明するのには
不適当。

慣性(気体の質量)で説明するのも問題あることは間違いが無い。揚力がゼロ(つまり失速)の時
に慣性が無かったというわけでもないし、もとい条件によっては失速が起こるのだから。
ただ、揚力が発生している状況で仮想的に大気の密度が局所的じゃなく全体的に増加
して空気自体の慣性パラメータが変動したと仮定すると、翼上面での圧力上昇(上面からの下
降気流流入、下面からの回り込み)がより妨げられるだろうということは言えそうだからこそ多少
説得力があるというものなのだが。(この考え方はレイノズル数を変化させて考えることに相当し
ます。)
もちろんこの考え方では、どのような条件で揚力が発生し得るのかは方程式を解いてみないと
わからない。ただし揚力が発生している状況は有限の時間は継続するだろうということは言えそう。
作用・反作用の法則だけでは多分出てこない。
862ご冗談でしょう?名無しさん:03/03/23 09:18 ID:???
流体の方程式だけ見ていて物理を置き去りにしているから、変な妄想がわいてくる。
計算できるかどーかってのが問題じゃないんだよ。

>>859
>密度が0に近い(つまり慣性が0)の流体でも、揚力が発生すると思っているのかな?

密度≒0なら揚力≒0だが何か?

>定常状態なのだから計算してもトータルな運動量変化はゼロだぞ。

飛行機通過後に生じているダウンウォッシュは空気の運動量変化だが、定常状態だと
消失するのかよ。そんなおバグなアタマの香具師に

>やっぱり圧力差じゃなきゃ説明はムリなんだろな。

と言われても困っちゃうなぁ。
ってゆーか、そろそろ圧力差で説明しておくれよ。>流体屋=745

>[流線上での単位時間毎の運動量変化]=[翼にかかる圧力の総和]ってこと示さなければ
>運動量変化だけで理解させるのは難しいだろうな。

「流線上での」ってのは不要だろ。「運動量変化=力積」は知ってるよな。そーしたら
「空気の単位時間毎の運動量変化」=「翼が空気から受ける力」=「圧力の総和」だぜ。
示せたけど、理解したか?

でも、翼の各部分の圧力を運動量変化で細かく求めるのは難しい(面倒)。
だから翼を設計するときなどは「計算はバッチリできる」流体力学が必要になってくる。
で、流体の方程式を解くのはカナーリ厄介だから、解を見つける過程でいろいろと問題が
生じてくる。流体屋が気にしているのはそこのところであって、本質的に物理を議論
しているわけではない。
863ご冗談でしょう?名無しさん:03/03/23 09:26 ID:???
>>861
前半のパラグラフは意味不明だが、何が言いたいの?
後半は流体の質量だけを考えていたのではダメで
質量×速度=運動量で考えないとダメってこと?
最後の部分は
「流体力学的には揚力の説明はできない。計算できるだけ」
と言ってるの?
864ご冗談でしょう?名無しさん:03/03/23 09:58 ID:???
>>862
密度が0の質点に加速度とか働く力という物理的意味を見出せる貴方は物
理を置き去りにしていないのだから、クッタの結果にそのままρ=0を代
入して揚力=0を華麗に計算してみせるんだね。

通過後に生じている下降速度成分は、局所的なものだろ。トータルで考える
場合は、通過前のものまで含めて考えなければならない。定常状態という
のが物理的に実現されるものだとしても、運動量の総和の時間変化はゼロ
だぞ。もっとも無限領域に広がる流れの場を移動しながら眺めているから
なんだが。
やはり、流体力学で具体的に流れの場を決定して、流れによって運ばれる
局所的な空気集団の運動量変化を追跡して、確かにそれは飛行機の重量に
匹敵する力積からもたらされたものだということを証明しなければならな
いぞ。そしたら、揚力は空気の運動量変化だということを皆が認めるよう
になる。(それが圧力を媒介としているかどうかは無関係)
だけど、その場合、飛行機が動くということを前提に書かなければならない
ので、座標系は地上にしっかり固定して考えないと。じゃ、頑張って。
865835:03/03/23 10:09 ID:???
>>853
私が847でクッタ条件という言葉を間違えて使っていた
かもしれません。境界層の外で速度の不連続があるという
意味で「クッタの条件を満たしてない」という言い方を
していました。すみませんでした。
境界層近似を考えると,境界層の外でせん断応力を働かせる
ということはないと思ったのですが,
836氏は境界層近似が適用されない一般的な場合の揚力の話を
されてたのかもしれません。
866835:03/03/23 12:12 ID:???
>>864
下図のように,翼の前方,後方に翼と同じ速さUで動く検査面A,Bを立て,
面を通る気流を考えると,
            ♂  |            |  \
            ♂  |  ┌――──┐ |  \
アップウォッシュ  ♂  |  └――──┘ |  \  ダウンウォッシュ
            ♂  |            |  \
               A           B
  (面Aに単位時間当たりに流入する運動量の鉛直成分)
  = (揚力) + (面Bから単位時間当たりに流出する運動量の鉛直成分)
の関係が成り立つと思います。
翼の循環Γの作る循環流が円形になるという近似をつかうと,
空気の密度をρとして,
   ρUΓ/2=L−ρUΓ/2
から,L=ρUΓとなるので,揚力=気流の運動量変化というのは間違い
ないと思います。
なお,計算過程は,
http://www002.upp.so-net.ne.jp/a-cubed/flight/chapter5.htmlの
「付記1: 運動量理論による揚力の計算」のようになるようです。
867ご冗談でしょう?名無しさん:03/03/23 12:25 ID:???
とゆーわけで、残る問題は
 「連続体近似の圧力差で揚力は説明できるか否か」
ってことだな。
これに的を絞った議論をお願いしますぜ。
868ご冗談でしょう?名無しさん:03/03/23 13:44 ID:???
>>866
まぁ、delivationの方法が完全なこじつけで、無理矢理結論から逆算し
た感ありありで、この理論を信じる奴は苦労することは明白。有害につき
早めにブックマークから外すように。
この作者の場合、循環と循環流の区別が多分滅茶苦茶なのでは?
円環流の速度成分を2面で区切ってその面で和を求めて引き算しても
単位時間辺りの運動量変化にならないよ(W
遠方の流れは遅いんだから。一定流速じゃない場で観測面を持ち出されて
議論されても....(W
翼周りの流体の質点の運動量変化を真面目に追跡しないと何も言えないぞ。
869745:03/03/23 15:59 ID:Si2JBhf5
>>862
‥‥あの、ちょっと確認なんだけど、

(翼によって引き起こされる周囲の流体の運動量変化)
=(翼表面の静圧を翼全周で積分したもの)
=(揚力)※垂直方向成分+(抗力)※水平方向成分

はいいんだよね?
870ご冗談でしょう?名無しさん:03/03/23 16:14 ID:???
>>868
件のページの作者のやり方がイマイチなのは、確かにそのとーりだな。
だが、クッタ・ジューコフスキーの定理で求まる揚力が、流れに垂直な方向の
運動量変化を表しているのは、式をちょっとイヂってみれば誰にでもわかる。

>翼周りの流体の質点の運動量変化を真面目に追跡しないと何も言えないぞ。

計算が厄介だとすぐに否定的になるのが流体屋の悪いクセだな (w
なにも「NS方程式を運動量の式で置き換えろ」とか言ってるわけではないぞ。

>>869=745
いいよ。
で、揚力の流体力学的な説明、まだぁ?
871745流体屋キリ号:03/03/23 16:43 ID:gDCUyP7x
翼の表面に、静圧の分布ができること。その原因は、

@翼が存在してること。
A流体が流れてること。

この2つが原因。はOK?

あと、870は864とは同一人物なの?別の人物なの?
864はなんとなく、前に「現実の静止系」という言葉を使った人に思えるけど‥。
できればコテハンで、アンチ流体屋1号、2号とでもしてほしいなぁ‥。
872ご冗談でしょう?名無しさん:03/03/23 17:04 ID:???
必死に粘着ご苦労様>>871
日曜も手淫と2ちゃんねるバッカですごすのはいかがなものかと。
873ぱだわん:03/03/23 17:09 ID:???
>>871
オッケーです。
「原因」とゆーより「前提条件」のような気もしますが、
本質的ではないので、説明を先に進めてください。
874ご冗談でしょう?名無しさん:03/03/23 17:11 ID:1jCK5Kjq
★男はココを見るべし★女と金とサンプルムービー★
↓↓↓↓↓↓↓↓↓↓↓↓↓↓↓↓↓↓↓↓
http://www.pink-angel.jp/betu/linkvp2/linkvp.html
875ご冗談でしょう?名無しさん:03/03/23 17:18 ID:???
飛行機が飛ぶのは気合ですよ!気合!
876745流体屋キリ号:03/03/23 17:39 ID:WqS8lRwD
>>ぱだわん氏

翼表面のある1点で、そこでの速度と圧力を、じーっと観察してる。
じーっと観察してる。‥‥‥変わらない。(836みたいなケースは除いて)
そこで、

翼形状or迎え角を変えてみた。→速度と圧力、同時に変化。
上流からの風速を変えてみた。→速度と圧力、同時に変化。

原因って言葉は広い言葉だけど、使ってOK?
877745流体屋キリ号:03/03/23 17:48 ID:WqS8lRwD
あと、この後進めるために、ぱだわん氏は、どれとどれの発言をしたのよ。
疑問点が何なのかわからないと、答えようがないので。よろしこ。
878ご冗談でしょう?名無しさん:03/03/23 18:06 ID:???
飛行機は飛ばない、飛んでるのは物理板の粘着の頭
879ご冗談でしょう?名無しさん:03/03/23 18:08 ID:???
飛行機は自分が飛べないことを知らない。
だから飛べるんだよ。
880ご冗談でしょう?名無しさん:03/03/23 18:09 ID:???
脳内お花畑
881ぱだわん:03/03/23 18:34 ID:???
>>876
これはダウトです。
実在気体には粘性がありますので、翼表面で流速を観測すれば常に0だと思います。
完全流体で近似するならば、速度と圧力が同時に変化することは観測事実としては
使えないと思います。

翼近傍の観測事実としてならばオッケーだと思います。
ここまでは前提条件と観測事実の提示ですので、原因はふさわしくないと思います。

>>877
そんなに多く発言してません。
知りたいのは、翼の上側が低圧になる理由です。元スレから読んでいますが、
最後まできちんと解説したレスはなかったと思います。
882ご冗談でしょう?名無しさん:03/03/23 18:40 ID:???
必死に粘着ご苦労様>>881
日曜も手淫と2ちゃんねるバッカですごすのはいかがなものかと。
883745流体屋キリ号:03/03/23 19:07 ID:fRWUa3HV
>>ぱだわん氏

782-785に漏れが書いた、低圧部発生理由の問題点は、

・翼後縁での回り込み問題を無視

でOK?
884ぱだわん:03/03/23 20:12 ID:???
>>883
疑問に思う点は少し違います。

まず>>782で『速度ベクトルは圧力勾配の低いほうに向きを変える』としています。
そして>>783の翼の回りで速度ベクトルが曲がる事実を説明するために、
>>784のような圧力勾配が生まれているとしています。
ここまでは理解できます。知りたいのはここから先です。
つまり、そのような圧力勾配の生まれる理由ですが、記述がありませんので
説明は完結していないと思います。

また、>>830では変数同士の原因と結果について考えるのは無意味としていますが
これは先の>>782の記述とは矛盾しているように思います。

どうぞ、もう一度わかりやすい説明をお願いします。
885745流体屋キリ号:03/03/23 21:30 ID:ItV9KOoT
>>ぱだわん氏

・引力が働く方向に物体が引っ張られる。
・物体が引っ張られる方向に引力が働いている。

みたいに、
・圧力が減る方向に速度(成分)が増える
・速度(成分)が増える方向に圧力が減る

の関係になってるんだよね。うーん‥。

左から流れてきた流体が、翼の背側に沿って流れるためには、
速度の垂直方向成分を次々と増やしていかないと、翼の背側形状に、
追随できない。そのために、翼の背側には次々と低圧領域が生まれる。

物体の背側に沿って流れるというのは、物体表面を流体が出入りできないという
条件から担保される、でどうでしょうか?
886ご冗談でしょう?名無しさん:03/03/23 21:50 ID:???
>>875
飛行機が飛ぶのは奇跡ですよ!奇跡!
887ご冗談でしょう?名無しさん:03/03/23 23:17 ID:???
>>886
奇跡は続かない。だから堕ちる。
888ご冗談でしょう?名無しさん:03/03/23 23:19 ID:gyUlQfJk
同意
889ぱだわん:03/03/24 00:01 ID:???
>>885
この説明はダウトです。

物体の背側に沿って流れるのが理由であれば、低圧が生まれなくても
流れをはさんで反対側に次々と高圧領域が生まれるでもいいと思います。
翼の背側が低圧になるのを説明するためには、別の理由が必要だと思います。
890横レス失礼1:03/03/24 00:47 ID:???
多数の質点を扱う体系では、原因と結果の間には実は明白な区別を付けることが出来ない。
でもある瞬間の結果が次の瞬間の原因となってる構造は変わらない。
しかし質点数が多くなるとある瞬間の結果がやや遅れた時刻の原因になっているかの如く
見える現象が出てくるからこそやっかいになる。

圧力と速度の関係は位置エネルギーと運動エネルギーの関係と同じで、その値のとれる
範囲が定まっていて相互に拘束されるというだけで、ある瞬間どういう分布になっているのか
を予測するのは不可能です。一方がわかれば他方がわかるというだけです。そしてある分布
がわかった場合、その分布が発生した正当な理由を求めることも不可能です。ただ、その
分布が安定であるかどうかはある程度わかるでしょう。
あと、翼の前後に平面状の観測領域を設けてその上で速度場を積分してその平面上の
運動量の合計を求め、前後の平面の差をもって運動量の差とする。これが揚力に等しい
のでは?という発想が見られましたが完全な間違いです。
891横レス失礼2:03/03/24 00:48 ID:???

翼の前(上流側)の平面の上の運動量の合計を積分で出すのは間違いではありませんが、
単位時間後の運動量変化は平面の質点が速度によってどのような曲面に運ばれたかを
考察しなければなりません。その曲面の上で積分しなければ正しい運動量変化が求まりま
せん。(定常状態であろうが、非定常状態であろうが無関係です。)その曲面を求めるのは
例え速度ベクトル場が具体的に与えられていたとしても、非常に難しいものになります。
あと無限領域での積分の問題ですが、これは翼が与えた力が無限まで瞬時に影響する
ことを考慮した考察で、完全流体を考える限りそれ自体は正しいのですが、実在流体は
有限の範囲にしか影響が及びません。その範囲で計算しなければなりません。但し粘性
を考えるとその有限の範囲がかなり遠くまで伸びるのではないかと考えられています。
その範囲を求めるのが非常に難しく、速度場の具体形が与えられていたとしても、求める
のは無理だと思います。ある質点を追跡してその運動量が変化したとしても、それが翼
から受けた力と、流体の内部応力との合力とが混在している状況だから難しいのです。
翼から受けた力がゼロである点もありますから、正確に計算するのが極度に難しいのです。
おそらく翼付近の空気は翼から受けた力に即応した運動量変化をするでしょう。遠く離れた
場合はその限りではありません。しかし量的には無視出来ないほど広い領域に分布している
でしょうから、翼周辺の運動量変化を見て揚力の大きさを見積もろうとすると大きな間違いを
起こすことになります。
892745流体屋キリ号:03/03/24 02:14 ID:7ZWJocnp
>>ぱだわん氏

「流れをはさんで反対側に次々と高圧領域ができる→×」

を説明できれば、

「流れをはさんで反対側は大気圧のままで翼表面に次々と低圧領域ができる→○」

はOK?
893ぱだわん:03/03/24 03:29 ID:???
>>892
言葉が足りなくて申し訳ないのですが>>889で言いたかったこととは少し違います。
命題「AならばB」と「BならばA」が両方とも真であればAとBが同値であるのは
ご存知だと思います。>>885で説明されていますように「流れが翼に沿う」ことと
「翼の背側が低圧になる」ことは同値になっています。
同値なものをならべても説明できたことにはならないと思います。
説明を完結させるためには流体力学的に別の理由が必要だと思います。
894ご冗談でしょう?名無しさん:03/03/25 04:02 ID:puhRITWF
>>893
ああなるほど。

・翼形状が与えられる
  @↓
・流れが翼に沿って流れる
  A↓
・流れが翼に沿って流れるための圧力分布が発生する

@とAと、どちらが問題?両方とも問題?

あと、漏れも流体屋の中では、「どうのつるぎ」に「ホイミ」覚えた程度なので、
その先のレベルに行ってる方々も、遠慮なく突っ込みorヘルプよろしこ。
895ぱだわん:03/03/25 08:35 ID:???
>>894
745流体屋キリ号さんですか?
@は観測事実ですので説明ではないと思います。
Aも観測事実の別の表現です。流れと圧力分布は同値なので
さらにその先の説明が必要だと思います。

・流れが翼に沿って流れる⇔圧力分布が発生する
  B↓
・○○○なので圧力分布が発生する

速度ベクトル以外で○○○の部分がわかれば揚力の説明は完結すると思います。
896ご冗談でしょう?名無しさん:03/03/25 16:02 ID:1Ti1mZ/C
>1
飛行機は落ちる力以上のスピード出せば落ちないんだ。
落ちないために出力を上げる。
ただそれだけ。
理論的講釈はここではやらん。
一銭の得にもならんから。
897ご冗談でしょう?名無しさん:03/03/25 16:33 ID:???
人工衛星ぢゃないんだつて
898ご冗談でしょう?名無しさん:03/03/26 01:34 ID:vU3TXX5g
遷音速で航行中の飛行機のはねの前縁を両手でつかんで空気抵抗に耐えることは可能か?
899745流体屋キリ号:03/03/27 02:04 ID:zRF1nfvs
>>895
失礼しました。894は漏れでつ。
>速度ベクトル以外で○○○の部分がわかれば揚力の説明は完結すると思います。
○○○の中を色々考えましたが、
@マクロには、
『境界条件がふにふになので‥‥』
 ↑予測される反論:方程式を解かないと分からないのは、説明したことにならない。
Aミクロには、
『速度ベクトルがふにふになので‥‥』(キシュツ)
 ↑予測される反論:同値のものでは説明にならない。

う〜ん、困ったな。ロジックとしては、『ふにふに』の中に何を入れても、
反論は同じになる訳よね。
ただ、○○○の中に、境界条件と速度のほかに、
入れられるものがないんだよね。

900745流体屋キリ号:03/03/27 02:22 ID:igLrbeGR
例えば、>>830でちょっと話をしてみた、
>バネ@の先にオモリ@つけて、オモリ@の先にバネAつけて、
>バネAの先にオモリAつけて、引っ張って離すと。
‥この系の挙動も、方程式解いてみないと、どう動くか分からない訳だけど、
バネの硬さ、オモリの重さ、オモリ@Aの初期位置で、いくつかのパターンというか、
代表的なモードに分類され、「こういう場合はこう」ということは言える。

流れの中に置かれた物体の周囲の流れ(≒物体に働く力)の場合は、>>784で説明した、
『速度ベクトルが時計周りに向きを変えていく場合は、揚力発生』
というのが、その、バネ-オモリ系でいう、モードというか、
「こういう場合はこう」に対応すると自分では思っているんですが、
う〜ん、これ以上はちょっと、今すぐ思いつかんね。スマソ。
901745流体屋キリ号:03/03/27 02:51 ID:yKTGkYz5
あと、自分でどうしても気になっているのが、やはり後縁の回り込み問題で、
教科書何冊かぱらぱらみたのだが、
「後縁の鋭さが、どれだけ回り込みや、揚力に影響するのか?」
っていうのがいまいちよく分からない。
後縁を鋭くすればするほど、後縁付近の境界層の厚みが0に漸近し、そこを回り込む
流れがどんどん完全流体のそれに漸近し、回り込むときに瞬間的に無限に近い
加速度が必要になり、そんなことはできないので回り込めない、というのは定性的には
分かり易いんだが、じゃあ、定量的にはどうなのかな?と思うと、
ヽ(>D<)ノ ウワアアアン! だよ。

バギマ、バギクロス使えるレベルの流体屋さん、知ってたら教えてクンクンです。
902ぱだわん:03/03/27 09:55 ID:???
>>899
結論がでたようですのでまとめます。

中学生「飛行機ってなんで飛べるんですか?」
流体屋「流体力学的には飛ぶ理由は示せませんが
     方程式を解けば圧力差が生じていることがわかります」

『翼の上下の圧力差』は揚力が生じている事実を言い換えただけで
説明にはなっていないことがはっきりしました。

中学生「飛行機ってなんで飛べるんですか?」
物理屋「空気に下向きの運動量を与えた力積の反作用が揚力です」

やはり揚力はこのように理解すべきだと思います。
運動量で理解するのに否定的なレスもありましたが、
それは単に計算しにくいということで間違いであるとは
言っていないと思います。

ありがとうございました。
903ご冗談でしょう?名無しさん:03/03/27 10:13 ID:???
>>900
確かにバネとオモリの挙動は運動方程式を解けばわかります。
でもオモリが振動するのはバネの伸びに比例する復元力が作用することで
理解できます。なぜ復元力が作用するのかは物性物理が回答を与えていて
バネを構成する原子が平衡位置からずれて力を受けるからです。

揚力に関しては流体力学ではこのような理由が示せないようですね。

>>901
これは板違いだと思います。少なくともスレ違いなのは確かでしょう。
904ご冗談でしょう?名無しさん:03/03/27 10:42 ID:???
>>903
901はドコの板なら適切だと思うの?
905ご冗談でしょう?名無しさん:03/03/27 11:24 ID:???
航空・船舶
906ご冗談でしょう?名無しさん:03/03/27 14:20 ID:???
中学生
「揚力の反動の一部がある程度広がりを持った周辺の空気に運動量を与えるのですね。
ありがとうございました。」
907ご冗談でしょう?名無しさん:03/03/27 14:21 ID:???
流体物理学者
「その通りです。力積や運動量は高等学校に入れば選択科目で習えます。
数学が多少得意であれば、覚えなければならないことが少なく、それでいて大学入
試ではかなり得点の差が付くので、腕に自信がある人は、皆物理を選択するようです。
でも、それを習っただけで森羅万象全てが全てわかったと錯覚しまう要素があるのです。
流体のように興味深い対象の深さを理解出来なくなる原因となっていることも悲しいこと
ながら事実です。」
908ご冗談でしょう?名無しさん:03/03/27 14:22 ID:???
中学生
「運動量とか力積という言葉を知らないので、よくわかりませんが、要するに速度が
低い場合は、飛行機の周囲の空気を下にえいやっと押し下げた反動で飛んでいる
わけではないのですね。」
909ご冗談でしょう?名無しさん:03/03/27 14:23 ID:???
流体物理学者
「完全に間違いだとも言い切れません。飛行機の速度によってはつまり、超音速機の
場合は、そういう飛び方に近いと言う事も出来ます。しかしかなり語弊がある説明だと
思います。通常は大気圧を無視することは出来ません。飛行機が飛ぶと、その周辺
の空気が目立って流動します。つまり実際の飛行機はそれなりに周囲の空気に
運動量を局所的に与えているわけです。しかしどのくらいの範囲の空気に与えている
のかを見積もるのは、速度が低い場合はかなり難しくなります。重力と向きが反対
で同じ大きさの揚力が空気に与えている力積が、どの程度空気の運動量に転換
されているのかは、速度が低い場合は実測してみなければわかりません。予想以上
に広い範囲に(しかし薄く)影響を与えている可能性があるので、実測は非常に
難しいのですが。少なくとも飛行機周囲の空気を押し下げただけでは重力に
対抗できるような反動は生まれません。」
910ご冗談でしょう?名無しさん:03/03/27 14:24 ID:???
中学生
「友達が、飛行機が飛べる理由は、羽で前方から来る空気を押し下げているからだよ
と言っていましたが間違いなのですか?」
911ご冗談でしょう?名無しさん:03/03/27 14:25 ID:???
流体物理学者
「確かに中学生にはちょっと難しい話ですね。だけど、本質的に一筋縄ではいかない
話という意味がわかりましたか?長い時間で見ると否定も肯定も出来ないわけです。
粘性は大体そういう理解でもいいのですが、粘っこさというよりも、引っ張りに対する
強さと捉えて下さい。餅みたいに一部を引っ張ると別の部分がへこむという性質が無く
一部を引っ張ると全体も追従する性質だと思って下さい。圧縮性とさきほど言いましたが
こちらは、押しに対する強さで、押しても容易に体積を変形しないことです。これらの
性質以外にも慣性という重要な性質があって、これも粘性と並んであるいはそれ以上に
圧力差が安定に維持されるのに重要な役割を果たしていると考えられています。慣性
とは、空気の流れが変化させられることに対して抵抗する性質のことです。密度が高い
とその性質が高くなります。」
912ご冗談でしょう?名無しさん:03/03/27 14:27 ID:???
中学生
「難しい話ですが、なんとなくわかるような気がします。要するにある飛行機が
ある瞬間重力に対して釣り合っていると、次の瞬間もその状態が維持されやすいわけ
ですね。その原因は、空気の粘性なのです、ね?ところで粘性って粘っこいことなんですか?」
913ご冗談でしょう?名無しさん:03/03/27 14:28 ID:???
流体物理学者
「その通りです。初期条件や加速履歴に強く依存するのです。
ただ普通の意味で飛行機が飛んでいるのを見て何故飛べているのかということに対する
疑問を持つのは自然です。人は良くそれを圧力差が生じているからと説明します。
実際そうなのですが、ではどうしてそのような圧力差が発生しなければならないのか
と言えば、実は多少やっかいな問題があるのです。ある瞬間に圧力差のお陰で落ちず
に同じ姿勢で飛べたという理由が次の瞬間にも圧力差が維持されている大きな原因に
なっているからです。さっき述べた通り、しかしこれは決定的な条件ではありません。
その圧力差が維持される保証は無いのです。しかし、飛行速度が適当に速ければ
例え揚力が低下しても、操縦する人間がそれを認識して、加速や迎え角操作による
揚力増強措置をするまでの時間程度の圧力差の安定維持は起きているわけで、空気
の流れの場がそうそう激変することが無いということが、大きな理由となっています。
これも、理論的にははっきりしたことが言えてはいないのですが、空気に粘性がある
ということが流れの場が激変することを防いでいるといわれています。同様にある程度
の時間は、流れの様相を維持するのに貢献しながらも後で突然場を激変させる原因
となる圧縮性(その極限が完全流体)もあるので様相は実に複雑です。」
914ご冗談でしょう?名無しさん:03/03/27 14:29 ID:???
流体物理学者
「流体力学的には飛ばなければならない理由も飛べてはならない理由も
理論的には実は良く分かっていません。実を言うと「飛べる」という判断は
時間スケールに依存する話なのです。時間スケールを変えると、飛べている筈の
飛行機の揚力が徐々に失われてしまうことや、逆に飛べない筈の船等の重いもの
に揚力が発生して.飛べてしまうとかの信じ難い話まで完全に否定することは出来
ないのです。
一見同じような状態なのに、時間が経つと様相が全く変わってしまうという
性質は、コマの運動のように非常に少ない状態変数で記述出来る系と異なって
よくある話です。
もっともこれは極めて理論的な話で、空気の摩擦(熱エネルギー転換)とかを
一切無視した話で、そうなるまでの時間が例えば宇宙の寿命の数十億倍かかる
とかそういう話です。
ある時間に限って飛んでいるのを見て、慣性による直線運動のように、永遠にそ
の状態を保ち続けるんだという先入観があると理解を妨げます。」

中学生「状態変数とか難しい言葉が出てきましたけど、理論的には飛行機が飛べて
も飛べなくても矛盾はしないんですね。」
915中学生:03/03/27 14:59 ID:???
飛行機って何で飛べるんですか?
916ご冗談でしょう?名無しさん:03/03/27 18:26 ID:???
>>907-915
なかなか大作なのだが、如何せん順番が入れ替わってしまっているようで
ちょっと読みにくいな。これを要約すると>>902になるのか。

何れにせよ、流体力学的には圧力差が生じる理由がわからない、とゆーことだな。
やはり、空気の運動量変化で揚力を理解するしかなさそう。測定に関しては、
大きな重量計の上にヘリを載せて少し浮かせる、とか元スレにあったぞ。
917ご冗談でしょう?名無しさん:03/03/27 19:13 ID:???
>>916
私には>>902を詳しくまとめたのが>>907-915だと思うんだが。
ま、運動量変化で揚力を想像するのは勝手だが、想像と現実は相当違うって
ことだね。
918ご冗談でしょう?名無しさん:03/03/27 19:51 ID:???
>>917
それは、運動量で揚力を計算するのは大変だってことだろ。
それとも、ヘリが離陸した途端に重量計のハリはゼロを指す
ってことかい?
ま、圧力差で揚力を説明できない流体屋ってのは確定だが。
919ご冗談でしょう?名無しさん:03/03/28 04:48 ID:???
>>915
つまり918は圧力差で揚力が説明出来ます。というわけで、翼の上下に慣性が
原因で安定した圧力差が出来るからなんでしょうね。圧縮性がその安定化に
大きく寄与し(速度が高すぎると妨害)、粘性が安定化を邪魔しつつも、
不安定化を妨害するということで理解出来るというわけですね。
920ご冗談でしょう?名無しさん:03/03/28 08:01 ID:???
ありゃりゃ、断末魔の悲鳴ってカンジだな>流体屋
一番肝心な「慣性が原因で安定した圧力差が出来る」ってのが説明できないわけだろ。
さらに、普段はジャマ者扱いして無視してる圧縮性や粘性が安定性に寄与するって
言われてモナー。
もしも「粘性があると○○○なので圧力差が生じる」がわかれば見込みはあるぞ。
しばらく頭を冷やしてよく考えてみるとイイかもしれない。でも連続体近似じゃ無理だな。

流体力学ってのは、現実の系をザックリと連続体で近似することで、流体としての
挙動を精密に計算するために作られたわけだろ。だから現象の説明ができなくても
何の問題もないと思うが、イヤなのか?
921745流体屋キリ号:03/03/28 08:22 ID:OPV9pYXL
>>902 ぱだわん氏
物理屋「空気に下向きの運動量を与えた力積の反作用が揚力です」
通行人「そうですか。‥ていうか、そりゃそうだろうけど、
    飛行機の翼に、何で下向きの運動量ができるんですか?」

‥との問いが当然帰ってくるでしょうが、これにはどう答えるのでしょうか。
1行目は、ただ、作用反作用の法則を言ってるだけですよね?
922ご冗談でしょう?名無しさん:03/03/28 09:06 ID:HfXuZdjJ
物理屋の議論って、教科書に載ってない事だったらこんなに延々と議論が続くのね・・・
逆に教科書に載ってる事だったら、それ以外の事を言った人をフルパワーで叩くだけだし
923ご冗談でしょう?名無しさん:03/03/28 09:34 ID:???
>>919
ちなみに圧縮性や粘性とかは、別に連続体も有限質点モデルでも、無限離散
個の質点モデルでも、速度ベクトルの相互従属性の傾向なんだから関係なく
表現出来るでしょう。
例えば粘性は2つの観測質点の位置をx1(t),x2(t)
速度をそれぞれv1(t),v2(t)と置いた時Const(v1(t)+v2(t))/|x1-x2|なる
力が等しく2つの質点にかかるとかね。
圧縮性のほうは、任意の
異なる質点i,jの間に|xi(0)-xj(0)|=|xi(t)-xj(t)|の関係さえ仮定しなければ
自然に満たされる。
モデルが連続体であるか否かは関係ないよ。
924ご冗談でしょう?名無しさん:03/03/28 10:45 ID:???
>>922
淘汰のための地道な努力
925ご冗談でしょう?名無しさん:03/03/28 16:46 ID:???
>>921
連続体近似が現実だって思い込んでるクモったアタマだと、気体分子って言葉すら
思い出せなくなるらしい。元スレ読んで勉強しておいで。

>>923
あのね、モデルにおける表現の仕方を問題にしてるんじゃないんだよ。
どーして空気に粘性があるのかを連続体近似で説明できるのかい?
926ご冗談でしょう?名無しさん:03/03/28 17:24 ID:???
>>925
どうしてそんなことが問題になるのか?本筋に関係のある話なのか?
確かに流体の方程式の粘性項は数学的にラクになるように簡単化され
たもので、現実との対応については?なところもあるけど、流体を考
える上で量の多少はあれど(ゼロも在り得る)粘性を考えるのは自然。
むしろ粘性が無い状況はかなり理想化している。
粘性があることを基本仮定としている流体近似(連続体近似)で粘性
そのものの存在を出せるわけが無い。分子論を持ち出しても、この
点は大して変わらないはず。所詮は分子間力に還元するんだろうからね。
927ご冗談でしょう?名無しさん:03/03/28 23:48 ID:???
>>926
分子論(分子運動論だろ)と連続体近似が「大して変わらないはず」かよ。
連続体近似マンセーにはいい加減ウンザリだな。
マトモな物理教育を受けてないのがバレバレだぜ。
それとも「分子=流体の微小要素」とゆーオチを用意してるのか?
928ご冗談でしょう?名無しさん:03/03/29 01:50 ID:XL/FTVxF
>>921
>飛行機の翼に、何で下向きの運動量ができるんですか?

翼が斜めに取り付けられてるからさ。
進行方向に対して後方が下がってる形でね。
これで空気を押し下げる事ができる。

もしこれで空気を押し下げる効果を生まないなら、扇風機がいくら回っても風を送る事ができない。
929745流体屋キリ号:03/03/29 03:21 ID:QLI+RORm
>>928
中学生「エンジンはどうして動くんですか?」
エンジニア「まず、ピストンが空気を圧縮します。ピストンの中は、
高温高圧になります。そこで点火プラグで燃料に火をつけ‥‥
‥‥でも未だに燃焼現象などには、分からないこともたくさんあるんですよ。」
似非物理屋「それは説明になってない。エンジンが動くのは、
熱力学第1法則で動くんだ。」
エンジニア「(‥そりゃそうだけど、この人何言ってるの?)」
中学生「(何?熱力学第1法則って?この人何言ってるの?)」

だと思うよ。
930745流体屋キリ号:03/03/29 03:29 ID:QLI+RORm
>>920 ○○○‥‥「コアンダ効果」じゃないよね。
でもとっても不安なので、キミ自身の言葉で聞きたい。

「コアンダ効果」

じゃないよね。‥ていうか、デビッド・アンダーソン?(だっけ?)を、
そのまま信じてる人は、いくらなんでもいないよね。確認だけど。
931暗黒大将軍:03/03/29 03:41 ID:OtBleNVB
932745流体屋キリ号:03/03/29 03:45 ID:QLI+RORm
>>928

>翼が斜めに取り付けられてるからさ。
>進行方向に対して後方が下がってる形でね。
>これで空気を押し下げる事ができる。

物理屋さんのみなさんに質問。
この人は、物理屋さんですか?
それとも、通りすがりの、物理に縁もゆかりもない人ですか?
Yes、Noで答えて下さい。
933ご冗談でしょう?名無しさん:03/03/29 04:36 ID:???
.>>930
じゃないよ。ってゆーか、ttp://www.allstar.fiu.edu/aero/airflylvl3.htm(必読HP)を、
間違ってると思う人は、いくらなんでもいないよね。確認だけど。

>>932
通りすがりの流体屋だろ (w
934745流体屋キリ号:03/03/29 05:07 ID:V9v87KGG
>>933
漏れは英語読めないんで、何が書いてあるのか分かりません。
キミがそれを理解して、それを信じてるんなら、自分の言葉で、要点を説明してよ。

それと、キミは物理屋の一人のようだけど、キミの仲間の発言、
902>>物理屋「空気に下向きの運動量を与えた力積の反作用が揚力です」
これってどうよ。これは、物理的に何かを説明したことになってるのかい?
935745流体屋キリ号:03/03/29 05:19 ID:V9v87KGG
(続き)
だってさ、考えてみてよ。
ある特定の境界条件(速度ベクトルが右回りになるとか)で発生する
現象を考察してるのに、その境界条件の性質をすっとばして、
何を説明したことになるのよ?すっとばしすぎだよ。なんぼなんでも。

「エンジンは熱力学第1法則で動いています。」

これとどこが違うの?違いが分からないので説明して下さい。
936ご冗談でしょう?名無しさん:03/03/29 07:22 ID:???
937ご冗談でしょう?名無しさん:03/03/29 08:42 ID:jfGyw0kv
マタ中学生かよ。
この宇宙で起こることはなんでも、アホの頭で理解できることじゃないと
いけないのかよ。
938ご冗談でしょう?名無しさん:03/03/29 09:22 ID:???
>>927
連続体近似が例え否定されたからといって、それはあくまでも一つの連続体近似
が否定されただけでしょ。連続体近似の否定に連続部分が存在しないということ
は証明すら出来ないはず。
939ご冗談でしょう?名無しさん:03/03/29 09:54 ID:???
>>938
流体の粘性と分子間力の間には似ていて超えられない相違があって
分子間力で粘性を導き出すことが出来るという前提は取りあえず捨てた
ほうがいいかもね。なぜなら、流体の粘性は流線に沿ってその平行性を
高めていこうという傾向のこと。境界を持つ場合境界付近では粘性
は発現しないと考える。自由性・慣性・圧縮性と並んで流体の大きな
特徴ね。流体の場合、その部分要素にNewtonの名前が冠された力学の
基本法則が適用出来るかどうかも仮定されていない。(否定はされて
ないけどね)
揚力にとっては、自由性と慣性が基本的要因だが、適度な圧縮性があって
流体の速度ベクトル変化が遅れ効果を持たなければ不安定になる。
しかし有り過ぎてもまた不安定になる。貯金と似ているわけね。
その安定領域を粘性は拡張する効果を持つと考えられてきたわけね。
自動車で例えるなら、サススプリングと、オイルダンパの関係に似
ているわけね。
従来は粘性は半永久的に減衰振動効果を与えると考えられてきたんだけど
ある条件では強制振動効果を与えるんじゃないか?とまで予想されてる。
粘性が分子間力の統計だけで説明出来ない一つの象徴的例だね。
940ご冗談でしょう?名無しさん:03/03/29 10:44 ID:???
>>930
人工衛星が地球の周りを回っている時、地球は人工衛星から引力を
受けるわけだが、それもまたコアンダ効果なんだよね。
941ご冗談でしょう?名無しさん:03/03/29 11:24 ID:???
>>927
流体力学は思考モデルとして必ずしも連続体モデルだけを取ってるわけでも
ないでしょ。
むしろ有限集団の力学が出来て以来、それ以外の一般的な多集団の力学(剛体の力
学も含むよ)がいつのまにか連続体の力学と同一ジャンルにあると誤解混同されて
るような感じ。
あなたが言っている分子運動論ってのは、有限集団の力学なんだろうね。多分。
連続という滅茶苦茶強い仮定を入れたモデルを使って議論する場合、
連続性の仮定が導き出せる結果にどの位影響しているかを調べてからやらないと
トンデモなものになっちゃうわけだけど、有限集団の力学も全く同じだろうね。
有限集団も連続体もどちらも数学とは相性が良くて式に乗るんで良く使われる
けど、どちらも所詮ラディカルなもので、現実の極端なモデル化に過ぎない。
現象を目の当たりにして無理にそのどちらかに当て嵌めようとすることにこそ
問題は無いのかな?
942ご冗談でしょう?名無しさん:03/03/29 11:55 ID:???
>>936
読んでみると、出だしは口当たりが良い。中間までは結構
目から鱗が落ちたような気がする。空洞(void)が出来てそこに
大気が落ちて下降流になるって辺りまでは問題が無い。
だけどコアンダ効果とか粘性とかCirculationが出てくる辺りから
流体力学とNewton力学を混同しているトンデモさが現れて
結論は完全に逝っちゃってるね。
943ご冗談でしょう?名無しさん:03/03/29 12:31 ID:???
流体屋はよほど連続体近似が大切らしい。ワケワカメなヤシも乱入してるし。

>>939
書いてあることの真偽は定かではないが、分子間力で説明できないとしたら
連続体近似で粘性を扱うのが限界だな、と思うのがフツーの物理屋。

>>941
分子運動論すらマトモに理解できなくて、妄想で補完かよ。トンデモの典型だな。

>>942
自分のアタマではページの内容が理解できません、とコクハクされてモナー。

「自分の理解できないことは間違いだ」とゆーマヌケなスタンスのヤシは、
これまでにも多数出没してるから珍しくは無いが。
944ご冗談でしょう?名無しさん:03/03/29 13:23 ID:???
>>943
分子運動論は連続体近似を一部含むとでも言っているのか?
>>941
力学=有限自由度の力学+(有限とは仮定しない自由度を持つ力学)
流体力学=力学ー有限自由度の力学
流体力学∩連続体力学≠φ
流体力学∩non 連続体力学≠φ
力学⊃連続体力学
力学⊃有限集団の力学
(これに>>943は有限集団の力学⊃分子運動論 って仮定を付け加えたいらしい。)
この思考系譜図は概ね正しいと思う。
ここで流体力学の仮定で物事を見る場合、どうして分子運動論の
不理解だとか連続体近似盲信者というヒボウにつながるのかが良く
分からんな。それこそ妄想だな。
>>942
ベルヌーイブランドの法則を使った説明では、大気圧と圧力の関係が
ぼかされている。実は問題アリアリなんだけど。相対的な圧力差が
あれば、絶対大気圧は無視しても良いという感じを与えるんで問題な
わけだ。その欠陥を逆手に取って、流体力学の用語である粘性を
トライポロジーとかの分野で出てくる粘着力という似ていて全然
異なる概念を意図的に混同させて、翼の上に出来ようとしている空洞
に粘着力で空気が落ち込んでその反動を翼が受けるという説に持ってい
こうとしているね。
945ご冗談でしょう?名無しさん:03/03/29 16:27 ID:???
>>944
ベルヌーイの法則を持ち出してきて圧力差で揚力が理解できる
と、いまだに思っているあたりが、連続体近似妄信者バレバレ (w
流体力学による説明以外は全く受けつけないのも信者のアカシ。

別に連続体近似が間違っているとゆーわけではないし、流体力学が
大成功を収めているのも周知の事実。
でも、それは流体の挙動を精密に計算できるようになった、とゆー意味であり、
流体力学で現象が根本的に理解できるようになった、わけではない。

そう指摘すると、血気盛んな「青年部」の連中が「流体力学は偉大なり。
連続体近似マンセイ。従わぬものには天罰を」とばかりに迷い出てくる、
が、揚力の説明がうまくできずに敢え無く殉教を遂げる。
第3スレはそんなカンジだったな。

「翼上下に圧力差が生じるから飛行機は飛びます」
ってのは、現象の記述としては間違いではないが、流体力学的にはその理由をちゃんと
説明できないのだから、それを吹聴して正しい理解を阻むのはもう止めましょう。
わかるかなぁ?
946ご冗談でしょう?名無しさん:03/03/29 16:34 ID:???
>>945
>それを吹聴して正しい理解を阻むのはもう止めましょう。
これは
 「揚力は圧力差」を吹聴して正しい理解を阻むのはもう止めましょう。
だな。
947ご冗談でしょう?名無しさん:03/03/29 17:15 ID:???
745流体屋キリ号 さん
過去レスにもどりますが,
>>889に対する答えとして,
鉛直軸をzとし,上方をプラスに取ると,
z→+∞で,気圧P→大気圧・・・(A)
かつ,翼上面からzが増加すると,気圧が単調減少するという条件(B)が成立する
(Bが成立しないと,翼上方に孤立した高圧部が存在するが,そのような物理的に
区切られていない高圧部は安定的に存在し得ない)ため,
翼上面で低圧になるしかないという論法はだめでしょうか。
948ご冗談でしょう?名無しさん:03/03/29 17:16 ID:???
>>945
流体力学=非有限集団力学
で現象が根本的に理解出来ないというのなら、有限集団の力学≒分子論で
現象を理解するのも同等に困難だろうな。
また、流体力学で精密な計算が良く出来るようになったというのは、
半分本当で半分嘘。実際は計算は極めて難しい。有限集団の力学で近似計算
しているのが実情。ある意味、流体力学を考察するプロセスで、有限集団の
力学の計算技術が高まり、それが有限集団の力学の理解を深め、さらには流
体力学の理解を深めることに役立っているというのは成り立つだろうね。
949947:03/03/29 20:19 ID:???
訂正があります。ごめんなさい。
>>948で,
>かつ,翼上面からzが増加すると,気圧が単調減少するという条件(B)が成立する
                            ~~~~~~~~→単調増加
950ご冗談でしょう?名無しさん:03/03/29 21:09 ID:???
>>948
流体力学で現象が根本的に理解できないというのは本当。
分子運動論でも同等に理解できないというのは嘘。
分子運動論を理解していないというのが真相。
951ご冗談でしょう?名無しさん:03/03/29 23:03 ID:yHSoWxTN
>>933
>>936
大間違いだと思う。
揚力は、後縁の吹き下ろしだけでなく前縁の吹き上げによっても発生している。
もし、吹き上げによって負の揚力が翼前半部に生じるなら、
循環という物理量の性質から、翼全体で揚力は発生しない筈。
コアンダ効果で翼上面の空気が翼上面に沿って流れるというのも間違い。
もし、これが本当なら、粘性の影響が無視できる境界層の外側の流れが、
なぜ境界層に沿って流れるのか。
952ご冗談でしょう?名無しさん:03/03/29 23:27 ID:???
素人からの疑問なんですが。

なぜピッチャーはカーブやシュートを投げることができるの?

っていう質問でも同じような議論になるんでしょうか。
953ご冗談でしょう?名無しさん:03/03/30 01:36 ID:???
>>951
青年部の斬りこみ隊長が自爆しちゃってるなぁ…
翼前縁部は圧力が高くなってるので「負の揚力」でイイのだよ。
翼の周りではトータルで「吹き下ろし」になってるから揚力が発生するのだよ。
「境界層」の外側でも粘性はあるのだよ。計算するときに無視しても大丈夫なだけ。

>>952
同じだよ。
ボールの場合は、流体屋の大好きな渦ができる理由が直接的だから
話は早いけどね。
954ご冗談でしょう?名無しさん:03/03/30 03:33 ID:???
分子運動論熱狂的信者が居るスレはここですね。
955ご冗談でしょう?名無しさん:03/03/30 05:42 ID:???
揚力は生命に似ていて、発生してもしなくても力学の基本法則には矛盾
しない。しかし揚力の「延命」を助ける因子が存在するので、一度発生
した揚力は「暫く」維持される。ずっと維持されるのか、或いはその因子
を増強してやることが延命効果を増強することにつながるのかは未だ不明
(大方の予想では否定的)
万有引力などのような永劫に渡って働く力とは似ていて異なるもの。
956ご冗談でしょう?名無しさん:03/03/30 07:47 ID:UQzPsgU9
>>953
確かに粘性は境界層の外側にも存在するが、
速度勾配の極めて小さな境界層の外側においては、
粘性によって発生するせん断応力は、流体の慣性力や圧力勾配に比して極めて小さい。
だから、その説明はおかしい。
吹き上げで、負の揚力が発生するなら、完全流体の二次元円筒周りの循環のある場合の流れでは、
揚力は0ということになる。
もっと言えば、吹き上げと吹き下ろしの大きさが同じ流れの場においては揚力は発生しないことになる。
吹き上げは、翼が流れの中に置かれることで、
翼周りの流れが流体間の相互作用で変化する結果として発生しているのであって、
翼自体が、流体を跳ね上げて発生させているのではない。
957745流体屋キリ号:03/03/30 08:27 ID:+/PVRlze
基本的なことだけどね、流体の「粘性」ってのは、この訳語がイマイチなんで
誤解をあたえるんだけど、

「粘ってひっつく性」‥× じゃないからね。
「摩擦性」‥○ のことだからね。

粘ってひっつく性だと、例えば、宇宙空間に石ころ置いて、スプレーで空気を吹き
付けたら、石の回りに空気がベタッとくっついてしまう訳だけど、んなわけない。

でもな〜。デビッド・アンダーソン?(だっけ?)の主張を、日本語で紹介している
HPはいくつか見たけど、彼は「粘ってひっつく性」と思ってるんだろうな‥‥。
((((( ゚Д゚;)))))))ガクガクブルブル
958ご冗談でしょう?名無しさん:03/03/30 09:47 ID:???
流体屋の最後のジハードかよ。

>>954
複雑系のトレンドも下火になったし、要素還元論は多くの物理屋にとってフツーのことが。
って書いても流体屋には意味不明か…

>>955
「流体力学的には」未だに不明、ってそのとーりだな。

>>956
「極めて小さい」から計算するときには「無視」できるんだろ。
翼の周りで「吹き上げ=吹き下げ」なら揚力は発生しないが、何か?

>>957
何だよ、キリ号、オマエは流体力学の問題を解いたこと無いのかよ。
流体力学の経典には、それこそ天下り的に「粘着の条件」を使えって
書いてあるぞ。物体の表面では流体の相対速度はゼロになるって
実験でも示されてるよな。
まぁ、連続体近似マンセーのヤシは、流体にどーして粘性と粘着の条件が
あるのかを考えられないので、仕方ないけどな。
あと、真空中の石の表面には空気の分子(酸素とか窒素とが)が吸着してるが、
これも知らないんだろ。
知らないこと、理解できないことは否定するって、お○な流体屋のセオリー通りの
思考パターンだぜ。
959ご冗談でしょう?名無しさん:03/03/30 10:19 ID:???
>>958
吹き上げ<吹き下げ?
密度は同じだろ?
速さがトータルで
吹き下げの速さ>吹き上げの速さ
にならなけりゃ、運動量はトータルで下向きにならない。
どー考えても、上下方向の体積収支は赤字になるな(W
つまり、翼の前後で、空気の積層量が変わってしまうことになる。
下向きに流れた空気はどこに消えたんだ?地球の中にめり込んだか?
まさか、地上の空気は無限の高さまで続いているとメルヘンチック
な神話信じているんじゃないだろうな(W
960ご冗談でしょう?名無しさん:03/03/30 11:00 ID:???
オマイの吐き出した臭い息は無限の彼方に到達してるでツカ?
961流体屋キリ号:03/03/30 11:08 ID:Im/XxM+Z
>>958
ちょっと待て。ホントに空気がモチみたいに、物体に引っ付くと思ってるの?

あのね、物体の表面に近づくにつれて、速度がゼロに近づくというのと、
モチみたいに引っ付くというのはだな、根本的に…(ry

モチみたいに引っ付くっていう現象は、たいていの場合、分子が糸状に長ーく
なっててだな、それがからまりあって外れないように…(ry

あと、物理板で宗教用語はヤメレ。

>>947 同意。ご説明ありがとうです。遅レススマソ。
962ご冗談でしょう?名無しさん:03/03/30 11:41 ID:???
この期に及んでまだ解脱できないのかよ (w

>>961
次なるアイテムはモチだってさ。
重合した高分子と気体分子の挙動が同じなわけないが、それが何か?

あと、物理板で連続体近似はヤメレ。
963ご冗談でしょう?名無しさん:03/03/30 11:46 ID:???
>>958
粘着の条件は,流体の,物体に接する部分と物体とで速度差が生じない
という条件で,その由来は,仮に両者に速度差があった場合,流体分子と
物体の原子とで弾性衝突をして,速やかに速度差を解消することから
くるもの思います。
それに対して,吸着は,吸着物質と被吸着物質との間の分子間力に
より生じ,せいぜい原子の大きさのオーダー(オングストローム単位)
の距離にしか働かないものと思います。
964ご冗談でしょう?名無しさん:03/03/30 12:14 ID:UQzPsgU9
>>958
なぜ、計算で無視できる粘性によるせん断応力が、
境界層の外側の流れにおいて支配的な役割を持つのか理解不能。
もう一つ、吹き上げと吹き下ろしが同じ流れにおいても揚力は発生する。
実際、936の図6地上の観測者から見た翼のまわりの空気の動きのベクトル図を見ても、
吹き上げと吹き下ろしが同程度であることが分かる。
965ご冗談でしょう?名無しさん:03/03/30 13:27 ID:???
>>963
弾性衝突じゃないけどな。
で、速度差を解消したした流体分子が物体に吸着しないのは
カミのゴカゴかい?

>>964
時速900キロで飛ぶ飛行機の中で男はトイレに行こうとして歩き出した。
が、「地上から見れば飛行機の速さに比べて歩く速さは無視できる」
と思い歩く速さをゼロにした…
下腹部の切迫感、永遠にたどり着けないトイレまでの道のり。
脂汗を流しながら、男は己のマヌケさを悟った。

ダウンウォッシュの写真は見えないらしい (w
おまいら、いつまで「気違い元スレ27」の相手してるんですか?
Kittyを相手にしても疲れるだけだし、何の進展もないぞなもし。
967ご冗談でしょう?名無しさん:03/03/30 14:09 ID:???
とゆーことにして、
マヌケな流体屋は自らを省みることもなく
世の中にウソを撒き散らしながら生きてゆく。
968ご冗談でしょう?名無しさん:03/03/30 14:58 ID:???
>>965
吸着するのは,分子間力が働く,せいぜい物体から原子の大きさの
オーダーの距離内にある分子だけで,それより外側にある流体が
曲がる理由の説明にはならないと思われます。
次に,運動量保存則については,
(翼へ流入する気流の運動量 P)
=(翼が受ける力積 2P)+(翼から流出する気流の運動量 -P)
とすべきと思われ,貴殿の主張のように
(翼へ流入する気流の運動量 P)+(翼から流出する気流の運動量 -(P+ΔP))
+(翼が受ける力積 ΔP)=0
とはならないと考えます。
ただし,三次元の有限翼では後流渦から生じるダウンウォッシュがあります
が,それは平行流を水平から少し下向きにする効果を持ち,結果,
揚力を鉛直上方から少し後に傾け,傾いた揚力の水平後ろ向きの
成分が誘導抗力となります。
969745流体屋キリ号:03/03/31 03:54 ID:Z+3QVvqO
正直、デビッド・アンダーソンは許せない。
でも、デビッド・アンダーソンにだまされた人たちは…。。・゚・(ノД`)・゚・。
もう、どんなことをしても、こっちに戻ってはこれないのか…!

科学の世界には、すごいモノを見つけてくる山師が、たまに出現する。
でも、その山師のふりをしたホラ吹きが、もっと多い頻度で出現する。
そういうホラ吹きは、永久機関とか、フリーエネルギーとか、
ギラギラしたのが多かったんだけど、流体力学みたいな、
こんな地味な分野にも出てくるとは…。

スレ違いゴメンネ。ほんとにゴメンネ。。・゚・(ノД`)・゚・。
970ご冗談でしょう?名無しさん:03/04/01 01:01 ID:pPh4Nety
結局、教科書に載ってない事に関しては物理屋は馬鹿って事ですか?
971ご冗談でしょう?名無しさん:03/04/01 01:18 ID:???
>>970
教科書に書いてある事も理解出来ないようなどっかの連中よりはましでしょう
972ご冗談でしょう?名無しさん:03/04/01 02:42 ID:???
格言

単純なことを複雑にしか考えてしまう人=凡人
複雑なことを真に複雑に考えられる人=秀才
単純なことを真に単純に考えられる人=天才
真に複雑なことを単純に考えてしまう人=どあほう

アンダーソン派は天才派なのか、ただの「どあほう」か?
973ご冗談でしょう?名無しさん:03/04/01 03:38 ID:???
>>966
その元スレ27にヤラレたのがトラウマになってて
揚力から逃避してるの?

>>968
流体分子間に力は作用しないのかい?
運動量に関しては何を主張したいのか意味不明だぜ。

>>969
「デビット・アンダーソンは理解できない」って、「正直」に書けよ (w
まぁ、キリ号はカナーリ前に臨界を越えてるから、もう休んでてイイよ。

>>972
たてた方程式を解くのに四苦八苦し、現象の理解もママならない、
そんな流体屋はマヌケなのか、ただの「どあほう」か?


しかし牛タン+テールスープ+麦飯の組み合わせはイイ。
スレ違いゴメンネ。ほんとにゴメンネ。でも物理屋ならわかるよネ (w
974ご冗談でしょう?名無しさん:03/04/01 08:02 ID:???
真に複雑なことに含まれる貴重な情報を無理矢理落として
「理解」してしまったがために、その情報を二度と認識
出来なくて四苦八苦するヤシよりマシと思われ。
975ご冗談でしょう?名無しさん:03/04/01 09:34 ID:???
>真に複雑なことに含まれる貴重な情報を無理矢理落として

複雑な気体分子の運動の情報を無理矢理落として
単純に考えている連続体近似のことだろ。
つまり、流体屋は「どあほう」であるが流体屋よりマシ、
とゆーことを言いたいわけだな。
まるで流速と圧力分布の堂々巡りの「説明」みたいだぜ (w
976ご冗談でしょう?名無しさん:03/04/01 09:47 ID:???
>>975
流速と圧力分布の堂堂巡りがヤなら、ポテンシャル場での分子の
運動はもっとヤな話になると思うが。(W
977ご冗談でしょう?名無しさん:03/04/01 14:38 ID:qko9GqvA
>>973
結局、デビッドアンダーソン教の信者には、間欠的に水平方向から飛んで来るパチンコ玉が、
傾けた平板によって斜め下に方向を変えられることで、揚力が発生しているという誤ったイメージしかないということか。
978ご冗談でしょう?名無しさん:03/04/01 19:45 ID:???
>>976
ボクは物理がニガテなんです、ってカミングアウトされてモナー (w

>>977
なるほど、マヌケな流体屋は分子運動論に対してそーゆー誤ったイメージを
抱いているのか。絶句。
別にアンダーソンを擁護するつもりはないが、彼の主張はパチンコ玉ではないぜ。
日本語訳を読んでも解らんのか?
979ご冗談でしょう?名無しさん:03/04/02 00:01 ID:d2gsyRQS
>>978
その例えが単純すぎるというなら、次のイメージか。
水平方向に間欠的に飛んで来るパチンコ玉を跳ね上げる斜めに置かれた第一の平板と、
それの斜め上方に水平方向に置かれた第二の平板からなる系。
それとデビッドアンダーソンの教義と分子運動論とは何か関係があるのか?
おれは分子運動論について一言も言及したおぼえはないが。
980745流体屋キリ号:03/04/02 01:38 ID:e9FwZNQN
まあ、このスレも残り少なくなってきて、もうそろそろ最後の機会
と思いますので、ぜびその「分子運動論」というのがどういうもの
なのか、お教えいただきたいと思います。

( ´D`)ノ >>978 サン ヨロシクデス!
981ご冗談でしょう?名無しさん:03/04/02 03:03 ID:???
>>979
助手「博士、やりました。ついにデムパ゚誕生の瞬間を捕らえました。」
博士「うむ。確かに。妄想に妄想を重ねそれを誇らしげに披露しておる。真性じゃな。」
助手「でも、なぜそのように次から次に妄想が涌いてくるのでしょうか?」
博士「それはじゃな、彼らの学習能力は極端に低いのじゃよ。特に論理的な思考が
    苦手なようじゃ。折角正しい情報が与えられているのに、それを理解して吸収し
    さらに再構成して応用することができんのじゃ。」
助手「それだけでは妄想が沸いていくる原因にはならないと思いますが…」
博士「そう、彼らにも人並みに物の理を究めたいという欲求があってな。それを満たす
    ための知識が圧倒的に不足している場合に妄想で補完してしまうのじゃ。」
助手「考えたことの真偽を疑うことはしないのでしょうか。」
博士「それができないからこそ妄想なのじゃよ。ある意味、可哀そうな連中じゃな。」
982ご冗談でしょう?名無しさん:03/04/02 03:07 ID:???
>>980
第3スレ的には

 流体屋は揚力を理解していなかった。圧力差では説明になっていなかった。

で終わって良いんじゃないの。キリ号のおかげでそれが明白になったわけだし。

分子運動論に関しては、アンダーソンのページの内容すら理解できないヤシには
難しすぎるね。ホントに知りたければ、教科書に載ってるから買って勉強するんだな。
多分「はがねのつるぎ」レベルでも歯が立たないかも。
でもスレに貢献したから、ちょっとだけヒント。
気体分子間には離れていると引力、接近すると斥力が作用するのはイイよね。
確認だけど (w
983ご冗談でしょう?名無しさん:03/04/02 10:59 ID:???
>>982
連続体ではないし、パチンコ玉の集合でも無い空気のイメージは
多くの流体学者のイメージそのものだぜ。ただ連続体近似も一つの手段
だし、この場合意外と良い近似となってる示唆を与えてくれるというとこ
か。
ここで流体屋を批判してる奴は、実は流体屋そのものだったりして。
ま、それはいいとして、流体の一部が独立して翼が落ちる時に地球から
貰うべき運動量を「盗んで」代わりに落ちてしまう(=ダウンウォッシュ)
ので、翼は落ちることが出来ないというのが、アンダーソン論文擁護者
の意見。何故空気が翼から運動量を盗むことが出来るのかと言えば、
つまるところ翼と空気との間に引力が働くからだと。失速という現象を
考えると、それは常に起こるとは言えない。ある場合において、引力
が働くんだと。翼のエネルギー消費量からみて、少ない量の空気が
運動量を盗む(つまり早いダウンウォッシュになる)んじゃなくて
相当に巨大な量の空気が翼から空気を盗んでゆっくりとしたダウンウォシュ
になる(質量が大きければ運動量は奪っていても、速い場合程エネルギーを
奪ってはいない)
ま、現象を見る限りこれを信じたとしても取り敢えずは矛盾を出しようが
無い。ただし、空気と翼が触れていない場合に相互に力が働かないという
前提と、空気は極度に密度が異なる面において、猛烈な拡散力(膨張力)
が働くという重要な物性があることを認めると別の視点もあるんじゃない
かな?
984ご冗談でしょう?名無しさん:03/04/02 12:34 ID:d2gsyRQS
>>982
分子運動論をいくらこねくり回しても、デビッドアンダーソンの教義は出てこない。
そろそろ、分子運動論を隠れ蓑にするのはやめたらどうか。
985ご冗談でしょう?名無しさん:03/04/02 17:32 ID:+Vjn64NF
986ご冗談でしょう?名無しさん:03/04/02 21:09 ID:ZmVWjOQe
>>985
99匹にすると動かんクなった。てゆうか気持ち悪かった・・・。
ピクピク動くし、退治したら変な液体デルシ…。
987ご冗談でしょう?名無しさん:03/04/02 21:14 ID:8Cdsbjq4
気のせいだ、
気にするな
988ご冗談でしょう?名無しさん:03/04/02 21:50 ID:OBOFoajo
名古屋大学いってこの勉強がしたい
989ご冗談でしょう?名無しさん:03/04/02 23:15 ID:???
>>973
分子間力についてですが,翼から離れたところでは,分子間力は全方向から
働くため,平均として力はゼロになると考えられます。(ただし,翼の近くの
流体分子には,片側からしか力が働かないので,流体の側へ引かれることに
なり,結果として,翼に働く圧力が,分子間力がないときと比べて小さくなる
ことになります。)
したがって,分子間力が気流を曲げるという説明は成り立たないと
思います。
990ご冗談でしょう?名無しさん:03/04/02 23:52 ID:???
>>983
ゴチャゴチャ書いているが、要するに、
 アンダーソンは間違ってませんでした。申し訳ありませんですた。
ってことね。
しかし、
>空気と翼が触れていない場合に相互に力が働かないという
だってさ。
「近似の一つの手段」であるはずなのに、連続体近似がアタマから
離れないところがカワイイね。

>>984
「ボクちゃん、いくらコネクリ回しても出せなかった」って言われてモナー。

>>989
間違っているがキミのレスは好印象だな。何故だろう?
「平均してゼロになる」ってのは「力のベクトル和がゼロ」ってことかな。
それは周りの分子の配置が特別な場合のときだけで、対称でないと
ゼロにはならないよ。力の大きさは距離に関係しているからね。
991ご冗談でしょう?名無しさん
>>990
他人の意見がすべて連続体近似だけだと思い込むところがかわいいね。分子モデル
も元々は化学反応の量関係の説明に起源がある。古典物理の基礎モデルとして相性
が良いとはとてもいえないでしょう。連続体近似のこと批判するんだったら
同時に分子モデル近似も批判してみることだね。
まぁ、大好きな分子モデル近似で表現すれば、>>983の助言は、
空気分子と翼の間に働く斥力が発現するが、その作用距離が空気分子間の
それよりもずっと小さく、一般に空気分子同士には斥力が支配的に働くも
のとするとに読み替えられる。後は自分で考えてみな。